Surgery PACKRAT

Ace your homework & exams now with Quizwiz!

Toxic Megacolon

Key Diagnostic Factors hx of inflammatory bowel disease hx of exposure to infectious agents hx of recent antibiotic use hx of HIV/AIDS/immunosuppressed state fevers/chills tachycardia mental status changes hypotension abdominal distention diarrhea abdominal pain abdominal tenderness

Gastrointestinal/Nutritional: Clinical Therapeutics, Gastroparesis A symptomatic chronic disorder of the stomach defined as delayed emptying of solids by the stomach in the absence of any mechanical obstruction.A condition that affects the stomach muscles and prevents proper stomach emptying

Key symptoms are chronic nausea, vomiting, epigastric pain, bloating, and early satiety. DX Gastric emptying scintigraphy Tx Gut motility stimulator Helps speed the movement of food through the stomach and intestines. Metoclopramide Erythromycin Antiemetics promethazine prochlorperazine maleate ondansetron : gastric electric stimulation surgery

Gastrointestinal/Nutritional : Clinical Intervention, Colon cancer

Localized (stage I-III): surgical resection and regional lymph node dissection ● Stage III & metastatic: chemotherapy mainstay of treatment. Adjuvant Tx for metastatic dz = FOlFOX (5-fluorouracil + leucovorin or levamisole and oxaliplatin) ● CEA are also monitored during treatment

M.E.N. Syndromes: Multiple Endocrine Neoplasia

MEN 1 = Parathyroid tumors, pancreatic tumors, and pituitary tumors. MEN 2a = Medullary thyroid cancers (MTC), pheochromocytoma, and parathyroid tumors. MEN 2b = Medullary thyroid cancers, pheochromocytoma and neuromas. Familial Medullary Thyroid Carcinoma (FMTC) is medullary thyroid cancer

One question showing Mallory Weiss tear patient with history of drinking alcohol and forceful vomiting, Complaining of hematemesis Upper endoscopy or esophagogastroduodenoscopy (EGD) showing superficial longitudinal mucosal lacerations of the gastroesophageal junction

Mallory Weiss tear Mallory-Weiss tears are lacerations of the gastroesophageal junction causing 5-10% of cases of upper GI bleeding. Many patients report a history of heavy alcohol use ,forceful vomiting retching.

Mesenteric ischemia gold statdart diagnostic test

Mesenteric angiography or CT angiography

Dermatology: Clinical Intervention, Necrotizing fasciitis Necrotizing Fasciitis a. lnfxn from skin layers down to fascial planes with severe pain, fever, increased white count, local inflammation may be deceptively absent but pt will appear very ill (i.e., pain out of proportion to exam findings") b. Caused by S. pyogenes (GAS), Clostridium perfringens, and now described with community acquired MRSA infxn c. Tx = immediate, extensive surgical debridement, add penicillin or ceftriaxone (for Streptococcus and Clostridium) +vancomycin (for MRSA) + clindamycin (protein synthesis inhibitor; shuts down the production of toxins that mediate the tissue necrosis)

Necrotizing Fasciitis Tx = immediate, extensive surgical debridement, add penicillin or ceftriaxone (for Streptococcus and Clostridium) +vancomycin (for MRSA) + clindamycin (protein synthesis inhibitor; shuts down the production of toxins that mediate the tissue necrosis)

Pt with pain trauma in the lower leg with with risk of Necrotizing Fasciitis or compartment syndrome how to treat it

Necrotizing Fasciitis Tx = immediate, extensive surgical debridement, add penicillin or ceftriaxone (for Streptococcus and Clostridium) +vancomycin (for MRSA) + clindamycin (protein synthesis inhibitor; shuts down the production of toxins that mediate the tissue necrosis)

Gastrointestinal/Nutritional: History and Physical, Pancreatic cancer

Pancreatic cancer is suggested by the vague epigastric pain with the jaundice resulting from biliary obstruction due to cancer involving the pancreatic head. The presence of a palpable non-tender gallbladder (Courvoisier's sign) also indicates obstruction due to the cancer. o If located in the head of the pancreas weight loss, jaundice, dark urine, light stool color, itching, nausea, vomiting, abdominal pain, back pain, and enlarged lymph nodes in the neck. o If located in the body or tail of the pancreas belly and/or back pain and weight loss.

Gastrointestinal/Nutritional: History and Physical, Pancreatitis Si/Sx = severe abd pain, prostration (fetal position opens up retroperitoneal space and allows more room for swollen pancreas), hypotension (because of retroperitoneal fluid sequestration), tachycardia, fever, serum amylase (90% sensitive)/ lipase, hyperglycemia, hypocalcemia 5. Dx = clinically and/or abd CT, classic x-ray finding = sentinel loop or colon cutoff sign (loop of distended bowel adjacent to pancreas) 6. Classic physical findings= Grey Turner's sign (discoloration of flank) and Cullen's sign (periumbilical discoloration) 7. Tx is aimed at decreasing stress to pancreas a. NPO until symptoms/amylase subside; total parenteral nutrition (TPN) if NPO for >7-10 days b. Demerol to control pain c. IV fluid resuscitation d. Alcohol withdrawal prophylaxis e. May require ICU admission if severe

Pancreatitis SifSx = severe epigastric pain radiating to the back, nausea/vomiting, signs of hypovolemia because of third spacing,"↓bowel sounds • In hemorrhagic pancreatitis, there are ecchymotic appearing skin findings in the flank (Grey Turner's sign) or periumbilical area (Cullen's sign) • Labs -leukocytosis, ↑serum and urine amylase, ↑ lipase • X-ray dilated small bowel or transverse colon adjacent to the pancreas, called "sentinel loop" • CT phlegmon, pseudocyst, necrosis, abscess

Pre-Operative/Post-Operative Care: Clinical Intervention, Abdominal aortic aneurysm

Patients are usually asymptomatic, and their abdominal aortic aneurysm is detected incidentally. patients who experience symptoms, present with palpable pulsatile abdominal mass abdominal, back, and groin pain, and hypotension are typical. Ultrasound remains the definitive test for initial diagnosis and screening. Imaging with computed tomography angiography or magnetic resonance angiography is used for anatomical mapping to assist with operative planning. Patients presenting with a ruptured aneurysm require emergent repair. For patients with symptomatic aortic aneurysms, repair is indicated regardless of diameter. o Open aneurysm repair if aneurysm is causing symptoms or is larger than about 2 inches (roughly 5.0 to 5.5 centimeters) or is enlarging under observation. o Endovascular stent graft if pt is high risk for open aneurysm repair,

Hematology: Diagnostic Studies, clotting factor hemophilia A and B bleeding disorder, usually inherited, characterized by the deficiency of coagulation factor VIII or IX. It causes heavy or unusual bleeding into the joints. Treatment consists of coagulation factor VIII or IX replacement. Von Willebrand's disease is the most common inherited bleeding disorder. It develops when the blood lacks von Willebrand factor, which helps the blood to clot.

Problems with factor VIII and factor IX are known as hemophilia A and B 1st Tests To Order CBC usually normal; low Hb if bleeding has been severe or prolonged activated partial thromboplastin time (aPTT) usually prolonged; plasma factor VIII and IX assay decreased or absent factor VIII or IX levels;

Cardiovascular: Diagnosis, Acute Pericarditis Pericarditis a. Caused by bacterial, viral, or fungal infxns, a b. Si/Sx = retrosternal pain relieved when sitting up, often following upper respiratory infxn (URI), listen for pleural friction rub c. EKG ~ ST elevation in all leads, also see PR depression d. Dx =clinical, confirm with echocardiography e. Tx = NSAIDs for viral, antimicrobial agents for more bacterial/fungal dz, Pericardiectomy for thickened pericardium after chronic infection

Si/Sx = retrosternal pain relieved when sitting up, often following upper respiratory infxn (URI), listen for pleural friction rub c. EKG ~ ST elevation in all leads, also see PR depression d. Dx =clinical, confirm with echocardiography

o Cardiovascular: Diagnostic Studies, Aortic dissection Aortic Dissection 1. Intimal tear through which blood can flow, creating a plane between the intima and remainder of vessel wall 2. Usually confined to thoracic aorta (e.g., syphilis) 3. These planes can progress proximally and distally to disrupt blood supply to intestines, spinal cord, kidneys, and even the coronary vessels 4. In general, type A affects ascending aorta only; type B can affect both ascending and descending aorta 5. Si/Sx =classic severe tearing (ripping) chest pain in hypertensive pts that radiates toward the back 6. Dx =clinical, confirm with CT or aortogram, but if pt unstable take immediately to OR 7. Tx a. Descending aortic dissection is usually medical (e.g., control of HTN and heart rate) unless life-threatening complications arise b. In contrast, ascending dissection ~ immediate surgical intervention with graft placemen

Si/Sx =classic severe tearing (ripping) chest pain in hypertensive pts that radiates toward the back 6. Dx =clinical, confirm with CT or aortogram,

Gastrointestinal/Nutritional : Clinical Intervention, Sigmoid volvulus

Sigmoid volvulus ● Endoscopic decompression initial tx of choice. Surgical correction 2nd line

Cardiovascular: History and Physical, Syncope, Subclavian steal syndrome o Presyncope/Syncope/Neurologic deficits/Blood pressure differential between the arms/Severe memory problems o Arm claudication: poor circulation in the hands(blotchy patches of red and white) (associated with other stigma to vascular disease (e.g. vascular insufficiency ulcers of the foot). ● Retrograde blood flow in a vertebral artery associated with proximal ipsilateral subclavian artery stenosis or occlusion

Subclavian Steal Syndrome 1. Caused by occlusive lesion in subclavian artery causing decreasing blood flow distal to the obstruction 2. This results in the "stealing" of blood from vertebral artery via retrograde flow 3. Si/Sx = Neurologic deficits, syncope, vertigo, nausea, confusion/Severe memory problem, / arm claudication, Blood pressure differential between the arms and supraclavicular bruits 4. Dx =angiogram, Doppler, MRI 5. Tx =carotid-subclavian bypass

Gastrointestinal/Nutritional : Diagnostic Studies toxic megacolon Toxic Megacolon o ‪Most common cause: inflammatory bowel disease‬‬ o ‪Colon dilated > 6 cm on radiographs‬‬ o ‪Systemic toxicity‬‬ o ‪High risk of perforation‬‬ ‪Rx: IVF, ABX, IV corticosteroids, emergent surgical consultation

Symptoms and signs of systemic toxicity are always present. Criteria for the diagnosis include radiographic evidence of acute colitis and 3 of the following 4 features: fever >101.5°F (38.6°C), heart rate >120 bpm, WBC count >10,500/microliter, or anemia. One of the following is also required: volume depletion, mental status changes, electrolyte abnormalities, or hypotension. stool studies may be positive for infectious organism CT abdomen/pelvis diffuse colonic wall thickening, submucosal edema, pericolic fat stranding, colonic dilatation, absence of normal haustral markings abdominal x-ray massively dilated colon chest x-ray free air under the diaphragm indicates perforation

Endocrinology: Diagnostic Studies, Hyperthyroidism Si/Sx of hyperthyroidism = tachycardia, isolated systolic hypertension, tremor, A-fib, anxiety, diaphoresis, weight loss with i appetite, insomnia/fatigue, diarrhea, exophthalmus, heat intolerance

TSH low serum free T4 elevated, serum free or total T3 elevated increased uptake on radionuclide scan.

Gastrointestinal/Nutritional: Diagnosis, Meckel diverticulum o Meckel's diverticulum results from the failure of the vitelline duct to separate from the intestine during early gestation leading to an outpouching in the intestinal tract. Si/Sx = GI bleed (melena, hematochezia), small bowel obstruction (intussusception, Littre's hernia), Meckel's diverticulitis (similar presentation to appendicitis)

Technetium scan ("Meckel scan") o If active bleeding colonoscopy or upper GI endoscopy 1st Tests To Order CBC low Hb and Hct; leukocytosis with left shift technetium-99m pertechnetate scan ("Meckel scan") ectopic focus or "hot spot"; enhancement of diverticulum plain abdominal radiography

Pulmonology : Diagnosis, Chronic bronchitis

The criteria for chronic bronchitis are a daily productive cough for three consecutive months for at least two consecutive years. This patient most likely has chronic bronchitis which is defined as sputum production and cough for most of the days for at least 3 months of the year for 2 consecutive years which is primarily caused by cigarette smoking. Dx clinical, confirmed by lung biopsy ↑Reid index (gland layer is > 50% of total bronchial wall thickness) Pulmonary function test

o Dermatology: Diagnosis, Basal cell skin cancer

The lesion of basal cell carcinoma is typically firm, round, and pearly or waxy. It is the most common cutaneous neoplasm in humans, with 85% of them occurring on the head or neck. Margin telangiectasis is classically associated with basal cell carcinomas. Risk factors include fair skin, excessive sun exposure, and male gender. BCC should be biopsied with a punch or shave biopsy. TX excisional

Patient with thoracic-abdominal surgery present with pain or numbness in the fourth and fifth fingers

The ulnar nerve is a major nerve in the arm and provides sensation to the fourth and fifth finger.

Gastrointestinal/Nutritional : History and Physical, Esophageal stricture or Peptic stricture An esophageal stricture is a narrowing of the esophagus. Causes Esophageal stricture can be caused by: Gastroesophageal reflux (GERD). Eosinophilic esophagitis. Injuries caused by an endoscope. Caused by an underlying infection(esophagitis) Long-term use of a nasogastric (NG) tube Swallowing substances that harm the lining of the esophagus. Symptoms of esophageal strictures include trouble swallowing Pain with swallowing Unintentional weight loss Regurgitation of food, heartburn, bitter or acid taste in the mouth. Exams and Tests Barium swallow X-ray while the patient swallows barium to look for narrowing of the esophagus Endoscopy establishes Dx Treatment Dilation (stretching) of the esophagus is the main treatment for acid reflux related strictures. Proton pump inhibitors (acid-blocking medicines) can keep a peptic stricture from returning. Surgery is rarely needed. If it is caused by esophagitis, in turn caused by an underlying infection, it is commonly treated by treating the infection (typically with antibiotics). surgical opening if unresponsive

(1) Results in gradual solid food dysphagia often with a concurrent improvement of heartburn Sx Exams and Tests Barium swallow X-ray while the patient swallows barium to look for narrowing of the esophagus Endoscopy establishes Dx

Dermatology: Diagnosis, Decubitus ulcer Stage III - A full-thickness loss of skin with extension into subcutaneous tissue but not through the underlying fascia

(Suspected) deep tissue injury - A purple or maroon localized area of discolored intact skin or blood-filled blister due to damage of underlying soft tissue from pressure or shear " Stage I - ntact skin with nonblanchable redness of a localized area usually over a bony prominence. Darkly pigmented skin may not have visible blanching; its color may differ from the surrounding area. " Stage II - A partial-thickness loss of skin involving epidermis and dermis " Stage III - A full-thickness loss of skin with extension into subcutaneous tissue but not through the underlying fascia Subcutaneous fat may be visible but bone, tendon, or muscle are not exposed. " Stage IV - A full-thickness tissue loss with extension into muscle, bone, tendon, or joint capsule " Unstageable - A full-thickness tissue loss in which the base of the ulcer is covered by slough or eschar to such an extent that the full depth of the wound cannot be appreciated

69. Scientific Concepts/Cardiology A patient presents with moderate mitral stenosis. Which of the following complications is associated with an increased risk of systemic embolization in this patient? A. atrial fibrillation B. pulmonary hypertension C. increased left atrial pressure D. left ventricular dilatation

(c) A. 50-80% of patients with mitral stenosis will develop paroxysmal or chronic atrial fibrillation; 20-30% of patients with atrial fibrillation will have systemic embolization.

History & Physical/Urology/Renal Which of the following is often described as feeling like a "bag of worms" on the testicular exam? A. Varicocele B. Testicular torsion C. Epididymal cyst D. Hydrocele

(c) A. A collection of varicose veins of the spermatic cord is known as a varicocele. The varicosities feel like a bag of worms when palpated on physical examination.

History & Physical/Gastrointestinal/Nutritional A 68-year-old male presents with jaundice, weight loss, and boring abdominal pain which radiates to the back. The gallbladder is palpable on physical examination. This finding is most consistent with which of the following? A. Pancreatic Tumor B. Hemorrhagic pancreatitis C. Cholecystitis D. Cholelithiasis

(c) A. A large palpable gallbladder resulting from pressure from a tumor in the pancreatic head is known as Courvoisier's sign.

What examination finding would be expected in a patient with von Willebrand disease? A. Gingival bleeding B. Splenomegaly C. Muscle weakness D. Hemarthrosis Explanations

(c) A. A patient with von Willebrand disease most commonly presents with mucosal bleeding seen in epistaxis, gingival bleeding, and menorrhagia.

What is considered the most common physical examination finding for intestinal obstruction? A. Distention B. Fluid wave C. Rigidity D. Tenderness

(c) A. Abdominal distention is the most common hallmark of all kinds of intestinal obstructions though its presence can be variable depending on the duration and exact location of the obstruction.

78. Diagnosis/Endocrinology A 30 year-old female complains of fatigue, weakness, diminished appetite, weight loss,and syncope. She denies fever, chest or abdominal pain, palpitations, changes in bowel patterns or sleep patterns. Physical examination reveals a thin female, BP 90/65 mmHg, and pulse 80 beats per minute. Pulmonary, cardiovascular, abdominal, and neurologic exam are without abnormalities. Areas of brown and bronze hyperpigmentation are noted on her elbows and the creases of her hands. Which of the following is the most likely diagnosis? Answers A. Addison's disease B. Cushing's disease C. Anorexia nervosa D. Porphyria Explanations .

(c) A. Addison's disease (adrenal insufficiency) would account for all her symptoms, the hypotension, and the hyperpigmentation of the skin

Scientific Concepts/Dermatology Which of the following is the most common type of skin cancer? A. basal cell B. melanoma C. atypical nevi D. squamous cell Explanations

(c) A. Basal cell cancer is the most common cause of skin cancer usually occurring on sun-exposed areas.

A 30-year-old male has a history of weakness without pain on the left side of his face for the past 4 days. Physical examination of the face reveals unilateral weakness to the left side, but not complete paralysis. The left eye does not close completely. The most appropriate initial treatment is to A. reassure the patient and provide patient education. B. recommend a nonsteroidal anti-inflammatory medication. C. recommend tricyclic antidepressants. D. recommend physical therapy.

(c) A. Bell's palsy will resolve completely without treatment in approximately 60% of all cases.

Clinical Intervention/Gastrointestinal/Nutritional The treatment of choice for patients with celiac disease is which of the following? Answers A. Gluten-free diet B. Small bowel resection C. Clindamycin D. Whipple procedure

(c) A. Celiac disease is a diffuse disease of the small bowel that is caused by immunologic malfunction. This disease is active only in the presence of gluten, a constituent of wheat. Avoidance of gluten-containing foods is the treatment of choice. Steatorrhea is usually present in mild disease of celiac sprue

Diagnosis/Gastrointestinal/Nutritional A 35-year-old female 2 months status-post gastric bypass surgery develops abdominal cramps, diarrhea, palpitations, sweating, and nausea shortly after eating. Which of the following is the most likely diagnosis? A. Dumping syndrome B. Inflammatory bowel disease C. Surgical Adhesions D. Zollinger-Ellison syndrome

(c) A. Gastric bypass puts this patient at risk for dumping syndrome. Her symptoms represent both cardiovascular and gastrointestinal reactions which occur after eating. The underlying effect is due to the stomach's inability to regulate its rate of emptying.

166. History & Physical/Cardiology A 26-year-old male presents with increased dyspnea with exercise. He has noted a decrease in his exercise tolerance over the past several months. He denies chest pain or skipped heartbeats. An echocardiogram reveals left ventricular hypertrophy with asymmetric septal hypertrophy. Ejection fraction is 65%. Which of the following is the most likely presenting history or physical exam finding? A. He has an older brother with the same diagnosis. B. An S3 gallop is heard. C. Patient notes completing a course of adriamycin. D. Elevated jugular venous distension is noted. Explanations

(c) A. Hypertrophic cardiomyopathy can be genetic and present in 25% of first degree relatives.

Clinical Therapeutics/Gastrointestinal/Nutritional A 56-year-old male with a history of hypertension and alcohol use presents with mid-epigastric pain and coffee-ground emesis since early this morning. The patient denies diarrhea, constipation or blood in his stools. Endoscopic evaluation is done after admission revealed peptic ulcer disease with active bleeding. Which of the following is the most appropriate therapy to reduce the risk of rebleeding in this patient? A. IV pantoprazole (Protonix) B. Oral hyoscyamine sulfate (Levsin) C. IV famotidine (Pepcid) D. Oral metoclopramide (Reglan)

(c) A. IV proton pump inhibitors reduce the risk of rebleeding after endoscopic treatment in patients with peptic ulcers that have high-risk features such as active bleeding.

NSAID-induced peptic ulcer disease may be prevented by the administration of A. misoprostol (Cytotec). B. omeprazole (Prilosec). C. ranitidine hydrochloride (Zantac). D. bismuth subsalicylate (Pepto-Bismol).

(c) A. NSAIDs inhibit prostaglandin synthesis and cause a deficiency of prostaglandins. Misoprostol, an E-class prostaglandin, is cytoprotective.

131. Clinical Intervention/Gastrointestinal/Nutritional A 45-year-old female presents with bowel movements associated with bright red blood after being diagnosed with a stage II hemorrhoid 1 month ago. She has used increased fiber and fluid intake as her primary treatment up to this point. Which of the following is the most appropriate next step in managing this patient's condition? A. Rubber band ligation B. Hemorrhoidectomy C. Diltiazem (Cardizem) therapy D. Iron supplementation

(c) A. Patients who have a stage I, II or III hemorrhoid with recurrent bleeding not responsive to conservative therapy should have injection sclerotherapy, rubber band ligation or application of electrocoagulation. Rubber band ligation is easy to use and has a high efficacy rate. (u) B. Hemorrhoidectomy is indicated in patients with a stage III or IV hemorrhoid with chronic bleeding or a stage II hemorrhoid that is acutely thrombosed.

174. Clinical Intervention/Endocrinology A 32-year-old male with a history of pheochromocytoma is seen in the office. The patient is scheduled for adrenalectomy, however, has developed a throbbing headache and racing heart. Vital signs reveal pulse 126 bpm, blood pressure 160/115 mmHg, and respiratory rate 20. The patient appears diaphoretic and anxious. Which of the following is the most appropriate acute management in this patient? A. Oral Phenoxybenzamine (Dibenzyline) B. Hydrochlorothiazide (Diuril) C. Lisinopril (Prinivil) D. Bumetanide (Bumex) Explanations

(c) A. Phenoxybenzamine is an alpha-blocker utilized to control hypertension in patient with a pheochromocytoma.

Urinary retencion : Hydronephrosis Prerenal Postrenal

(c) A. Prerenal azotemia can present as nonspecific symptoms including nausea, vomiting, malaise, and confusion. Prerenal causes are most common and include hypovolemia, which is the likely etiology in this case as her BUN: Creatinine ratio is consistent with dehydration. (u) B. Postrenal azotemia is relatively uncommon and can occur as the result of obstruction of urine flow from both kidneys. Causes include ureteral obstruction and bladder obstruction. Kidney stones (or in men, BPH) are a common cause. (u) C. Intrinsic renal failure occurs as a direct result of insult to the kidney. It should be considered if pre- or postrenal azotemia is not the appropriate diagnosis. Hydronephrosis is caused by a blockage in the tube that connects the kidney to the bladder (ureter). Possible causes include a kidney stone, an infection, an enlarged prostate, a blood clot, or a tumor.

Scientific Concepts/Urology/Renal Which of the following is a cause of acute kidney failure due to prerenal azotemia? A. Excessive diuresis B. Urinary tract obstruction C. Radiologic contract media D. Aminoglycosides Explanations

(c) A. Prerenal azotemia is due to renal hypoperfusion which can occur with intravascular volume depletion such as excessive diuresis, hemorrhage, and gastrointestinal losses.

150. Clinical Intervention/Cardiology A patient with renal artery stenosis is unresponsive to medical therapy. Which of the following is the next most appropriate intervention? A. Stenting of the renal artery B. Nephrectomy C. Radioactive iodine D. Lifestyle modifications

(c) A. Stenting of the renal artery will dilated the narrowed area interfering with blood supply to the kidney.

A patient with renal artery stenosis is unresponsive to medical therapy. Which of the following is the next most appropriate intervention? A. Stenting of the renal artery B. Nephrectomy C. Radioactive iodine D. Lifestyle modifications

(c) A. Stenting of the renal artery will dilated the narrowed area interfering with blood supply to the kidney.

Diagnosis/Endocrinology A 53-year-old female who is well known to the practice presents to the office complaining of increasing fatigue, constipation, and a weight gain of 10 lb (4.5 kg) over the past year. She also states others have noticed a recent hoarseness to her voice, and she is bothered by "charley horses" in her legs that wake her up at night. Her past medical history is unremarkable except for a history of hyperthyroidism treated by radioactive iodine 5 years ago. She is currently taking no medications and has no known drug allergies. Which of the following is the most likely cause of the patient's symptoms? A. Hypothyroidism B. Hypoparathyroidism C. Vocal cord paralysis D. Radiation thyroiditis

(c) A. The current symptoms, along with the past treatment of hyperthyroidism with radioactive iodine, would indicate hypothyroidism.

A 42 year-old male presents complaining of a sudden onset of a severe intermittent pain originating in the flank and radiating into the right testicle. He also complains of nausea and vomiting. On examination the patient is afebrile, but restless. Examination of the abdomen reveals tenderness to palpation along the right flank with no rebound or direct testicular tenderness. Urinalysis reveals a pH of 5.4 and microscopic hematuria, but is otherwise unremarkable. Which of the following is the most likely diagnosis? A. Bladder cancer B. Nephrolithiasis C. Acute appendicitis D. Acute epididymitis

(c) B. A sudden onset of severe colicky flank pain associated with nausea and vomiting as well as the absence of rebound or direct testicular tenderness makes nephrolithiasis the most likely diagnosis. This is further supported by the presence of hematuria on the urinalysis.

207. History & Physical/Gastrointestinal/Nutritional An afebrile 22-year-old female presents with 2 days of a painful rectal mass which worsens with defecation. She denies any blood with bowel movements. She admits to ongoing constipation and generally poor diet of fast food. She is otherwise healthy. On examination, there is a tender, swollen, bluish, ovoid mass visible at the anal margin. Which of the following is the most likely diagnosis? A. Rectal prolapse B. Thrombosed Hemorrhoid C. Internal Hemorrhoid D. Rectal polyp

(c) B. A thrombosed hemorrhoid causes acute pain increased with defecation and sitting.

A patient on acetazolamide for glaucoma complains of increasing shortness of breath. The patient denies a cough, chest pain, or fever. Physical examination is unremarkable except for an increased respiratory rate of 30/minute. Labs include Na 132 mEq/L, K 5.6 mEq/L, Cl 120 mEq/L, and CO2 10 mEq/L. Arterial blood gas results are pH 7.18, pO2 98 mmHg, pCO2 22 mmHg, and HCO3 8 mEq/L. Based upon these results, what is the diagnosis? Answers A. Metabolic alkalosis B. Metabolic acidosis C. Respiratory alkalosis D. Respiratory acidosis

(c) B. Acetazolamide inhibits carbonic anhydrase, which inhibits bicarbonate ion regeneration, with resulting normal anion-gap, hyperchloremic acidosis. Respiratory compensatory mechanisms lead to hyperventilation.

70 year old man complains of abdominal pain of four hours duration. He states that he has vomited twice since the onset of pain. He also complains of three days of constipation. He is afebrile and the physical examination is noteworthy for a distended, diffusely tender abdomen with normoactive bowel sounds. His rectal exam reveals hemoccult positive brown stool. Medications include omeprazole (Prilosec) for GERD, digoxin and warfarin (Coumadin) for atrial fibrillation, OTC multivitamins and stool softeners. The abdominal and chest x-rays show no abnormalities. Which of the following is the most likely diagnosis? A. Acute cholecystitis B. Mesenteric infarction C. Perforated duodenal ulcer D. Small bowel obstruction

(c) B. Acute onset of severe diffuse abdominal pain in a person with atrial fibrillation warrants the suspicion of mesenteric infarction. Vomiting and constipation may be seen, along with occult blood in the stool. Bowel sounds may be normal.

A 72-year-old male presents to the ED complaining of acute onset of severe diffuse abdominal pain of four hours duration. He states that he has vomited twice since the onset of pain. He also complains of three days of constipation. He is afebrile and the physical examination is noteworthy for a distended, diffusely tender abdomen with normoactive bowel sounds. His rectal exam reveals hemoccult positive brown stool. Medications include omeprazole (Prilosec) for GERD, digoxin, and warfarin (Coumadin) for atrial fibrillation, OTC multivitamins and stool softeners. The abdominal and chest x-rays show no abnormalities. Which of the following is the most likely diagnosis? A. Acute cholecystitis B. Mesenteric infarction C. Perforated duodenal ulcer D. Small bowel obstruction

(c) B. Acute onset of severe diffuse abdominal pain in a person with atrial fibrillation warrants the suspicion of mesenteric infarction. Vomiting and constipation may be seen, along with occult blood in the stool. Bowel sounds may be normal.

A 72 year-old woman presents to your clinic complaining of constipation. Which of the following presentations would be most concerning? A. Fewer than one stools per week B. Weight loss C. Recurrent hemorrhoids D. Scybala

(c) B. Alarm symptoms are concerning for colorectal malignancy and include weight loss, anemia, hematochezia, or positive fecal occult blood test. These symptoms are particularly worrisome in a patient with a family history of colorectal cancer.

141. Clinical Intervention/Endocrinology In the treatment of alcohol-related hypoglycemia, thiamine is administered before glucose to prevent which of the following? A. Acute renal failure B. Wernicke's encephalopathy C. Korsakoff psychosis D. Mesenteric thrombosis

(c) B. Alcoholics are typically deficient in thiamine. If glucose alone is given to treat hypoglycemia, Wernicke encephalopathy can be precipitated since thiamine is not available for nutritional purposes when glucose is replaced. Therefore, thiamine is always given prior to glucose until a satisfactory diet can be given in order to prevent Wernicke's encephalopathy.

A 56 year-old male with a known history of polycythemia suddenly complains of pain and paresthesia in the left leg. Physical examination reveals the left leg is cool to the touch and the toes are cyanotic. The popliteal pulse is absent by palpation and Doppler. The femoral pulse is absent by palpation but weak with Doppler. The right leg and upper extremities has 2+/4+ pulses throughout. Given these findings what is the most likely diagnosis? A. Venous thrombosis B. Arterial thrombosis C. Thromboangiitis obliterans D. Thrombophlebitis

(c) B. Arterial thrombosis has occurred and is evidenced by the loss of the popliteal and dorsalis pedis pulse. This is a surgical emergency. Venous occlusion and thrombophlebitis do not result in loss of arterial pulse.

Diagnosis/Pulmonology A 60-year-old patient returned from the recovery room to the floor following a subtotal gastrectomy. At 3 AM the next morning, the patient's temperature is 102° F (39° C) and pulse is 112/min. Which of the following is the most likely cause? A. wound infection B. atelectasis C. phlebitis D. shock Explanations

(c) B. Atelectasis is the most common pulmonary complication, affecting 25% of patients with abdominal surgery. It is more common in elderly and overweight patients and occurs within the first 12 to 24 hours postoperatively.

Treatment of Bell's palsy includes which of the following? A. acyclovir 400 mg tid B. reassurance of the patient's recovery C. referral to a neurosurgeon D. electromyography

(c) B. Bell's palsy is a peripheral neuropathy of cranial nerve VII. Although it has been suggested to be related to an activation of herpes simplex virus, there is little empiric evidence for this. Approximately 60% of cases of Bell's palsy recover without treatment and patient reassurance of this is advised. Electromyography may provide aid in the prognosis, but not as a treatment option. A neurosurgeon has no role in the management of Bell's palsy.

A 54 year-old female who has diabetes presents with rubor, absence of hair, and brittle nails of her left foot. She complains of leg pain that awakens her at night. Examination reveals a femoral bruit with diminished popliteal and pedal pulses on the left side. The most appropriate therapy would be A. vasodilator therapy. B. bypass surgery. C. exercise program. D. embolectomy.

(c) B. Bypass surgery is indicated in the presence of rest pain (leg pain that awakens her at night) and provides relief of symptoms in 80 to 90% of patients. (u) C. While an exercise program is appropriate with claudication, rest pain is a surgical indication. (u) D. Embolectomy is used for acute arterial occlusion

A 65 year-old patient with adenocarcinoma of the colon in remission presents for follow-up. Which of the following tumor markers should be monitored? A. AFP B. CEA C. CA 19-9 D. CA-125

(c) B. CEA is used to monitor recurrence of colon carcinoma. (u) C. CA 19-9 is used to monitor recurrence of pancreatic carcinoma. (u) D. CA-125 is used to monitor recurrence of ovarian carcinoma

An 18 year-old male who was struck in the left eye with a baseball presents with ocular pain, periorbital ecchymosis, and restricted upward gaze. Which of the following is the diagnostic study of choice in this patient? A. Zygomatic arch x-ray B. CT scan of the orbit C. Ultrasonography D. Fluorescein staining

(c) B. CT scan of the orbit is the study of choice to evaluate a suspected blowout fracture.

Diagnostic Studies/Gastrointestinal/Nutritional Which of the following tumor markers is useful in monitoring a patient for recurrence of colorectal cancer after surgical resection? A. CA-125 B. Carcinoembryonic antigen(CEA) C. 5-hydroxindoleacetic acid D. Alpha-1-fetoprotein

(c) B. Carcinoembryonic antigen(CEA) can be used to monitor a patient for the return of colorectal cancer after treatment.

146. Diagnosis/Cardiology A patient presents with chest pain that initially occurred three days ago. The EKG shows loss of R wave progression across the anterior chest leads. Which of the following is the best explanation for this finding? A. Unstable angina B. Acute myocardial infarction C. Prinzmetal's angina D. Hyperventilation

(c) B. Chest pain that is ongoing and that has been present for several days may be due to ongoing infarction or recurrent infarction. EKG changes revealing loss of R waves across the precordium may reflect an acute anterior wall myocardial infarction.

History & Physical/Gastrointestinal/Nutritional A 45-year-old type 2 diabetic female with a history of cholelithiasis presents to the clinic with 2-3 episodes of sudden, severe epigastric pain that radiates to her shoulder. She has associated nausea and vomiting. Temperature is 101 degrees F and she is experiencing chills. Today her eyes appear yellow in color. Which of the following is the most likely diagnosis for this patient A. Postcholecystectomy syndrome B. Cholangitis C. Gastroesophageal reflux disease D. Pancreatic cancer

(c) B. Cholangitis is characterized by a history of biliary pain, fever, chills, and jaundice associated with episodes of abdominal pain Cholangitis Choledocholithiasis + infection Charcot triad: RUQ pain+fever w/chills+jaundice Reynolds pentad is a collection of signs and symptoms suggesting the diagnosis obstructive ascending cholangitis, a serious infection of the biliary system. It is a combination of Charcot's triad (right upper quadrant pain, jaundice, and fever) with shock (low blood pressure, tachycardia) and an altered mental status

45 year-old type 2 diabetic female with history of cholelithiasis presents to the clinic with 2-3 episodes of sudden, severe epigastric pain that radiates to her shoulder. She has associated nausea and vomiting. Temperature is 101 degrees F and she is experiencing chills. Today her eyes appear yellow in color. Which of the following is the most likely diagnosis for this patient? A. Postcholecystectomy syndrome B. Cholangitis C. Gastroesophageal reflux disease D. Pancreatic cancer

(c) B. Cholangitis is characterized by a history of biliary pain, fever, chills, and jaundice associated with episodes of abdominal pain.

Scientific Concepts/Pulmonology Which of the following pathophysiological processes is associated with chronic bronchitis? A. Destruction of the lung parenchyma B. Mucous gland enlargement and goblet cell hyperplasia C. Smooth muscle hypertrophy in the large airways D. Increased mucus adhesion secondary to reduction in the salt and water content of the mucus

(c) B. Chronic bronchitis results from the enlargement of mucous glands and goblet cell hypertrophy in the large airways

3. DIAGNOSIS A 60-year-old male presents with a recent history of change of bowel habits, weight loss, and blood and mucus in his stools. The most likely diagnosis is A. hemorrhoids. B. colorectal carcinoma. C. acute diverticulitis. D. fistula-in-ano.

(c) B. Colorectal cancer presents with weight loss, change of bowel habits, and blood in stool.

A 60-year-old male presents with a recent history of change of bowel habits, weight loss, and blood and mucus in his stools. The most likely diagnosis is A. hemorrhoids. B. colorectal carcinoma. C. acute diverticulitis. D. fistula-in-ano.

(c) B. Colorectal cancer presents with weight loss, change of bowel habits, and blood in stool.

CLIN INTERVENTION An elderly patient with long-standing Type 2 diabetes and renal disease develops a fever of 102° F orally, productive cough, and dyspnea. Physical examination demonstrates a respiratory rate of 32/min, labored breathing and rales at the left base. Which of the following is the next appropriate step in the management of this patient? A. administer nebulized corticosteroids B. admit to the hospital C. begin oral antimicrobial therapy D. endotracheal aspiration

(c) B. Community-acquired pneumonia is the most deadly infectious disease in the U.S. Important risk factors for increased morbidity and mortality include advanced age, alcoholism, comorbid medical conditions, altered mental status, respiratory rate greater than 30 breaths/min, hypotension, and a BUN greater than 30.

A 16-year-old male is found on physical examination to have a history of cryptorchidism of the right testes. This was surgically corrected at age 18 months. This patient should be monitored for the development of A. prostatic cancer. B. testicular cancer. C. bladder cancer. D. penile cancer.

(c) B. Cryptorchidism increases the risk of testicular cancer. In fact, the major predisposing risk factor is cryptorchidism unrepaired until after age two and Caucasian men at highest risk

Which of the following studies is most appropriate to diagnose celiac disease? A. Urinary D-xylose test B. Small bowel biopsy C. Barium contrast x-ray D. Schilling test

(c) B. Definitive diagnosis of celiac disease is made by small bowel biopsy.

A 60 year-old female presents for pre-op evaluation for surgical treatment of a tri-malleolar fracture of the left ankle. Which of the following puts her at an increased risk for infection post-surgery? A. Recent URI B. Diabetes mellitus C. Heart murmur D. Swelling of the ankle

(c) B. Diabetes mellitus puts her at risk for infection post surgery.

A 26 year-old female required 12 units packed red blood cells during a trauma resuscitation and surgical repair of liver and splenic lacerations. The patient is now 6 hours postoperative and has blood oozing from the suture line and IV sites. There is bloody urine in the Foley bag. Laboratory evaluation demonstrates a platelet count of 10,000/microliter, prolonged prothrombin level, and the presence of fibrin split products. Which of the following is the most likely diagnosis? A. acute ABO incompatibility reaction B. disseminated intravascular coagulation C. exacerbation of idiopathic thrombocytopenia D. inadequate repair of the liver lacerations

(c) B. Disseminated intravascular coagulation is characterized by bleeding from many sites as all coagulation factors are consumed and then broken down, leading to decreased fibrinogen level and platelet count, prolonged PT and PTT, and presence of fibrin split products.

Diagnosis/Hematology A 26-year-old female required 12 units packed red blood cells during a trauma resuscitation and surgical repair of liver and splenic lacerations. The patient is now 6 hours postoperative and has blood oozing from the suture line and IV sites. There is bloody urine in the Foley bag. Laboratory evaluation demonstrates a platelet count of 10,000/microliter, prolonged prothrombin level, and the presence of fibrin split products. Which of the following is the most likely diagnosis? A. acute ABO incompatibility reaction B. disseminated intravascular coagulation C. exacerbation of idiopathic thrombocytopenia D. inadequate repair of the liver lacerations

(c) B. Disseminated intravascular coagulation is characterized by bleeding from many sites as all coagulation factors are consumed and then broken down, leading to decreased fibrinogen level and platelet count, prolonged PT and PTT, and presence of fibrin split products.

6. CLIN THERAPEUTICS A 75-year-old patient is 48 hours status post lumbar laminectomy and develops a fever, cough with purulent sputum, and leukocytosis. Chest x-ray demonstrates right lower lobe consolidation with air bronchograms. Which of the following is the most appropriate treatment? A. Await results of sputum cultures. B. Administer antibiotics. C. Administer prednisone. D. Begin chest physiotherapy.

(c) B. Empirical therapy for mild to moderate nosocomial pneumonia may consist of a second-generation cephalosporin because of the high mortality rate with nosocomial pneumonia, treatment must be empiric and cannot wait for the results of sputum cultures.

Scientific Concepts/Gastrointestinal/Nutritional A 42 year-old male with chronic complaints of heartburn and regurgitation presents for follow-up after undergoing endoscopic evaluation which shows evidence of Barrett's esophagus. Which of the following is the most serious potential complication in this patient? A. Peptic stricture B. Esophageal adenocarcinoma C. Esophageal web formation D. Mallory-Weiss tear

(c) B. Esophageal carcinoma is the most serious complication of Barrett's esophagus.

144. Clinical Intervention/Gastrointestinal/Nutritional A 77-year-old male presents with a two-week history of a cough. The patient also complains of a bad taste in his mouth. He complains of mild midsternal chest pressure but denies any dyspnea. He denies any dysphagia or history of recent URI. Evaluation of the cardiac and pulmonary systems, including pulmonary function tests and exercise stress testing, is normal. Upper endoscopy is normal. Which of the following is the next step in the evaluation of this patient? Answers A. Upper GI study with barium B. Esophageal pH monitoring C. CT of the thorax D. D-xylose test of the stool Explanations

(c) B. Esophageal pH monitoring records the pH of the lower esophagus on a 24-hour continuous basis. This may be the only abnormality that occurs in a patient with GERD who presents with coughing as the chief complaint.

An 18 month-old female presents to the Emergency Department having possibly swallowed a hearing aid battery within the past hour. She is drooling and appears anxious but parents have noticed no stridor or dyspnea. She has no history of previous esophageal injury. Physical examination is unremarkable. Chest radiograph reveals a radiopaque round object at the distal esophagus. Which of the following is the most appropriate treatment option? A. Observation for 24 hours B. Esophagoscopy for removal C. Barium swallow D. Bronchoscopy

(c) B. Esophagoscopy is the procedure of choice for acutely ingested foreign bodies.

A 40 year-old patient with a history of recurrent kidney stones presents with acute onset of right flank pain and hematuria. The patient is afebrile and pain is poorly controlled on oral medications. On CT scan a 1 cm stone is noted in the renal pelvis. Which of the following is the most appropriate intervention for this patient? A. Antibiotics B. Shock wave lithotripsy C. Ureterolithotomy D. Fluid hydration

(c) B. Extracorporeal shock wave lithotripsy is indicated in patients with stones greater than 6 mm in size or intractable pain.

CLIN THERAPEUTICS In the treatment of simple hypochromic, microcytic anemia due to chronic blood loss, the treatment of choice is which of the following? A. vitamin B12 B. ferrous sulfate C. folic acid D. erythropoietin

(c) B. Ferrous sulfate is the treatment of choice for hypochromic, microcytic anemia due to iron deficiency from the blood loss.

130. Health Maintenance/Cardiology A new patient with a history of hypertension on verapamil (Calan) presents for routine examination. Electrocardiogram reveals irregular R-R intervals, with narrow QRS complexes. There are no definable P-waves. Which of the following is most appropriate for this patient? A. Annual echocardiogram B. Warfarin (Coumadin) therapy C. Internal defibrillator D. Infective endocarditis prophylaxis

(c) B. For patients with atrial fibrillation, even when it is paroxysmal or occurs rarely, anticoagulation with warfarin to an INR target of 2.0-3.0 should be established and maintained indefinitely.

219. Clinical Intervention/Cardiology Patients who undergo percutaneous angioplasty or who have coronary artery revascularization often are treated with glycoprotein IIb/IIIa inhibitors. What is the major side effect associated with these agents? A. Hypotension B. Bleeding C. Coronary vasospasm D. Acute renal failure

(c) B. Glycoprotein IIb/IIa inhibitors have their activity in the final stages of platelet bridging and are associated with bleeding when used in the management of acute myocardial infarction. Since they are effective at treating and preventing new clot formation, bleeding is the main concern and complication with the use of these agents.

Scientific Concepts/Gastrointestinal/Nutritional A 40-year-old male presents with several months of abdominal pain and nausea. Endoscopy reveals an irregular 1 x 2 cm area of loss of rugal folds near the antrum. Biopsies show extensive mucosal and submucosal infiltration by Blymphocytes.The patient is given appropriate treatment and the lesion resolves. Which infectious agent is most likely to be associated with these findings? A. Aspergillus fumigatus B. Helicobacter pylori C. Human papilloma virus D. Salmonella typhi

(c) B. H. Pylori causes gastric mucosal inflammation with PMN's and lymphocytes. Infection causes nausea and abdominal pain. Inflammation may be confined to the superficial gastric epithelium or may extend deeper resulting in varying degrees of gland atrophy. Eradication of H. Pylori with appropriate therapy leads to resolution of the chronic gastritis.

What is the pathologic mechanism of Hirschsprung's disease? A. Pyloric circular muscle hypertrophy causing gastric outlet obstruction B. Absence of ganglion cells in the mucosal and muscular layers of the colon C. A defect in the diaphragm leading to protrusion of the abdominal viscera into the thoracic cavity D. Absence of relaxation of the lower esophageal sphincter and lack of peristalsis in the esophageal body

(c) B. Hirschsprung's disease results from an absence of ganglion cells in the mucosal and muscular layers of the colon.

46. History & Physical/Cardiology Which of the following physical examination findings will be associated with hypertrophic cardiomyopathy? A. Murmur will worsen with squatting B. Murmur will decrease with deep breathing C. Fixed splitting of S2 D. Narrowed S2

(c) B. In hypertrophic cardiomyopathy, maneuvers that reduce left ventricular volume, such as standing, further narrow the left ventricular outflow and increase the degree of obstruction. An increase in the left ventricular volume that occurs with squatting will expand the outflow tract and reduce the pressure gradient decreasing the murmur.

Health Maintenance/Dermatology Which of the following is considered a risk factor for the development of malignant melanoma? A. male gender B. inability to tan C. Japanese ethnicity D. brown-haired individuals

(c) B. Inability to tan and propensity to burn are risk factors for developing malignant melanoma.

Which of the following is the most common indication for operative intervention in patients with chronic pancreatitis? A.Weight loss B. Intractable pain C. Exocrine deficiency D. To decrease risk of cancer

(c) B. Indications for surgical treatment of chronic pancreatitis include severe pain that limits the patient's functioning or intractable pain despite the use of non-narcotic analgesics and absence of alcohol intake.

A 52-year-old female presents with diffuse abdominal pain accompanied by distention and visible peristalsis. Ausculation reveals hyperactive bowel sounds. Percussion is tympanic throughout. Palpation reveals mild diffuse tenderness without masses. The most likely diagnosis is A. intra-abdominal abscess. B. intestinal obstruction. C. paralytic ileus. D. cholecystitis.

(c) B. Intestinal obstruction without complications is suggested by crampy pain, abdominal distention, hyperactive bowel sounds, visible peristalsis, and minimal tenderness.

102. Health Maintenance/Cardiology A patient 10 years status-post deep vein thrombosis presents with chronic lower extremity edema. Which of the following prevention strategies is most appropriate for this patient? A. Catheter directed thrombolysis B. Compression stockings C. Warfarin (Coumadin) therapy D. Vein stripping .

(c) B. Irreversible tissue damage changes and associated complications in the lower legs can be minimized through early and aggressive anticoagulation of acute DVT to minimize the valve damage and by prescribing stockings if chronic edema develops in subsequent years.

Health Maintenance/Gastrointestinal/Nutritional An asymptomatic 30-year-old male with no significant past medical history presents to the office. He is concerned about his father being diagnosed with colon cancer at the age of 60. Which of the following is the most appropriate health counseling for this patient? A. Flexible sigmoidoscopy now B. Colonoscopy at age 40 C. Flexible sigmoidoscopy age 35 D. Colonoscopy age 50

(c) B. It is recommended that patients with a single first-degree relative with colon cancer diagnosed at or after age 60 should begin screening at age 40 preferably with a colonoscopy.

A 56 year-old female four days post myocardial infarction presents with a new murmur. On examination the murmur is a grade 3/6 pansystolic murmur radiating to the axilla. She is dyspenic at rest and has rales throughout all her lung fields. Blood pressure is 108/68 mmHg, pulse 70 bpm. Which of the following would be the definitive clinical intervention? A. Intra-aortic balloon counterpulsation B. Mitral valve replacement C. Coronary artery bypass surgery D. Immediate fluid bolus

(c) B. MVR is the definitive intervention to correct MR caused by papillary muscle rupture.

204. History & Physical/Cardiology Which of the following would you expect on physical examination in a patient with mitral valve stenosis? A. Systolic blowing murmur B. Opening snap C. Mid-systolic click D. Paradoxically split S2

(c) B. Mitral stenosis is characterized by a mid-diastolic opening snap.

224. Scientific Concepts/Cardiology The most common arrhythmia encountered in patients with mitral stenosis is A. atrial flutter. B. atrial fibrillation. C. paroxysmal atrial tachycardia. D. atrio-ventricular dissociation.

(c) B. Mitral stenosis leads to enlargement of the left atrium, which is the major predisposing risk factor for the development of atrial fibrillation

A patient presents complaining of vague anal discomfort. On examination, the patient is noted to have a few small external hemorrhoids and edema in the anal region. Which of the following is the most appropriate intervention? A. proctoscopy followed by a hemorrhoidectomy B. increased dietary fiber and sitz baths C. hemorrhoidal banding D. inject a sclerosing agent

(c) B. Most hemorrhoids respond well to conservative treatment such as fiber and sitz baths.

63 year-old female presents with a complaint of chest pressure for one hour, noticed upon awakening. She admits to associated nausea, vomiting, and shortness of breath. 12 lead EKG reveals ST segment elevation in leads II, III, and AVF. Which of the following is the most likely diagnosis? A. Aortic dissection B. Inferior wall myocardial infarction C. Acute pericarditis D. Pulmonary embolus

(c) B. Myocardial infarction often presents with chest pressure and associated nausea and vomiting. ST segment elevation in leads II, III, and AVF are classic findings seen in acute inferior wall myocardial infarction.

A person presenting with bleeding esophageal varicies should be treated with which of the following while awaiting arrival of endoscopy? A. Carafate (Sucralfate) B. Octreotide (Sandostatin) C. Omeprazole (Prilosec) D. Enoxaparin (Lovenox)

(c) B. Octreotide is a vasoacctive drug used in the treatment of GI bleeding as well as somatostatin, vasopressin, and terlipressin. Somatostatin and octreotide are preferred due to safety and less incidence of serious side effects.

65 year-old female presents to the office with a six-month history of back pain. The patient states that she is shrinking and thinks she is about an inch shorter than she was a year ago. Serum parathyroid hormone, calcium, phosphorus, and alkaline phosphatase are all normal. Which of the following would you most likely see on the x-ray of her spine? A. Radiolucent lesions B. Demineralization C. Chondrocalcinosis D. Subperiosteal resorption

(c) B. Osteoporosis presents with varying degrees of back pain and loss of height is common. The serum calcium, parathyroid hormone, phosphorus, and alkaline phosphatase are normal. Xray findings demonstrate demineralization in the spine and pelvis.

A 62 year-old male presents with complaints of vague epigastric abdominal pain associated with jaundice and generalized pruritus. Physical examination reveals jaundice and a palpable non-tender gallbladder, but is otherwise unremarkable. Which of the following is the most likely diagnosis? A. Viral hepatitis B. Pancreatic cancer C. Acute cholecystitis D. Gilbert's syndrome

(c) B. Pancreatic cancer is suggested by the vague epigastric pain with the jaundice resulting from biliary obstruction due to cancer involving the pancreatic head. The presence of a palpable non-tender gallbladder (Courvoisier's sign) also indicates obstruction due to the cancer.

Primary hyperparathyroidism is characterized by which of the following? A. decreased serum magnesium B. decreased serum phosphate C. increased hematocrit D. increased bone density

(c) B. Parathyroid hormone stimulates the osteoclasts to increase bone resorption, leading to elevated calcium levels. It works in the kidney to increase calcium reabsorption and increase renal excretion of phosphorous.

167. 6. CLIN THERAPEUTICS In addition to verapamil (Calan), which of the following medications is considered the treatment of choice for patients with hypertrophic cardiomyopathy? A. ACE inhibitors B. beta-blockers C. antiarrhythmics D. alpha-agonists

(c) B. Patients with hypertrophic cardiomyopathy are treated with both calcium channel blockers and beta-blockers because these agents improve symptoms, especially dyspnea and chest pain. These agents slow the heart rate, which prolongs diastole and allows for increased passive ventricular filling. When the ventricle is allowed to fill, there is less outflow obstruction.

50 year-old male with history of alcohol abuse presents with acute, severe epigastric pain radiating to the back. The patient admits to an episode of coffee ground emesis. On examination he is ill-appearing with a rigid, quiet abdomen and rebound tenderness. Which of the following is the most likely diagnosis? A. Abdominal aortic aneurysm B. Perforated duodenal ulcer C. Acute myocardial infarction D. Cholecystitis

(c) B. Perforation of a duodenal ulcer causes sudden, severe pain, with rebound tenderness and rigid abdomen on physical examination. It is often associated with coffee ground emesis.

Which of the following treatments is the most appropriate for a patient with an acutely tender, fluctuant perirectal mass? A. Fistulectomy B. Drainage of an abscess C. Antibiotic therapy D. Removal of an anal fissure

(c) B. Perirectal abscesses should be treated by drainage as soon as a diagnosis is established.

142. Clinical Therapeutics/Endocrinology Which of the following is the rationale for the utilization of beta blocker therapy in a patient with hyperthyroidism? A. Reduce goiter size B. Alleviate symptoms C. Regulate TSH D. Normalize free T4

(c) B. Propranolol is used for symptomatic relief until the hyperthyroidism is resolved. It relieves tachycardia, tremor, diaphoresis and anxiety.

25. Clinical Intervention/Cardiology A 20 year-old male presents with complaint of brief episodes of rapid heart beat with a sudden onset and offset that have increased in frequency. He admits to associated shortness of breath and lightheadedness. He denies syncope. Electrocardiogram reveals a delta wave prominent in lead II. Which of the following is the most appropriate long-term management in this patient? A. Implantable cardio defibrillator B. Radiofrequency ablation C. Verapamil (Calan) D. Metoprolol (Lopressor)

(c) B. Radiofrequency ablation is the procedure of choice for long-term management in patients with accessory pathways (WPW) and recurrent symptoms.

Scientific Concepts/Cardiology A 36-year-old male presents for follow-up of his hypertension. The patient is currently on three anti-hypertensive medications without improvement of his blood pressure. On examination his BP is 170/86mmHg and his HR is 60bpm and regular. His heart examination reveals a regular rate and rhythm without S3, S4 or murmur and his lungs are clear to auscultation bilaterally. Abdominal examination reveals a bruit over his left upper abdomen. Which of the following is the most likely underlying etiology of this patient's hypertension? A. Pheochromocytoma B. Renal artery stenosis C. Cushing syndrome D. Coarctation of the aorta

(c) B. Renal artery stenosis is characterized by hypertension that is resistant to three or more medications and renal artery bruits on examination.

Urology/Renal: Clinical Intervention, Chronic kidney disease A 73-year-old male with chronic renal failure due to longstanding hypertension and diabetes is being discharged from the hospital today. Which dietary recommendation should be made to prevent further complications and progression of his renal disease? A. Increase salt intake B. Restrict dietary protein C. Add a daily potassium supplement D. Decrease gluten consumption

(c) B. Restricting protein in the diet may slow the progression of chronic kidney disease. When protein is ingested, protein waste products are created. Healthy kidneys have millions of nephrons that filter this waste. It's then removed from the body in the urine. Unhealthy kidneys lose the ability to remove protein waste and it starts to build up in the blood.

Which of the following is the therapy of choice for long-term management of esophageal varices in a patient who cannot tolerate beta blocker therapy? A. Octreotide (Sandostatin) B. Sclerotherapy C. Transjugular intrahepatic portosystemic shunt D. Sengstaken-Blakemore tube

(c) B. Sclerotherapy is effective in decreasing the risk for rebleeding in a patient with esophageal varices.

Which of the following is the therapy of choice for long-term management of esophageal varices in a patient who cannot tolerate beta-blocker therapy? A. Octreotide (Sandostatin) B. Sclerotherapy C. Transjugular intrahepatic portosystemic shunt D. Sengstaken-Blakemore tube D. Sengstaken-Blakemore tube

(c) B. Sclerotherapy is effective in decreasing the risk for rebleeding in a patient with esophageal varices. (u) A. Octreotide is used to control acute variceal bleeding but does not have a role in long-term management. (u) C. Transjugular intrahepatic portosystemic shunt (TIPS) decreases rebleeding more than sclerotherapy or ligation but is associated with a higher incidence of encephalopathy and has shown no decrease in mortality. TIPS is used in patients who fail endoscopic or pharmacologic therapy.

Diagnostic Studies/Gastrointestinal/Nutritional A patient presents with a long history of dyspepsia refractory to maximum appropriate therapy. A recent upper GI series revealed multiple gastric and duodenal ulcerations with prominent mucosal folds. What would diagnostic study be confirmatory of this patient's suspected diagnosis? A. EGD with duodenal biopsy B. Serum gastrin level C. Helicobacter pylori serology D. Abdominal CT scan

(c) B. Serum gastrin levels can be elevated for many reasons; however significant elevations in a fasting state as well as with the secretin stimulation test are confirmatory for Zollinger-Ellison syndrome which is highly suspect in this case

152. Diagnosis/Gastrointestinal/Nutritional A 65-year-old homeless male with a history of pancreatitis is seen in the emergency department for vomiting, upper abdominal pain, back pain, and weakness. He is cachectic, pale and jaundiced. A 4-5 cm mass is palpable in the mid to right hypochondrium. What is the most likely diagnosis? A. Chronic cholecystitis B. Carcinoma of the head of the pancreas C. Fibrolipoma D. Primary biliary cirrhosis

(c) B. Seventy-five percent of pancreatic cancers are in the head. Risk factors include age, tobacco use, obesity, chronic pancreatitis, family history and previous abdominal radiation.

A 65 year-old homeless male with a history of pancreatitis is seen in the emergency department for vomiting, upper abdominal pain, back pain and weakness. He is cachetic, pale and jaundiced. A 4-5 cm mass is palpable in the mid to right hypochondrium. What is the most likely diagnosis? A. Chronic cholecystitis B. Carcinoma of head of pancreas C. Fibrolipoma D. Primary biliary cirrhosis

(c) B. Seventy-five percent of pancreatic cancers are in the head. Risk factors include age, tobacco use, obesity, chronic pancreatitis, family history and previous abdominal radiation.

Which medication is considered the mainstay of therapy for mild to moderate inflammatory bowel disease? A. Prednisone B. Sulfasalazine C. Metronidazole D. Azathioprine (Imuran)

(c) B. Sulfasalazine and other 5-aminosalicylic acid drugs are the cornerstone of therapy in mild to moderate inflammatory bowel disease as they have both anti-inflammatory and antibacterial properties. (u) D. Azathioprine is used in severe, glucocorticoid-dependent inflammatory bowel disease.

214. Clinical Intervention/Gastrointestinal/Nutritional Which of the following complications of Crohn's disease most frequently requires surgery? A. Hemorrhage B. Intra-abdominal abscess C. Malabsorption D. Perforation

(c) B. Surgery is the treatment of choice for an abscess, which is a common complication of Crohn's disease

History & Physical/Dermatology A 72-year-old farmer comes to the office for evaluation of a pearly ulcerated papule on his right nostril. The papule has been bleeding off and on for the past couple weeks. Which of the following findings would be most concerning on the physical examination of this patient? A. Cherry angioma B. Telangiectasia C. Spider angioma D. Pyogenic granuloma

(c) B. Telangiectatic vessels are often visible skin findings with basal cell carcinoma.

149. Clinical Intervention/Dermatology A 37-year-old obese, otherwise healthy female presents with complaints of an enlarging painful, red mass on her left medial thigh. She had tried over-the-counter hydrocortisone cream and triple antibiotic ointment without improvement. Examination reveals a 4 cm, warm, tender, fluctuant mass with surrounding erythema and no inguinal lymphadenopathy. Which of the following is the most appropriate clinical intervention for this patient? A. Warm compresses and topical antibiotic therapy B. Incision and drainage wound culture and oral antibiotic therapy C. Incision and drainage only D. Systemic antibiotic therapy only

(c) B. The patient has a carbuncle with early cellulitis. The mainstay of treatment is incision and drainage with wound culture followed by administration of oral antibiotics, primarily for the early cellulitis. Cephalexin; Dicloxacillin ○ Erythromycin or Clindamycin (if PCN allergy) ○ Vancomycin or Linezolid (if MRSA) ● If due to cat bite (Pasteurella multocida): ○ Augmentin MRSA → cellulitis → Antibiotics ○ IV vancomycin or Linezolid ○ Oral: Bactrim ( Clindamycin, Doxycycline, Daptomycin, Linezolid

Diagnostic Studies/Endocrinology A patient complains of fatigue, tremors, palpitations, and heat intolerance. The thyroid is diffusely enlarged and firm on palpation. Which of the following laboratory findings is the most consistent with this presentation? Answers A. Low T4 B. Low TSH C. Decreased bilirubin D. Normal radionuclide scan planation.

(c) B. The presentation is consistent with hyperthyroidism. Laboratory findings include low TSH, elevated free and total thyroid hormone levels, and an increased uptake on radionuclide scan. There may also be elevated bilirubin, liver enzymes, and ferritin levels, along with anemia and thrombocytopenia.

Diagnostic Studies/Pulmonology A 56-year-old male with a 40 pack-year smoking history presents complaining of progressive shortness of breath. Spirometry reveals an FEV1 of 2 L (40% of predicted), an FVC of 4 L (80% of predicted) and an FEV1/FVC of 50%. These findings are most consistent with A. sarcoidosis. B. chronic bronchitis. C. interstitial lung disease. D. congestive heart failure.

(c) B. The reduced FEV1 and FEV1/FVC is characteristic of an obstructive pattern that is seen in chronic obstructive pulmonary diseases, such as chronic bronchitis.

16. Health Maintenance/Cardiology A 36-year-old female presents for a routine physical. She has no current complaints and her only medication is oral contraceptives. The patient is preparing for a trip to Australia and is worried about the long flight as her mom has a history of deep vein thrombosis after a long trip several years ago. Physical examination reveals BP 110/60 mmHg, HR 66 bpm, regular. The heart is regular rate and rhythm without murmur, lungs are clear to auscultation bilaterally and extremities are without edema. Which of the following is the most appropriate recommendation for your patient? A. Discontinue oral contraceptives B. Recommend walking frequently during the flight C. Begin daily aspirin therapy D. Increase fluid intake 2-3 days prior to the flight

(c) B. The risk of deep vein thrombosis after air travel increases with flight duration. Preventive measures for patients include using support hose and performing in-flight exercises and walking.

88. History & Physical/Gastrointestinal/Nutritional The initial manifestation of acute pancreatitis is often A. generalized pruritus. B. epigastric pain. C. epigastric mass. D. anorexia.

(c) B. The typical findings in acute pancreatitis include nausea, vomiting, and abdominal pain. Epigastric pain, generally abrupt in onset, is steady and severe.

History & Physical/Neurology A 75-year-old male presents to the ER with the following stroke findings: right-sided hemiparesis (face and hand more affected than leg), homonymous hemianopsia of the right half of both visual fields, and aphasia. Where is the location of his stroke? A. Anterior cerebral artery B. Middle cerebral artery C. Posterior cerebral artery D. Internal carotid artery

(c) B. This case is a description of a middle cerebral artery stroke.

28. Diagnosis/Endocrinology A 55-year-old female presents to the clinic with lethargy, fatigue, constipation, and menorrhagia. Physical examination reveals an enlarged thyroid, dry skin, and a heart rate of 50 bpm. Laboratory results show a decrease in free T4 and an elevation in TSH. Which of the following is the most likely diagnosis? A. Secondary hypothyroidism B. Primary hypothyroidism C. Primary hyperthyroidism D. Secondary hyperthyroidism

(c) B. This is a classic presentation of symptoms of primary hypothyroidism. Symptoms include weight gain, fatigue, lethargy, depression, weakness, constipation, menorrhagia; and patients often present with a palpable, enlarged thyroid.

Diagnosis/Endocrinology A 55-year-old female presents to the clinic with lethargy, fatigue, constipation, and menorrhagia. Physical examination reveals an enlarged thyroid, dry skin, and a heart rate of 50 bpm. Laboratory results show a decrease in free T4, and an elevation in TSH. Which of the following is the most likely diagnosis? A. Secondary hypothyroidism B. Primary hypothyroidism C. Primary hyperthyroidism D. Secondary hyperthyroidism

(c) B. This is a classic presentation of symptoms of primary hypothyroidism. Symptoms include weight gain, fatigue, lethargy, depression, weakness, constipation, menorrhagia; and patients often present with a palpable, enlarged thyroid.

A 60-year-old male presents with a slowly developing facial lesion first noticed 4-5 months ago. He describes it as non-painful and non-pruritic but notes it to be extremely scaly. He denies a history of similar lesions or dermatologic disease. Examination reveals a one centimeter, firm nodule at the right temple with heavy keratinization. There is no fluctuance or skin discoloration. Which of the following is the most likely diagnosis? A. Actinic keratosis B. Squamous cell cancer C. Granuloma annulare D. Merkel cell carcinoma

(c) B. This is a very typical scenario for a squamous cell cancer occurring in a sun-exposed area, with slow development and heavy keratinization.

69. Diagnosis/Endocrinology A 32-year-old male presents with a one-month history of weakness, anorexia, and weight loss. On physical examination, he is hypotensive and his skin has diffuse hyperpigmentation. Plasma cortisol levels drawn at 8 am are low. Which of the following is the most likely diagnosis? A. Pituitary insufficiency B. Addison disease C. Cushing syndrome D. Primary aldosteronism

(c) B. This is the classic presentation of Addison's disease (chronic adrenocortical insufficiency).

32. Clinical Therapeutics/Obstetrics/Gynecology What is the initial treatment of choice for hyperthyroidism in a 10-week pregnant patient? A. No treatment is necessary. B. Propylthiouracil (PTU) C. Radioiodine treatment D. Subtotal thyroidectomy

(c) B. This is the initial treatment of choice.

A 45 year-old male presents for a routine physical. His mother and father both had ischemic strokes in their 70's. He does not smoke. His blood pressure is 128/80 mmHg, pulse 78/minutes and regular, respiratory rate of 12/minute. He has no bruits on examination. Which of the following studies should be ordered to further evaluate this patient's risk of stroke? A. Electrocardiogram B. Fasting lipid profile C. Carotid Doppler ultrasound D. MRI with gadolinium Explanations

(c) B. This patient has a genetic risk for stroke secondary to family history. Having an elevated LDL cholesterol and low HDL cholesterol puts him at a greater risk for stroke.

A 65-year-old woman is admitted to the hospital for severe abdominal pain and vomiting over the last 6 hours. Her laboratory findings include: WBC Count - 19,000 Serum triglyceride level - 1100 mg/dL Glucose - 280 mg/dL Which of the following is the most likely diagnosis? A. Acute cholecystitis B. Acute pancreatitis C. Hepatitis D. Diabetic ketoacidosis

(c) B. This patient has acute pancreatitis. Typical symptoms include abdominal pain and nausea and vomiting. Significant hypertriglyceridemia is present. Transient hyperglycemia is common, in addition to elevation of lipase and amylase, serum aminotransferase and alkaline phosphatase are elevated in 50% of patients with acute pancreatitis

A 75 year-old female with history of coronary artery disease and dyslipidemia presents for routine follow-up. Physical examination reveals loss of hair on the lower extremities bilaterally with thinning of the skin. Femoral pulses are +2/4 bilaterally, pedal pulses are diminished bilaterally. Ankle brachial index is reduced. Which of the following signs or symptoms is this patient most likely to have? A. Lower extremity edema B. Calf pain with walking C. Numbness of the lower extremities D. Itching of the lower extremities

(c) B. This patient has signs and symptoms consistent with arterial insufficiency and would most likely complain of intermittent claudication. (u) A. This patient has signs and symptoms consistent with arterial insufficiency. Lower extremity edema is seen in patients with venous insufficiency. (u) C. Numbness of the lower extremities would be seen with acute arterial occlusion. (u) D. Itching of the lower extremities may be seen in chronic venous insufficiency because of secondary skin changes, but is not common in arterial insufficiency.

A patient who was recently diagnosed with viral pericarditis now complains of severe dyspnea and non-productive cough with pain over the precordial region. His vital signs reveal a heart rate 130bpm and respiratory rate 26. Blood pressure is 130/105 mmHg but fluctuates with inspiration resulting in a 20 mmHg decline in the systolic pressure. Which of the following is the most appropriate therapy for this patient? A. Serial echocardiography B. Urgent pericardiocentesis C. Surgery for pericardial window D. Furosemide (Lasix) bolus

(c) B. This patient has signs and symptoms consistent with pericardial tamponade. Urgent pericardiocentesis is required.

185. Clinical Intervention/Cardiology A 45 year-old female presents with complaint of lower extremity discomfort. The patient admits to dull aching of the left lower extremity. The discomfort is worse after standing for long periods of time. Examination reveals dilated, tortuous and elongated veins on the medial aspect of the left leg. Pedal pulses are +2/4 bilaterally. There are no skinchanges or lower extremity edema noted. Which of the following is the most appropriate initial treatment of choice int his patient? A. heparin B. compression stockings C. furosemide (Lasix) D. thrombectomy

(c) B. This patient has signs and symptoms of varicose veins. Initial treatment with compression stockings may prolong or avoid the need for surgery.

A 19 year-old female presents with complaint of palpitations. On examination you note the patient to have particularly long arms and fingers and a pectus excavatum. She has a history of joint dislocation and a recent ophthalmologic examination revealed ectopic lentis. Which of the following echocardiogram findings would be most consistent with this patient's physical features? A. right atrial enlargement B. aortic root dilation C. pulmonic stenosis D. ventricular septal defect

(c) B. This patient has the signs and symptoms consistent with Marfan's syndrome. Ectopia lentis, aortic root dilation and aortic dissection are major criteria for the diagnosis of the disease.

A 76 year-old female presents to the ED with the worst abdominal pain in her life. The pain began following a large meal and is located periumbilically. Although she is writhing in pain, she does not have an exacerbation of the pain on palpation of the abdomen. She has a history of coronary artery disease, asthma, and atrial fibrillation. Which of the following is the most likely diagnosis? A. Toxic megacolon B. Mesenteric thrombosis C. Fulminant hepatitis D. Acute diverticulitis with perforation

(c) B. This patient is at risk for mesenteric ischemia due to advanced age, atherosclerosis and atrial fibrillation. This is the classic presentation for this condition with pain out of proportion to physical examination findings.

Diagnosis/Gastrointestinal/Nutritional A 76-year-old female presents to the ED with the worst abdominal pain in her life. The pain began following a large meal and is located periumbilically. Although she is writhing in pain, she does not have an exacerbation of the pain on palpation of the abdomen. She has a history of coronary artery disease, asthma, and atrial fibrillation. Which of the following is the most likely diagnosis? Answers A. Toxic megacolon B. Mesenteric thrombosis C. Fulminant hepatitis D. Acute diverticulitis with perforation

(c) B. This patient is at risk for mesenteric ischemia due to advanced age, atherosclerosis and atrial fibrillation. This is the classic presentation for this condition with pain out of proportion to physical examination findings.

Diagnostic Studies/Neurology A 74-year-old female is being treated for mild hypertension. She is found at home with right hemiparesis and brought to the emergency department. Her daughter states that the patient fell in her kitchen 2 days ago, but had no complaints at that time. She did state that her mother sounded a little confused this morning. The patient's left pupil is dilated. Which of the following diagnostic studies should be ordered first? A. MRI of the brain B. CT scan of the brain C. Skull x-ray D. Lumbar puncture Explanations

(c) B. This patient presents with a history of minor trauma and progressive neurological abnormalities consistent with a subdural hematoma. Diagnosis would be confirmed by CT scan, which is less expensive and more sensitive for blood than an MRI.

Diagnostic Studies/Neurology A 53-year-old female with sudden onset "of the worst headache she has ever had" presents to the emergency department. She has a history of migraines but states that the current headache is not like her usual headaches. Results of her physical examination are unremarkable. Which of the following is the next best step in the evaluation of this patient? A. Angiogram B. CT scan C. Transcranial Doppler D. MRI

(c) B. This patient's history is highly suggestive of subarachnoid hemorrhage. CT is best to screen for intracranial hemorrhage. It is faster than MRI and more sensitive in the first 24 hours.

34. Diagnosis/Cardiology A 60-year-old female recently discharged after an 8-day hospital stay for pneumonia presents complaining of pain and redness in her right arm. The patient thinks this was the area where her IV was placed. The patient denies fever or chills. Examination of the area reveals localized induration, erythema, and tenderness. There is no edema or streaking noted. Which of the following is the most likely diagnosis? A. Acute thromboembolism B. Thrombophlebitis C. Cellulitis D. Lymphangitis

(c) B. This patient's signs and symptoms are consistent with thrombophlebitis. Short-term venous catheterization of a superficial arm vein is commonly the cause and thrombophlebitis characterized by dull pain, induration, redness and tenderness along the course of the vein.

A 26 year-old patient is brought to the emergency department after a head on collision. The patient complains of chest pain, dyspnea and cough. Examination reveals the patient to be tachypneic and tachycardic with a narrow pulse pressure. Jugular venous distension is noted. Electrocardiogram reveals nonspecific t wave changes and electrical alternans. Which of the following is the most appropriate management plan for this patient? A. Serial echocardiograms B. Pericardiocentesis C. Cardiac catheterization D. Pericardiectomy

(c) B. Urgent pericardiocentesis is the initial treatment of choice in a patient with cardiac tamponade.

Diagnostic Studies/Urology/Renal A 65 year-old male smoker presents with a new onset of hematuria. Which of the following studies would be the most definitive diagnostic study in this patient? A. Ultrasound B. Cystoscopy C. KUB radiograph D. CT of the pelvis

(c) B.Painless Hematuria is the presenting symptom in 85-90% of patients with bladder cancer. Bladder cancer is the second most common urologic cancer and occurs more commonly in men than women. Cigarette smoking is a risk factor in over 60% of patients. The diagnosis and staging of bladder cancer are made by cystoscopy followed by transurethral resection of the bladder tumor.

History & Physical/Urology/Renal Which of the following is the most consistent physical examination finding associated with a hydrocele? A. Tender and swollen testicle B. Palpable painless mass on the testicle C. Nontender scrotal mass that transilluminates D. Inability to get exam finger above scrotal mass

(c) C. A hydrocele is a fluid-filled mass that is nontender to palpation. Diagnosis is readily made by transillumination, however, evaluation is still warranted as 10% of testicular tumors may have an associated hydrocele.

A patient presents with abdominal pain in the right lower quadrant, examination reveals increased pain in the right lower quadrant on deep palpation of the left lower quadrant. This commonly known as which of the following? A. Psoas sign B. Murphy's sign C. Rovsing's sign D. Obturator sign

(c) C. A positive Rovsing's sign can be elicited in a patient with appendicitis when increased pain occurs in the right lower quadrant upon palpation of the left lower quadrant. (u) D. Obturator sign is right lower quadrant pain with internal rotation of the hip.

History & Physical/Gastrointestinal/Nutritional What physical examination finding is most specific for acute cholecystitis? A. Psoas sign B. Rovsing's sign C. Murphy's sign D. Cullen's sign

(c) C. A sharp increase in tenderness with a sudden stop in inspiratory effort constitutes a postive Murphy's sign and is most specific for cholecystitis.

A 29 year-old male is involved in a motor vehicle crash. On the secondary survey it is noted that there is blood at the meatus and the patient is suspected of having a pelvic fracture. The patient is otherwise stable. Which of the following tests should be done to evaluate the urinary system? A. Voiding cystourethrogram (VCUG) B. Intravenous pyelogram (IVP) C. Urethrogram D. Renal arteriography

(c) C. A urethrogram is the only procedure that should be done to evaluate this type of injury as urethral integrity may have been compromised secondary to the pelvic fracture. Blood at the meatus is the most important finding of suspected urethral injury.

A 60-year-old male with a history of hypertension presents for routine physical examination. He has no current complaints. Vital signs are BP of 136/70 mmHg, HR 60 bpm, regular, RR 14, unlabored. Heart shows regular rate and rhythm with no S3, S4 or murmur, Lungs are clear to auscultation bilaterally, and the abdomen is soft, nontender. There is a 5cm palpable pulsatile abdominal mass noted. Which of the following is the best initial diagnostic study in this patient? A. Magnetic resonance imaging (MRI) B. Arteriography C. Ultrasound (US) D. Plain film

(c) C. Abdominal ultrasound is the diagnostic study of choice for the initial diagnosis of an abdominal aortic aneurysm.

4. History & Physical/Dermatology A 46 year-old male is brought to the emergency department after sustaining burns to his entire right arm and leg in a house fire. Which of the following percentages best classifies the extent of his burns? A. 12 B. 18 C. 27 D. 36

(c) C. According to the "rule of nines" the percentages can be calculated as follows: entire arm = 9% of body surface area (BSA). Anterior surface of each leg = 9% BSA, posterior surface of each leg = 9% BSA. The patient has a total of 27% BSA with burns.

Diagnosis/Dermatology A 56 year-old, right hand dominant, carpenter presents to your clinic complaining of a prolonged bruise under his left thumbnail. He states that he first noticed it one year ago. Physical examination reveals a nontender left thumb with a 6 mm macular lesion located under the distal nail bed. It is mixed dark brown and black in color, with irregular borders. The most likely diagnosis is Answers A. lentigo. B. trauma. C. melanoma. D. nevus.

(c) C. Acral lentiginous melanoma may occur on the palm, sole, nail bed, or mucus membrane. This lesion is suspicious for a melanoma due to its irregular borders, being variegated in color, and its size. An excisional biopsy is required and will ensure the diagnosis.

A 56 year-old male presents to the office with a history of abdominal aortic aneurysm. He was told that he will need on going evaluation to assess whether the aneurysm is expanding. What is the recommended study to utilize in this situation? A. plain film of the abdomen B. serial abdominal exam C. ultrasound of the abdomen D. angiography of the abdominal aorta

(c) C. An abdominal ultrasound can delineate the transverse and longitudinal dimensions of an abdominal aortic aneurysm and may detect mural thrombus. Abdominal ultrasound is best used to screen patients at risk for the development of this condition.

133. Clinical Intervention/Cardiology A 48 year-old male with a history of coronary artery disease and two myocardial infarctions complains of shortness of breath at rest and 2-pillow orthopnea. His oxygen saturation is 85% on room air. The patient denies any prior history of symptoms. The patient denies smoking. Results of a beta-natriuretic peptide (BNP) are elevated. What should be your next course of action for this patient? Answers A. Send him home on 20 mg furosemide (Lasix) p.o. every day and recheck in one week B. Send him home on clarithromycin (Biaxin) 500 mg p.o. BID and recheck in 1 week C. Admit to the hospital for work up of left ventricular dysfunction D. Admit to the hospital for work up of pneumonia

(c) C. An elevated BNP is seen in a situation where there is increased pressure in the ventricle during diastole. This is representative of the left ventricle being stretched excessively when apatient has CHF. Sending a patient home would be inappropriate in this case.

pts with a mole that has changed appearance over the past few months. She says it has enlarged. Also noted is an asymmetric lesion with an irregular border and variation in color from light brown to dark blue/black. Which of the following is the most appropriate tx? A. curettage B. shave biopsy C. excisional biopsy D. aspiration for cytology

(c) C. An excisional biopsy is indicated for suspected cases of melanoma.

History & Physical/Gastrointestinal/Nutritional A 3 week-old male infant presents with recurrent regurgitation after feeding that has progressed to projectile vomiting in the last few days. The mother states that the child appears hungry all of the time. She denies any diarrhea in the child. Which of the following clinical findings is most likely? A. Bile-stained vomitus B. Hemoccult positive stools C. Olive-sized mass in the right upper abdomen D. Sausage-shaped mass in the upper-mid abdomen

(c) C. An olive-sized mass may be palpated in the right upper abdomen in pyloric stenosis and if found, is pathognomonic for pyloric stenosis

CLIN THERAPEUTICS Long-term treatment of Crohn's disease includes A. allopurinol (Zyloprim). B. omeprazole (Prilosec). C. azathioprine (Imuran). D. 5-Fluorouracil (5-FU).

(c) C. Azathioprine is effective in the long-term treatment of Crohn's disease.

A 60 year-old male has unstable angina, but is otherwise healthy. A 90% lesion is found in the left main coronary artery. Which of the following interventions is most appropriate? A. Thrombolysis with t-PA B. Medical management with nitrates C. Coronary artery bypass graft (CABG) D. Percutaneous transluminal coronary angioplasty

(c) C. CABG is indicated in patients with stenosis of the left main coronary artery and those with three-vessel CAD

A 22 year-old male received a stab wound in the chest an hour ago. The diagnosis of pericardial tamponade is strongly supported by the presence of A. pulmonary edema. B. wide pulse pressure. C. distended neck veins. D. an early diastolic murmur.

(c) C. Cardiac compression will manifest with distended neck veins and cold clammy skin.

114. Clinical Intervention/Cardiology Which of the following can optimize quality of life and is an definitive treatment for a patient with refractory heart failure? A. Ventricular assist device B. Intra-aortic balloon counterpulsation C. Cardiac transplantation D. Partial resection of the left ventricle

(c) C. Cardiac transplantation is effective, with survival rates of 80-90% in 1 year, 60-70% over 5 years. It does improve quality of life despite the immunosuppression medications.

Which of the following can optimize quality of life and is a definitive treatment for a patient with refractory heart failure? A. Ventricular assist device B. Intra-aortic balloon counterpulsation C. Cardiac transplantation D. Partial resection of the left ventricle

(c) C. Cardiac transplantation is effective, with survival rates of 80-90% in 1 year, 60-70% over 5 years. It does improve quality of life despite the immunosuppression medications.

A 35 year-old female who was a back seat passenger in a vehicle which was involved in a head-on collision is brought to the ED. She is able to tell you that she is having difficulty moving both of her legs and is experiencing bilateral leg pain as well. She is embarrassed because she has "wet myself." Physical examination reveals markedly diminished sensory and motor function of both legs and decreased rectal sphincter tone. Which of the following is the most likely diagnosis? A. Herniated disc at L5-S1 and L4-L5 B. An anterior cord lesion C. Cauda equina syndrome D. An L2 lesion

(c) C. Cauda equina syndrome is a massive central disc protrusion that causes variable degrees of paralysis. Bowel and bladder function may be impaired with saddle anesthesia. This condition is a surgical emergency.

6-year-old child with sickle cell anemia presents with fever and pain over the right tibia. There is tenderness along the anterior tibia, but no pain with motion of the knee or ankle. Which of the following is the most appropriate initial treatment? A. aspirin B. codeine C. cefotaxime D. norfloxacin

(c) C. Cefotaxime provides coverage for osteomyelitis caused by staphylococcus or salmonella.

Health Maintenance/Gastrointestinal/Nutritional Which of the following is associated with an increased risk of developing esophageal cancer? Answers A. Helicobacter pylori B. Familial polyposis C. Chronic gastric reflux D. Hepatitis C

(c) C. Chronic gastric reflux is associated with an increased risk of esophageal cancer.

Which of the following is a proven risk factor for the development of abdominal aortic aneurysm? A. Infective endocarditis B. Diabetes mellitus C. Cigarette smoking D. Alcohol abuse

(c) C. Cigarette smoking is the primary risk factor for the development of aortic aneurysms.

A 12-year-old boy presents to the office with pain in his legs with activity gradually becoming worse over the past month. He is unable to ride a bicycle with his friends due to the pain in his legs. Examination of the heart reveals an ejection click and accentuation of the second heart sound. Femoral pulses are weak and delayed compared to the brachial pulses. Blood pressure obtained in both arms is elevated. Chest x-ray reveals rib notching. Which of the following is the most likely diagnosis? A. abdominal aortic aneurysm B. pheochromocytoma C. coarctation of the aorta D. thoracic outlet syndrome

(c) C. Coarctation is a discrete or long segment of narrowing adjacent to the left subclavian artery. As a result of the coarctation, systemic collaterals develop. X-ray findings occur from the dilated and pulsatile intercostal arteries and the "3"is due to the coarctation site with proximal and distal dilations.

HEALTH MAINT Fecal occult blood testing and sigmoidoscopy would be indicated for which of the following patients? A. 40 year old with a parental history of irritable bowel syndrome B. 40 year old with a parental history of Crohn's disease C. 40 year old with a parental history of colon cancer D. 40 year old with a parental history of diverticular disease

(c) C. Current recommendations for colorectal cancer screening in individuals with a family history of a first-degree member include initial screening at age 40 with fecal occult blood testing and sigmoidoscopy.

A 22 year-old patient complains of sudden onset of chest pain accompanied by shortness of breath. The patient appears dyspneic. On examination, the trachea is deviated to the left, breath sounds are faint on the right, and the right chest is hyperresonant to percussion. The preferable treatment for this patient would be A. a tracheostomy. B. insertion of a chest tube with underwater seal, left 2nd intercostal space. C. needle thoracotomy right 2nd intercostal space D. a lung scan for pulmonary embolus and begin heparin sodium (Heparin) therapy.

(c) C. Decreased breath sounds and hyperresonance are noted on the side of the pneumothorax; tracheal deviation to the opposite side indicates development of a tension pneumothorax. Treatment consists of inserting a chest tube on the side of the pneumothorax and connecting to an underwater seal. (

A pregnant female presents at 32 weeks gestation with painless rectal bleeding and a bulging perianal mass when straining which goes away. Which of the following is the most appropriate management of this patient? A. Hemorrhoidectomy B. Metronidazole (Flagyl) C. Psyllium (Metamucil) D. Sclerotherapy

(c) C. Dietary fiber or psyllium bulk laxatives can be used to decrease straining with defecation. Banding and injection of sclerosing agents are used if mild prolapse, enlargement, or intermittent bleeding is present. A. Hemorrhoidectomy and sclerotherapy are reserved for severe Grade III and IV hemorrhoids.

A patient with a history of severe peptic ulcer disease is 5 weeks status post Billroth I surgery. One week ago he restarted his normal diet and has had the onset of severe nausea, abdominal cramping, and light-headedness that occur approximately thirty minutes after eating. The abdominal exam reveals a healing surgical scar without areas of unusual tenderness or any palpable masses. Which of the following is the most likely diagnosis? A. Anxiety disorder B. Celiac sprue C. Dumping syndrome D. Irritable bowel syndrome

(c) C. Dumping syndrome typically occurs after Billroth type I surgeries as well as gastric bypass surgeries when the patient attempts to eat a large amount of simple sugars.

Diagnostic Studies/Gastrointestinal/Nutritional Which of the following diagnostic studies is indicated in the evaluation of an upper gastrointestinal bleed? A. Esophageal manometry B. Bleeding scan C. Upper endoscopy D. Barium swallow

(c) C. Endoscopy is the evaluation modality of choice in patients with upper GI bleeding. The advantage of this technique is that it can be used for both diagnostic and therapeutic purposes.

Which of the following is the most consistent physical examination finding in a patient with duodenal ulcer? A. Flank tenderness B. Right upper quadrant tenderness C. Epigastric tenderness D. Rebound tenderness

(c) C. Epigastric tenderness is a key feature of duodenal ulcer.

Diagnosis/Psychiatry/Behavioral Medicine Formications are most commonly associated with which of the following? A. Delusional disorder B. Adverse drug reaction C. Alcohol withdrawal D. Obsessive compulsive disorder

(c) C. Formications are the sensation of insects crawling on the skin and is commonly associated with delirium tremens from alcohol withdrawal and cocaine addiction.

A 29 year-old female G4P2Ab1 at 20 weeks gestation complains of nausea and vomiting with tenderness in the RUQ. Vital signs reveal the patient to be febrile. On physical examination, the abdominal examination reveals positive bowel sounds in all quadrants with a positive Murphy's sign. Fundus can be palpated at the level of the umbilicus. The skin is warm and dry with slight tenting. Oral mucosa is dry as well. What is the most likely diagnosis? A. Peptic ulcer disease B. Hyperemesis gravidarum C. Cholecystitis D. Viral gastroenteritis

(c) C. Gallbladder disease represents one of the most common medical and surgical conditions seen during pregnancy. This is thought to be due to a decrease in gallbladder contractility and lithogenicity of the bile. There is an increased risk in multiparous women.

A 32-year-old male with a history of tobacco abuse presents with an intermittent burning sensation in his chest for six months, worsening over the past 2 weeks. His wife has noticed episodes of coughing at night. He denies dysphagia, weight loss, hematemesis, or melena. His vital signs are all normal and physical examination is unremarkable. Which of the following is the most likely diagnosis? A. Peptic ulcer disease B. Acute gastritis C. Gastroesophageal reflux disease D. Esophageal Stricture

(c) C. Gastroesophageal reflux disease presents with at least weekly episodes of heartburn and typically occurs after meals and upon reclining. Patients may complain of regurgitation, chronic cough, laryngitis, or a sore throat.

Diagnostic Studies/Urology/Renal A 38 year-old female presents with right flank pain for several days, shaking chills, fever to 102°F, and general malaise. The flank pain has been intermittently severe, and she has a history of kidney stones. Urinalysis reveals 3+ red blood cells, 3+ leukocyte esterase, trace protein and negative glucose. Which of the following findings would most likely be seen on a renal ultrasound? Answers A. Small echogenic kidneys B. Cysts C. Hydronephrosis D. Capsular hemorrhage Explanations

(c) C. Hydronephrosis, dilation of the collecting ducts, may be present due to a stone or other source of obstruction.

A 67-year-old female with a history of hypertension, diabetes mellitus, and smoking presents to the emergency department with mild expressive aphasia, right facial weakness, and mild right arm weakness. She had awakened 60 minutes ago and was speaking to her husband when her speech suddenly became difficult to understand and weakness was noted. Physical examination reveals a blood pressure of 165/85 mm Hg. A CT of the head shows no intracranial hemorrhage. Which of the following is the most appropriate intervention? A. Aspirin B. Warfarin (Coumadin) C. Tissue plasminogen activator (rt-PA) D. Clopidogrel (Plavix)

(c) C. IV thrombolytic therapy with recombinant tissue plasminogen activator is effective in reducing the neurological deficit in selected patient without CT evidence of intracranial hemorrhage and when administered within 3 hours after onset of ischemic stroke.

Health Maintenance/Gastrointestinal/Nutritional An asymptomatic 50-year-old person has no risk factors for colorectal cancer. In addition to yearly hemoccult screening of the stool, how often should flexible sigmoidoscopy be performed? A. Every year B. Every other year C. Every 5 years D. Every 10 years

(c) C. In a person with no risk factors for colorectal cancer, screening should include annual Hemoccult testing and flexible sigmoidoscopy every 5 years

On physical examination there is evidence of head trauma. The left side of the chest wall appears to move inward with inspiration and outward with expiration. A chest x-ray reveals multiple rib fractures on the left. Which of the following is the most appropriate intervention? A. Surgical fixation of the fractured ribs B. Application of elastic binders and adhesive tape C. Endotracheal intubation and mechanical ventilation D. Chest physiotherapy that encourages frequent coughing

(c) C. Indications for early endotracheal intubation and mechanical ventilation in treatment of flail chest include patients that are over the age of 65, have comorbid lung disease and associated severe head trauma. Other indications include shock, three or more associated injuries and fracture of eight or more ribs.

Clinical Intervention/Gastrointestinal/Nutritional A 34-year-old female 3 days status-post appendectomy complains of mild diffuse abdominal pain with associated nausea and vomiting. On examination, you note diminished bowel sounds. There is generalized abdominal distension with mild tenderness on palpation. There are no signs of peritoneal inflammation. Abdominal radiographs show distended gas-filled loops of the small and large intestine. Abdominal CT scan shows no mechanical obstruction. Which of the following is the most appropriate management for this patient? A. Docusate sodium (Colace) B. Colonoscopic decompression C. Restrict oral intake D. Exploratory laparoscopy

(c) C. Most adynamic ileuses respond to restriction of oral intake and gradual advancement of diet as bowel function returns.

38. Clinical Intervention/Gastrointestinal/Nutritional A 34-year-old female 3 days status-post appendectomy complains of mild diffuse abdominal pain with associated nausea and vomiting. On examination, you note diminished bowel sounds. There is generalized abdominal distension with mild tenderness on palpation. There are no signs of peritoneal inflammation. Abdominal radiographs show distended gas-filled loops of the small and large intestine. Abdominal CT scan shows no mechanical obstruction. Which of the following is the most appropriate management for this patient? A. Docusate sodium (Colace) B. Colonoscopic decompression C. Restrict oral intake D. Exploratory laparoscopy

(c) C. Most adynamic ileuses respond to the restriction of oral intake and gradual advancement of diet as bowel function returns Fasting from food and drink(bowel rest) Nasogastric intubation descompression IV fluids

A patient with a bowel perforation secondary to a gunshot wound is being prepped for surgery. Appropriate antibiotic prophylaxis and treatment includes which of the following? A. cefoxitin and gentamicin B. vancomycin and penicillin G C. nafcillin, gentamicin, and metronidazole D. metronidazole, clindamycin, and cefoxitin

(c) C. Nafcillin provides treatment for penicillinase-resistant organisms Gentamicin covers many gram negative aerobes, and metronidazole is effective against a wide variety of anaerobic bacteria.

189. Clinical Intervention/Hematology A patient receiving heparin therapy for 6 days for deep vein thrombosis develops thrombosis at the IV site. The INR is 1.1. The aPTT is 66 seconds and the platelet count is 47,000 down from 148,000 on admission. Which of the following is the most appropriate treatment? A. Protamine sulfate administration B. Platelet administration C. Discontinue the heparin D. Schedule the patient for plasmapheresis planation.

(c) C. Once the diagnosis of heparin induced thrombocytopenia is suspected treatment should include immediate discontinuation of all forms of heparin and treatment with a direct thrombin inhibitor should begin. Platelet administration is generally not indicated. Protamine sulfate is used for heparin overdosage and plasmapheresis would not be indicated as the platelet count will resume when the heparin is discontinued.

A 42 year-old patient who is being treated for colon cancer with chemotherapy develops nausea and vomiting. Which of the following drugs would be the most effective in controlling the nausea and vomiting? A. scopolamine (Scopace) B. meclizine (Antivert) C. ondansetron (Zofran) D. loperamide (Imodium)

(c) C. Ondansetron selectively blocks 5-HT3 receptors in the periphery (visceral afferent fibers) and in the brain (chemoreceptor trigger zone). It is indicated for use in chemotherapy induced nausea and vomiting.

A 22 year-old male presents several hours after sustaining a hand injury when he punched a wall. X-rays of the hand demonstrate fracture of the fifth metacarpal neck with 65 degrees dorsal angulation and a claw hand. What is the most appropriate intervention? A. Antibiotic treatment and ulnar gutter splint immobilization B. Closed reduction and ulnar gutter splint immobilization C. Open reduction and ulnar gutter splint immobilization D. Ulnar gutter splint immobilization only

(c) C. Open reduction is indicated with angulation of greater than 40 degrees.

215. Clinical Therapeutics/Cardiology Which of the following is the optimal therapy for a 76-year-old patient with no allergies who has chronic atrial fibrillation? A. Aspirin B. Clopidogrel (Plavix) C. Warfarin (Coumadin) D. Low molecular weight heparin exists.

(c) C. Patients older than age 75 who have chronic atrial fibrillation should be anticoagulated with warfarin to maintain an INR between 2.5 and 3.0 for optimum therapy unless a contraindication to therapy

98. History & Physical/Endocrinology Which of the following findings is usually associated with Addison's disease? A. Weight gain B. Hypertension C. Increased pigmentation D. High plasma cortisol levels

(c) C. Patients with Addison's disease have diffuse tanning over nonexposed and exposed skin due to increased melanocytic factor that is released with the adrenocorticotropic hormone.

39. History & Physical/Cardiology A patient with a history of chronic venous insufficiency presents for routine follow-up. Which of the following findings is most likely on physical examination? A. Cold lower extremities B. Diminished pulses C. Lower extremity edema D. Palpable cord

(c) C. Patients with chronic venous insufficiency will commonly have lower extremity edema.

A 22 year-old female complains of worsening pain, swelling, and tenderness in her left heel for 1 week. She sustained a penetrating injury to the heel two weeks ago when she stepped on a nail while running in tennis shoes. Examination reveals a draining puncture wound with surrounding erythema and exquisite tenderness. X-ray of the left foot demonstrates periosteal reaction associated with the wound. Which organism is classically responsible for this infection? A. Escherichia coli B. Streptococcus pyogenes C. Pseudomonas aeruginosa D. Staphylococcus aureus

(c) C. Pseudomonas aeruginosa is frequently associated with osteomyelitis involving puncture wounds of the foot. This is believed to result from direct inoculation with P. aeruginosa via the foam padding found in tennis shoes.

A 16 day-old male presents in the office with a history of vomiting after feeding for the past 2 days. The vomiting has become progressively worse and the mother describes it as very forceful, sometimes hitting the floor 6 feet away. She says the neonate is always hungry. On physical examination, it is noted that he is 2 ounces below birth weight, and has a small palpable mass (about 1.5 cm) in the epigastrium. The most likely diagnosis is A. achalasia. B. tracheoesophageal fistula. C. pyloric stenosis. D. Meckel's diverticulum.

(c) C. Pyloric stenosis begins between 2 to 4 weeks of age with vomiting that becomes projectile after each feeding. An olive-sized mass can often be felt in the epigastrium.

Which of the following is a cause of prerenal azotemia? A. Infection B. Renal toxins C. Poor renal perfusion D. Urinary tract obstruction

(c) C. Renal hypoperfusion is the cause of prerenal azotemia, which may be rapidly reversible when renal blood flow and glomerular ultrafiltration pressure are restored.

Diagnostic Studies/Psychiatry/Behavioral Medicine A 17-year-old girl presents with complaints of being fat and tired, and that her periods are irregular. She says she has been dieting by limiting caloric intake, and typically runs six miles a day, but lately, even five miles makes her very tired. On exam, height is 5'5", weight 92 lbs, temperature 96 degrees F, pulse 48, respiratory rate 18, and BP 88/58 mm Hg. Skin is dry and scaly. The remainder of the exam is unremarkable. Which of the following diagnostic studies would be consistent with your diagnosis? A. Elevated serum albumin B. Short QT interval C. Hypokalemia D. Thrombocytosis Explanations

(c) C. Routine laboratory studies in anorexia nervosa show decreased red blood cells, white blood cells, platelets, potassium, magnesium, thiamine levels, and serum albumin. BUN, AST, ALT, LDH, and beta-carotene levels may be elevated.

Diagnostic Studies/Endocrinology Which of the following laboratory values would be noted in a patient with primary hyperthyroidism? A. Increased TSH, increased free T4 B. Increased TSH, decreased free T4 C. Decreased TSH, increased free T4 D. Decreased TSH, decreased free T4

(c) C. Serum TSH is depressed and serum T3, T4 and uptake are usually all increased in patients with primary hyperthyroidism.

88. Clinical Therapeutics A patient who has chronic Crohn's disease now complains of a nondeforming asymmetric oligoarthritis of his large joints. The patient also notes that he has been having a flare of his Crohn's disease. Which of the following is the best treatment to alleviate both the inflammatory bowel disease and arthritic symptoms? A. Indomethacin B. Methotrexate C. Infliximab (Remicade) D. Cyclophosamide (Cytoxan)

(c) C. Spondyloarthritides respond to both infliximab and etanercept, where infliximab has greater efficacy in Crohn's disease.

2. DIAG STUDIES A 26-year-old male patient presents with complaints of diarrhea with marked flatulence and weight loss for the past 6-8 months. In addition, he states that his stools are loose and soft with an oily appearance and foul smelling. The patient has not traveled out of the country. Which of the following laboratory tests would be most helpful based upon this history? A. stool for leukocytes B. stool for ova and parasites C. stool for 72-hour fecal fat D. stool for culture and sensitivity

(c) C. Steatorrhea is usually present, but may be absent in mild disease of celiac sprue. A quantitative 72-hour stool collection taken while patients are consuming a 100 gm fat diet is a more sensitive means of detecting fat malabsorption.

Gastrointestinal/Nutritional : Clinical Intervention, Ulcerative colitis Which of the following medications is most useful in maintaining remission in a patient with ulcerative colitis? Answers A. Oral corticosteroids B. Corticosteroid enemas C. Sulfasalazine D. Macrodantin

(c) C. Sulfasalazine, olsalazine, and mesalamine are effective in maintaining remission in patients with ulcerative colitis. Gastrointestinal/Nutritional : ● Aminosalicylates (sulfasalazine, mesalamine) → corticosteroids (acute flares only) → immune modifying agents ● Surgery is curative 1st topical + oral mesalamine plus oral corticosteroids 2nd admission + intravenous corticosteroids adjunct cyclosporine or infliximab 3rd colectomy

Clinical Intervention/Dermatology A 36-year-old female comes to the office because a mole on her left calf has changed. On physical examination of the left posterior lower leg, there is a 12 mm, asymmetrical, variegated blue-black macule with a raised pink plaque in the upper half of the lesion. Which of the following is the most appropriate clinical management of this lesion? A. Cryosurgery B. Topical retinoids C. Excisional surgery D. Topical chemotherapy

(c) C. Surgical excision of suspected melanoma is necessary for histologic diagnosis and treatment of the lesion.

42. Clinical Intervention/Cardiology You are evaluating a patient who was brought in secondary to an acute onset of repeated syncopal episodes. His electrocardiogram (ECG) shows wide QRS complexes with a fixed R-R interval at a rate of 40 bpm. The P waves occur with a fixed P-P interval at a rate of 70 bpm. The PR interval is variable. Which of the following is the most appropriate initial treatment for this condition? A. Balloon angioplasty B. Endarterectomy C. Transthoracic pacemaker D. Unsynchronized cardioversion

(c) C. The ECG this patient's symptoms are consistent with a third degree heart block. Patients with episodic or chronic infranodal complete heart block require permanent pacing, and temporary pacing is indicated if implantation of a permanent pacemaker is delayed.

Adults and intravenous drug abusers, which of the following bones is most commonly affected with acute osteomyelitis? A. Femur B. Humerus C. Vertebral spine D. Tibia

(c) C. The bones of the vertebral spine are most commonly affected in a patient with osteomyelitis. Organisms reach the well-perfused vertebral body of adults via spinal arteries and quickly spread from the end plate into the disk space and then to the adjacent vertebral body. The infection may originate in the urinary tract and intravenous drug use carries an increased risk of spinal infection (

36. 2. DIAG STUDIES A 24-year-old male is initially found to have a single nodule in the right lobe of his thyroid gland. He is clinically and chemically euthyroid. The next step is to A. reassure the patient and reassess yearly. B. recheck in 1-3 months. C. do a fine needle aspiration. D. obtain a CT scan of the neck.

(c) C. The combination of fine needle aspiration and radioisotope scanning of a solitary thyroid nodule provides the best diagnostic yield. Because cold nodules may be cancerous, they are generally referred for surgical removal. It is not reasonable to delay the diagnosis ● Cold nodules (no/low iodine uptake) are highly suspicious for malignancy ● Thyroid ultrasound- often used to help obtain a specimen during Fine-needle aspiration (FNA) with biopsy, used to see if the nodule is cystic or solid, to monitor suspicious nodule or to see if growing or shrinking ● Surgery- if thyroid cancer is suspected or if an indeterminate FNA with a cold thyroid scan ○ Total thyroidectomy vs. subtotal thyroidectomy

A 24-year-old male is initially found to have a single nodule in the right lobe of his thyroid gland. He is clinically and chemically euthyroid. The next step is to A. reassure the patient and reassess yearly. B. recheck in 1-3 months. C. do a fine needle aspiration. D. obtain a CT scan of the neck.

(c) C. The combination of fine needle aspiration and radioisotope scanning of a solitary thyroid nodule provides the best diagnostic yield. Because cold nodules may be cancerous, they are generally referred for surgical removal. It is not reasonable to delay the diagnosis. Ultrasound is the best initial step for evaluation of a palpable thyroid nodule. Microcalcifications, hypoechogenicity, a solid nodule, irregular nodule margins, chaotic intranodular vasculature, and a nodule that is more tall than wide are poor prognostic indicators. The combination of fine needle aspiration and radioisotope scanning of a solitary thyroid nodule provides the best diagnostic yield. Because cold nodules may be cancerous, they are generally referred for surgical removal.

79. Diagnosis/Endocrinology A 72-year-old female is being evaluated for recurrent kidney stones. Physical examination reveals no abnormal findings. Laboratory findings show elevated calcium and decreased phosphate levels. The most likely diagnosis? Answers A. Pheochromocytoma B. Adrenal insufficiency C. Hyperparathyroidism D. Breast cancer

(c) C. The majority of patients with hyperparathyroidism are asymptomatic. Recurrent nephrolithiasis may be one of the presentations of primary hyperparathyroidism. Measurement of parathyroid levels would be the initial laboratory test for the evaluation of hypercalcemia

224. Diagnosis/Endocrinology A 72-year-old female is being evaluated for recurrent kidney stones. Physical examination reveals no abnormal findings. Laboratory findings show elevated calcium and decreased phosphate levels. The most likely diagnosis? A. pheochromocytoma B. adrenal insufficiency C. hyperparathyroidism D. vitamin D deficiency

(c) C. The majority of patients with hyperparathyroidism are asymptomatic. Recurrent nephrolithiasis may be one of the presentations of primary hyperparathyroidism. Measurement of parathyroid levels would be the initial laboratory test for the evaluation of hypercalcemia.

A patient diagnosed with Barrett's esophagus is at an increased risk for the development of what type of cancer? A. Squamous cell B. Transitional cell C. Adenocarcinoma D. Atypical carcinoid

(c) C. The most serious complication of Barrett's esophagus is esophageal adenocarcinoma.

History & Physical/Neurology A 24 year-old pregnant female presents with an abrupt onset of facial weakness and mild disturbance of taste this morning upon awaking. She denies drug use and other neurological complaints. Examination findings include a loss of forehead markings on right. Which of the following additional exam findings would likely be present? A. Agitation and word searching B. Contralateral unequal and dilated pupil C. Ipsilateral restriction of eye closure D. Deviated tongue upon protrusion

(c) C. The patient has signs and symptoms of Bell's palsy. With Bell's palsy, it is common to have cranial nerve VII paralysis which will not allow the patient to completely blink the eye on the affected side.

Which of the following is the single most important prognostic factor in regard to melanoma? A. Histopathology of the lesion B. Diameter of the lesion C. Thickness of the lesion D. Ulceration of the lesion

(c) C. The single most significant prognostic factor in melanoma is tumor thickness.

A 60 year-old female injured her right wrist when she slipped and fell onto her outstretched hand. Radiographs show a fracture through the metaphysis of the distal radius with dorsal displacement and angulation. Which of the following splints is the best method of temporary immobilization? A. Dorsal forearm B. Ulnar gutter C. Volar forearm D. Volar with thumb spica

(c) C. The volar forearm splint is best for temporary immobilization of forearm, wrist and hand fractures and is the splint of choice for Colles' fracture.

87. Diagnosis/Gastrointestinal/Nutritional A patient is hospitalized with a change in mental status. Examination reveals that he is unable to maintain dorsiflexion of the wrists after pronating his arms in front of his body.Which of the following is the most likely diagnosis?Answers A. Cocaine overdose B. Hyperthyroidism C. Hepatic encephalopathy D. Parkinson's Disease

(c) C. This is the description for asterixis that is seen with hepatic encephalopathy, uremia, and carbon dioxide narcosis

134. Clinical Intervention/Cardiology An 80 year-old female presents with syncope and recent fatigue and lightheadedness over the past month. She denies chest pain or dyspnea. Physical examination reveals BP 130/70 mmHg, HR 40 bpm, regular, and RR 16.Electrocardiogram reveals two p waves before each QRS complex. Which of the following is the treatment of choice for this patient? A. Cardio defibrillator insertion B. Atropine as needed C. Permanent dual chamber pacemaker insertion D. Ritalin therapy daily

(c) C. This patient has findings consistent with symptomatic second degree AV block Mobitz type II for which permanent pacing is the treatment of choice.

A 70 year-old male with history of ischemic cardiomyopathy presents with a syncopal episode. He denies complaints of chest pain, palpitations, or dyspnea. ECG shows no acute ST-T wave changes. An echocardiogram reveals an ejection fraction of 25% with no valvular abnormalities. Which of the following is the most appropriate management for this patient? A. Dual chamber permanent pacemaker B. Diltiazem (Cardizem) C. Implantable cardio defibrillator D. Midodrine (ProAmatine)

(c) C. This patient has ischemic cardiomyopathy and syncope, which is most likely due to ventricular tachycardia. Insertion of a cardio defibrillator is the management of choice for this patient.

Clinical Intervention/Gastrointestinal/Nutritional A 50-year-old female presents with constipation following an episode of tearing pain associated with bleeding while defecating 4 days ago. She admits to a residual throbbing pain and she is afraid to have a bowel movement. On examination, you note what appears to be a crack in the epithelium of the anal verge. Which of the following is the most appropriate initial management for this patient? A. Anal sphincter strengthening exercises B. Injection sclerotherapy C. Fiber supplementation and sitz baths D. Analgesics with incision and drainage nation.

(c) C. This patient has signs and symptoms consistent with an anal fissure. Most patients will respond to fiber supplementation and sitz baths.

Diagnosis/Pulmonology A 45-year-old male presents with complaints of a chronic cough productive of mucopurulent sputum. cough has been present for the past 3 years, but he has attributed it to a "smoker's cough". He has been coughing up a lot of sputum lasting all winter long for the past 2 years. He denies any hemoptysis, weight loss or chest pain. Physical examination reveals a moderately obese male in no acute respiratory distress. Lung fields reveal the presence of scattered rhonchi and wheezes. There is 1+ peripheral edema. Which of the following is the most likely diagnosis? A. Lung cancer B. Bronchiectasis C. Chronic bronchitis D. Interstitial lung disease

(c) C. This patient most likely has chronic bronchitis which is defined as sputum production and cough for at least 3 months of the year for 2 consecutive years which is primarily caused by cigarette smoking.

Patient with adrenal insufficiency is taking hydrocortisone 25 mg daily. What should the patient do with the hydrocortisone dose when they develop a minor illness such as a cold? A. Stop the hydrocortisone until the illness resolves. B. Continue the current dose that the patient is taking. C. Increase the dose to 50 mg daily until the illness resolves. D. Increase the dose to 250 mg daily until the illness resolves

(c) C. To better mimic the normal physiologic response the baseline dose should be doubled for the duration of the illness. Doses should be increased 5-10 fold with major events such as surgery.

A 72 year-old male with a new diagnosis of congestive heart failure and atrial fibrillation, develops episodes of hemodynamic compromise secondary to increased ventricular rate. A decision to perform elective cardioversion is made and the patient is anticoagulated with heparin. Which test should be ordered to assess for atrial or ventricular mural thrombi? A. Electrocardiogram B. Chest x-ray C. Transesophageal Echocardiogram D. C-reactive protein

(c) C. Transesophageal echocardiography allows for determination of mural thrombi that may have resulted from atrial fibrillation.

A middle-aged patient is being treated for recurrent diarrhea and peptic ulcer disease that is refractory to adequate standard therapy. Which of the following is the most likely diagnosis? A. Achlorhydria B. Drug resistant H. pylori infection C. Zollinger-Ellison syndrome D. Giardiasis

(c) C. Zollinger-Ellison syndrome is the result of unregulated release of gastrin resulting in gastric acid hypersecretion. Up to 50% of patients complain of diarrhea along with peptic ulcer

A 55-year-old male presents complaining of episodic substernal chest pain that occurs especially during strenuous exercise. Suspecting coronary artery disease (CAD), an exercise stress test is ordered. The test is considered to be abnormal if which of the following occurs? A. Systolic blood pressure increases during exercise. B. The heart rate reaches maximal value during exercise. C. Random premature ventricular beats occur at peak exercise. D. A 2 mm ST-segment depression is seen on the ECG at peak exercise.

(c) D. A 2 mm ST-segment depression is suggestive of cardiac ischemia and is considered to be an abnormal finding.

lobe with decreased tactile fremitus and dullness to percussion. Which of the following is the most likely cause? A. asthma B. consolidation C. pneumothorax D. pleural effusion

(c) D. A decreased tactile fremitus and dullness to percussion would be found in a pleural effusion.

Which of the following is the recommended treatment of a large volume hemothorax? A. perform pleurodesis B. drain the empyema C. administer antibiotics D. remove blood

(c) D. A hemothorax is defined as bleeding into the pleural space. A large volume hemothorax is treated by immediate insertion of a large bore thoracostomy tube to (1) drain existing blood and clot, (2) quantify the amount of bleeding, (3) reduce the risk of fibrothorax, and (4) permit apposition of the pleural surfaces in an attempt to reduce hemorrhage.

History & Physical/Gastrointestinal/Nutritional Which of the following is a common physical examination finding in early intestinal obstruction? A. high fever. B. profuse flatulence. C. rebound tenderness D. hyperactive, high-pitched bowel sounds

(c) D. Abdominal distention and high-pitched, hyperactive bowel sounds are common in early intestinal obstruction.

A 72 year-old man presents with acute left lower quadrant abdominal pain. He has nausea, vomiting, and constipation. He has a fever of 101° F and guarding and rebound tenderness in his left lower quadrant. His white blood cell count is elevated. He has no prior history of gastrointestinal disease. Which of the following is the most likely diagnosis? A. Inflammatory bowel disease B. Irritable bowel syndrome C. Viral gastroenteritis D. Acute diverticulitis

(c) D. Acute abdominal pain, fever, left lower abdominal tenderness, and leukocytosis are hallmark signs of acute diverticulitis.

The diagnostic gold standard to rule out pulmonary embolism is A. chest x-ray. B. spiral CT scan. C. ventilation/perfusion scan. D. pulmonary angiography.

(c) D. Although chest x-ray, arterial blood gases, and ventilation-perfusion scans may be obtained in the work-up of suspected pulmonary embolism, the arteriogram remains the "gold standard" for diagnosis.

153. Clinical Intervention/Cardiology A 58 year-old male who is otherwise healthy presents with chest pain and is found to have left main coronary artery stenosis of 75%. The most important aspect of his management now is A. daily aspirin to prevent MI. B. nitrate therapy for the angina. C. aggressive risk factor reduction. D. referral for coronary artery revascularization.

(c) D. Although medical therapy is important, revascularization is indicated when stenosis of the left main coronary artery is greater than 50%.

Diagnostic Studies/Urology/Renal A 20 year-old male presents with a hard mass on the testicle. There has been no previous infection or trauma to the area. Which of the following is the initial diagnostic evaluation to pursue? A. serum alpha fetoprotein levels B. serum human chorionic gonadotropin hormone C. CT scan of the pelvis D. ultrasound of the testicles

(c) D. An ultrasound of the testes will enable the clinician to discriminate between testicular tumors and epididymitis, orchitis, hematomas, hydroceles, and infiltrative diseases of the testes.

Diagnostic Studies/Urology/Renal A 20-year-old male presents with a hard mass on the testicle. There has been no previous infection or trauma to the area. Which of the following is the initial diagnostic evaluation to pursue? A. serum alpha-fetoprotein levels B. serum human chorionic gonadotropin hormone C. CT scan of the pelvis D. ultrasound of the testicles .

(c) D. An ultrasound of the testes will enable the clinician to discriminate between testicular tumors and epididymitis, orchitis, hematomas, hydroceles, and infiltrative diseases of the testes.

The treatment of choice for a patient with WPW (Wolff-Parkinson-White Syndrome) who has recurrent episodes of supraventricular tachycardia is which of the following? A. IV Verapamil (Calan, Isoptin) B. Digoxin (Lanoxin) C. Surgical ablation of the accessory pathway D. Radiofrequency ablation of bypass tracts

(c) D. Catheter ablation of bypass tracts is possible in more than 90% of patients and is the treatment of choice in patients with symptomatic arrhythmias. It is safer, more cost-effective, and just as successful as surgery.

A 57 year-old male presents with acute bilateral lower extremity weakness and urinary incontinence that began after he fell earlier today. His examination is significant for bilateral lower extremity sensory deficits and weakness along with decreased rectal sphincter tone. Which of the following is the most appropriate intervention? A. Epidural steroids B. Oral NSAIDs C. Physical therapy D. Surgery

(c) D. Cauda equina syndrome is a rare but serious surgical emergency because the duration of nerve compression is inversely correlated with the likelihood of full neurologic recovery

An 18 year-old male is involved in a motor vehicle accident with a question of cervical spine fracture. What is the imaging test of choice to initially evaluate this patient and clear his cervical c-spine? A. Positron emission tomography B. Magnetic resonance imaging C. Computed tomography D. Lateral radiograph

(c) D. Cervical spine x-rays are most commonly used as the initial screen for cervical spine injury. A cervical spine series consists of a lateral view, anteroposterior (AP) view, and an odontoid view. The lateral view detects up to 80% of traumatic spine injuries.

A 62 year-old male presents with a right hilar mass. Needle-biopsy of the mass reveals the presence of small-cell carcinoma and a bone scan reveals the presence of scattered hot spots throughout the skeleton. Which of the following is the most appropriate treatment? A. Lobectomy B. Pneumonectomy C. Thoracic radiation therapy D. Combination chemotherapy

(c) D. Combination chemotherapy is the treatment of choice for a patient with small-cell carcinoma of the lung. (u) A. Small-cell carcinoma of the lung is rarely treatable with surgical resection. Surgery may be indicated as part of the treatment protocol for small peripheral lesions without any evidence of metastasis. (u) C. While thoracic radiation therapy has been shown to be beneficial for patients with limited small-cell lung cancer, no benefit has been observed for patients with extensive disease defined as the presence of metastatic disease.

Clinical Therapeutics/Gastrointestinal/Nutritional A 19-year-old female presents with intermittent episodes of loose stools over the past 6 to 8 months. She admits to crampy abdominal pain and bloating that seems to be more associated with her stress level than with food intake. She denies fever, melena, or nocturnal symptoms. Examination reveals normoactive bowel sounds and mild lower abdominal tenderness on palpation without rebound tenderness or organomegaly. Which of the following is the treatment of choice in this patient? A. Cimetidine (Tagamet) B. Metronidazole (Flagyl) C. Metoclopramide (Reglan) D. Dicyclomine (Bentyl)

(c) D. Dicyclomine (Bentyl) is a gut antispasmodic that It can help relieve abdominal symptoms of cramping and bloating associated with irritable bowel syndrome.

Diagnostic Studies/Dermatology A 32-year-old female presents with complaints of gradual color change in a mole that has been present since birth. The patient also notes the recent onset of tenderness when her clothes rub up against it and itchiness for the past three weeks. An asymmetrical flat plaque with irregular and sharply defined margins with colorvariegation is noted on examination.Which of the following is the most appropriate diagnostic evaluation? A. Diascopy B. Patch testing. C. Acetowhitening D. Excisional biopsy

(c) D. Excisional biopsy is indicated for this pigmented lesion that exhibits several of the warning signs of melanoma including asymmetry, irregular border, color variegation and the presence of itching and tenderness for more than 2 weeks.

A 40-year-old patient presents with fever, night sweats, and a 3 cm, nontender lymph node in the neck. Chest x-ray shows mediastinal adenopathy. The most likely diagnosis is A. chronic lymphocytic leukemia. B. hairy cell leukemia. C. multiple myeloma. D. Hodgkin's disease.

(c) D. Hodgkin's disease typically presents with nontender lymphadenopathy in the neck and axilla and B-symptoms-fever, night sweats, malaise, weight loss. Mediastinal adenopathy is also common.

DIAG STUDIES The physician assistant is evaluating a patient suspected of having an iron deficiency anemia. When examining the patient's peripheral blood smear, the diagnosis is reinforced by noting A. shistocytes. B. Howell-Jolly bodies. C. macrocytic red blood cells. D. hypochromic, microcytic red blood cells.

(c) D. Hypochromic, microcytic red blood cells are common in patients with iron deficiency anemia.

98. Clinical Intervention/Cardiology Which of the following is the first complication seen with mitral stenosis? A. Aortic regurgitation B. Aortic stenosis C. Left ventricular failure D. Right ventricular failure

(c) D. In long standing, severe MS, patients may develop elevated right-sided pressures and right ventricular dysfunction due to blood backing up from the left atrium. These patients can present with signs and symptoms of right-sided heart failure such as peripheral edema.

A 71-year-old male with a history of diabetes and dyslipidemia presents to the emergency department with complaints of exertional chest pain for the past two hours. He gets some relief with rest but admits nausea & left-sided jaw pain. On examination, he appears diaphoretic and tachypnic. Electrocardiogram (ECG) is unchanged from a previous ECG 1 year ago. His cardiac enzymes are within normal limits. Which of the following is the most appropriate next step? A. Referral to cardiology for an outpatient thallium stress test B. Discharge patient home with nitroglycerine C. Transfer to cardiac catheterization lab D. Monitor with repeat enzymes and ECG in 4-6 hours

(c) D. In one-fourth to one-half of patients with acute MI, the first ECG does not demonstrate typical ST segment changes. Serial ECG's should be obtained to increase diagnostic yield. Serologic identification of myocyte necrosis is another beneficial diagnostic tool. CK-MB is detectable in the blood within 3-6 hours of the onset of the MI. Troponins begin to rise within 4-6 hours and remain elevated for 7-10 days.

Clinical Intervention/Urology/Renal A 23 year-old male being treated for an acute bacterial prostatitis has been taking antibiotics for less than 24 hours. He presents to the emergency room today with acute urinary retention for 12 hours. Which of the following is the most appropriate next step? A. Insert a Foley catheter. B. Initiate diuretic therapy. C. Schedule for cystoscopy. D. Insert a percutaneous suprapubic tube.

(c) D. Inserting a percutaneous suprapubic tube is the treatment of choice in a patient with acute bacterial prostatitis who develops acute urinary retention.

A 55-year-old female with a history of pulmonary embolus and hypertension presents with a recurrent deep vein thrombosis. The patient's medications include warfarin 3mg daily and hydrochlorothiazide 25 mg daily. The patient's INRs have been averaging 2.6 over the past 3 months. The patient denies complaint of shortness of breath or chest pain. What is the most appropriate next step in the treatment of this patient? A. IV heparin therapy B. Increase warfarin dose C. Perform embolectomy D. Insert an inferior vena cava filter

(c) D. Interruption of the inferior vena cava is indicated in patients with recurrent thromboembolism despite adequate anticoagulation to reduce the short-term incidence of pulmonary embolism in patients with proximal lower extremity DVT.

Clinical Intervention/Pulmonology A 55-year-old female with a history of pulmonary embolus and hypertension presents with a recurrent deep vein thrombosis. The patient's medications include warfarin 3mg daily and hydrochlorothiazide 25 mg daily. The patient's INRs have been averaging 2.6 over the past 3 months. The patient denies complaint of shortness of breath or chest pain. What is the most appropriate next step in the treatment of this patient? A. IV heparin therapy B. Increase warfarin dose C. Perform embolectomy D. Insert an inferior vena cava filter

(c) D. Interruption of the inferior vena cava is indicated in patients with recurrent thromboembolism despite adequate anticoagulation to reduce the short-term incidence of pulmonary embolism in patients with proximal lower extremity DVT.

Diagnosis/Pulmonology Which histologic type of lung cancer is typically centrally located? A. Adenocarcinoma B. Bronchoalveolar C. Large cell D. Squamous cell

(c) D. Most squamous cell lung cancers are centrally located.

Diagnosis/Urology/Renal A 54-year-old man comes to the urgent care because he was awoken suddenly from his sleep this morning with severe left flank pain radiating to his left testicle with associated nausea and vomiting. He is afebrile and vital signs are normal. He is constantly moving to find a comfortable position. On physical examination, left flank tenderness is noted with no direct testicular tenderness. Urinalysis reveals a pH of 5.5 and microscopic hematuria but is otherwise unremarkable. Which of the following is the most likely diagnosis? A. Acute epididymitis B. Appendicitis C. Diverticulitis D. Nephrolithiasis

(c) D. Nephrolithiasis usually presents as a sudden onset of colicky flank pain with associated nausea and vomiting. Urinalysis often reveals gross or microscopic hematuria.

Clinical Intervention/Urology/Renal = A patient with prostate cancer has a nonpalpable, focal lesion, and the patient is reluctant to have surgery at this time. Which of the following would best monitor disease progression? A. Periodic rectal exams B. Transrectal ultrasonography C. Measurements of serum acid phosphatase D. Measurements of prostate-specific antigen

(c) D. PSA measurement correlates well with volume and stage of disease and is the recommended examination for monitoring disease progression.

A patient presents complaining of periumbilical pain. Which of the following anatomical sites is this finding associated with? A. Bladder B. Stomach C. Pancreas D. Small bowel

(c) D. Pain from the small intestine, appendix, or proximal colon causes periumbilical pain

81. Diagnosis/Endocrinology A patient presents to the office with worsening fatigue, weight loss, and weakness. She notes that she is having recurrent bouts of abdominal pain and has been losing her pubic hair. Patient is found to have orthostatic hypotension. Which of the following conditions is most likely? A. Cushing's syndrome B. Pheochromocytoma C. Primary hyperparathyroidism D. Addison's disease

(c) D. Patients with Addison's disease have primary adrenal failure from an autoimmune problem in the adrenal gland or due to hemorrhage into the adrenal gland. These patients are not able to make glucocorticoids, mineralocorticoids, or sex hormones which result in hypotension, hyperpigmentation (from an increase in the ACTH and MSH hormones) and are hyponatremic.

A patient presents to the office with worsening fatigue, weight loss, and weakness. She notes that she is having recurrent bouts of abdominal pain and has been losing her pubic hair. Patient is found to have orthostatic hypotension. Which of the following conditions is most likely? A. Cushing's syndrome B. Pheochromocytoma C. Primary hyperparathyroidism D. Addison's disease

(c) D. Patients with Addison's disease have primary adrenal failure from an autoimmune problem in the adrenal gland or due to hemorrhage into the adrenal gland. These patients are not able to make glucocorticoids, mineralocorticoids, or sex hormones which result in hypotension, hyperpigmentation (from an increase in the ACTH and MSH hormones) and are hyponatremic.

A 74-year-old patient presents with signs and symptoms of heart failure. EKG shows the patient to be in atrial fibrillation at a rate of 80 bpm. Blood pressure is 120/76. The patient denies complaint of palpitations, chest pain, or syncope. Which of the following is the most important long-term therapy in this patient? A. verapamil (Calan) B. amiodarone (Cordarone) C. furosemide (Lasix) D. warfarin (Coumadin)

(c) D. Patients with atrial fibrillation have an increased risk of stroke, therefore these patients need anticoagulation with warfarin to an INR of 2.0-3.0.

A patient is scheduled for an elective cardioversion. Which of the following agents should be used prophylactically? A. Unfractionated heparin B. Low-molecular weight heparin C. Aspirin. D. Warfarin (Coumadin)

(c) D. Patients with more than 72 hours of atrial fibrillation or in whom duration of the arrhythmia is unknown, are at increased risk of having atrial thrombi and should be treated with rate control and anticoagulation for at least 3 weeks before an attempt at cardioversion. Anticoagulation with warfarin to an INR of 2-3 is recommended.

Which of the following is the most common presentation for an elderly female patient with primary hyperparathyroidism? A. Abdominal pain B. Renal lithiasis C. Acute pancreatitis D. Asymptomatic

(c) D. Patients with primary hyperparathyroidism are most commonly found to have this disease by an incidental finding of hypercalcemia on routine laboratory testing as a result of screening. Up to 0.1% of the adult population has this condition which is most commonly seen in females over age 50.

Clinical Intervention/Urology/Renal A 28-year-old man comes to the office for his yearly follow up of testicular cancer. He has successfully treated with an orchiectomy two years ago. Which of the following tumor markers can be used to identify early relapse in testicular cancer? A. Carcinoembryonic antigen (CEA) B. Prostate specific antigen (PSA) C. CA 125 glycoprotein D. Alpha fetoprotein (AFP)

(c) D. Periodic surveillance of alpha-fetoprotein is a sensitive method for detecting early relapse of testicular cancer.Hepatocellular and testicular cancer can be followed by the use of alpha fetoprotein as a tumor marker. CA-125 tumor marker is useful as a tumor marker in ovarian cancer. Carcinoembryonic antigen monitor colorectal cancer after treatment. Prostate specific antigen (PSA)

A 45 year-old male presents with a long history of ulcerative colitis and recent progressive complaints of right upper quadrant pain, weight loss, fever and most recently, a rapid onset of jaundice with pruritus. Labs revealed elevated bilirubin and alkaline phosphatase. Viral serologies were negative. An endoscopic cholangiogram showed areas of stenosis and dilation throughout the bile duct system. What is the most likely diagnosis? A. Choledocholithiasis B. Hepatic carcinoma C. Portal hypertension D. Primary sclerosing cholangitis

(c) D. Primary sclerosing cholangitis (PSC) results in diffuse intra- and extrahepatic duct sclerosing with dilatation proximal to these areas.

A 28 year-old female with diabetes mellitus type 2 sustains a partial thickness burn to her left upper arm and her chest when hot grease spilled on her at home. The burn to her arm is circumferential and the estimated total body surface burned is 18%. She has no allergies. The most appropriate treatment of this patient would include A. outpatient application of silver sulfadiazine. B. debridement of all intact blisters. C. IV cefazolin (Ancef, Kefzol). D. transfer to a burn center.

(c) D. Reasons for transfer to a burn center include a partial thickness burn covering greater than 10% of total body surface area. In addition, burns in patients with pre-existing medical conditions, such as diabetes, that could complicate their management, prolong recovery, or affect their outcome, is also a reason for transfer to a burn center.

A 41 year-old female presents to you for medical screening advice. Her 44 year-old sister passed away recently 18 months after diagnosis of metastatic colon cancer. Which of the following is the most appropriate advice for this patient? A. Double contrast barium enema now and repeat every 5 years if normal B. Rectal occult blood testing annually until age 50 then sigmoidoscopy every 3 years C. Rectal occult blood testing annually until age 50 then screening colonoscopy every 5 years D. Screening colonoscopy now and repeat every 3-5 years if normal

(c) D. Recommended screening in a single first degree relative with colorectal cancer diagnosed before age 60 is beginning colonoscopy at age 40 or ten years younger than age at diagnosis of youngest affected first-degree relative. Then if negative, every 5 years

The most reliable site from which to identify the causative organism in cases of osteomyelitis is the A. base of ulcer. B. blood. C. sinus tract. D. bone.

(c) D. Samples from needle aspiration of pus in the bone, or from a bone biopsy, are essential to determine the exact causative agent.

Which of the following forms of lung cancer is associated with the poorest prognosis? A. squamous cell B. adenocarcinoma C. large cell D. small cell

(c) D. Small cell lung cancer is the most common type of lung cancer that is metastatic at the time of discovery, and therefore has the poorest prognosis

Diagnosis/Pulmonology A 65-year-old male with a history of 50 pack year smoking presents with weight loss, cough and two episodes of hemoptysis. Chest x-ray reveals a hilar nodule with hilar adenopathy and mediastinal widening. What is the most likely diagnosis? A. Large cell lung cancer B. Adenocarcinoma C. Bronchiectasis D. Squamous cell lung cancer

(c) D. Squamous cell cancer is common in patients with a smoking history and presents with a central mass, hilar adenopathy, and mediastinal widening.

Which of the following is better visualized with transesophageal echocardiogram (TEE) than transthoracic echocardiogram? A. Ventricular wall motion B. Pulmonary arteries C. Right ventricle D. Left atrial appendage

(c) D. TEE allows 2-D and Doppler imaging of the heart through the esophagus. Given the close proximity of the esophagus to the heart, high-resolution images can be obtained, especially of the left atrium, mitral valve apparatus, and aorta

The main complication with the use of transjugular intrahepatic portosystemic shunt (TIPS) procedure is which of the following? A. increased portal pressures resulting in further esophageal varices B. increased portal pressures resulting in a worsening of cirrhosis C. Budd-Chiari syndrome D. increased risk of encephalopathy

(c) D. TIPS procedures involve the placement of a stent in the liver in order to shunt blood away from the portal vein into the hepatic vein which bypasses the cirrhotic liver parenchyma. Its main complication is encephalopathy from the accumulation of toxic substances in the brain since the liver no longer acts as a filter.

History & Physical/Dermatology Which of the following lesions is often associated with the nodular type of basal cell carcinoma? A. Spider vein B. Cherry angioma C. Spider angioma D. Telangiectatic vessels

(c) D. Telangiectatic vessels are often visible in the lesions seen with basal cell carcinoma.

DIAG STUDIES In renal insufficiency, which of the following is used to assess glomerular filtration rate (GFR)? A. urinalysis B. blood urea nitrogen C. renal plasma flow measure D. creatinine clearance

(c) D. The GFR provides an overall index of renal function. Creatinine is normally cleared by renal excretion; as renal GFR declines, serum creatinine will increase.

Renal cell carcinoma most commonly presents with which of the following symptoms or signs? A. hypocalcemia B. inguinal pain C. anemia D. hematuria

(c) D. The most common presenting symptom/sign of renal cell carcinoma is hematuria (approximately 60%). Flank pain or abdominal mass is present in about 30% of new cases

127. Clinical Intervention/Cardiology A 34 year-old male presents with an acute onset of fatigue and dyspnea. He has experienced repeated episodes of near-syncope and an unresolved chest discomfort described as a "fluttering" sensation over the past 3 hours. His electrocardiogram reveals no definable p waves and his R-R interval is irregular. His blood pressure is 88/60 mmHg. Which of the following is most appropriate for this patient? A. Initiate warfarin (Coumadin) therapy to an INR target of 2.0 B. Consult for radiofrequency ablation therapy C. Transfer to cardiac catherization lab D. Sedate for synchronized cardioversion

(c) D. The patient has symptoms and ECG findings consistent with atrial fibrillation. Urgent cardioversion is indicated with shock or severe hypotension, pulmonary edema, or ongoing myocardial infarction or ischemia.

Patient presents to the emergency department with right upper quadrant pain over eight hours, nausea, and vomiting. On exam there is a fever of 101.2 degrees F. Ultrasound shows a distended gallbladder. What is the most appropriate management of this patient? A. Oral analgesics B. Diagnostic peritoneal lavage C. Proton pump inhibitors D. Laparoscopic cholecystectomy

(c) D. The proper treatment for acute cholecystitis is IV fluids, antibiotics, pain control, and surgery. Cholecystectomy is the definitive treatment for acute cholecystitis and laparoscopic cholecystectomy is the procedure of choice.

Diagnostic Studies/Gastrointestinal/Nutritional A 3 week-old infant is evaluated for persistent projectile vomiting described as breast milk without bile or blood. The abdomen is distended before vomiting and a small, mid-epigastric mass is palpable after vomiting. Which of the following is the most appropriate diagnostic study for the evaluation of this patient? A. Barium enema B. Esophageal manometry C. H. pylori stool antigen D. Upper GI contrast radiographs

(c) D. This infant has pyloric stenosis and an upper GI series will reveal a narrowed distal stomach with double tract of barium.

Diagnostic Studies/Dermatology At the urging of his wife, a 47-year-old construction worker presents to your office with a skin lesion. He reports he has had a slowly growing nodule on the side of his nose for the past year or two. It recently developed a small central area of erosion. He denies pain, fever, chills, pruritus or other similar lesions. On physical exam, the lesion is approximately 1 cm as described above, with a few telangiectasias. Which of the following is the best diagnostic study for this condition? A. Skin scraping B. Dermascopy C. Excision D. Punch biopsy (c) D. This patient has a basal cell carcinoma (BCC), which is the most common form of skin cancer and occurs in sun-exposed areas of the skin. Any lesion suspected of being a BCC should be biopsied with a punch biopsy.

(c) D. This patient has a basal cell carcinoma (BCC), which is the most common form of skin cancer and occurs in sun exposed areas of the skin. Any lesion suspected of being a BCC should be biopsied with a punch biospy.

A 42 year-old male is brought to the emergency department with a stab wound to his right lateral chest wall. On physical examination, the patient is stable with decreased breath sounds on the right with dullness to percussion. An upright chest x-ray reveals the presence of a moderate pleural effusion. Subsequent diagnostic thoracentesis contains bloody aspirate. Which of the following is the next most appropriate intervention? A. Thoracotomy B. Needle aspiration C. Close observation D. Tube thoracostomy

(c) D. This patient has a hemothorax. Drainage of a hemothorax is best obtained through insertion of a chest tube (tube thoracostomy).

Which of the following typical findings would be revealed during a sigmoidoscopy on a patient with Crohn's disease of the intestine? A. Rectal pseudopolyps B. Diffuse ulceration and bleeding C. Sheets of WBCs with inflamed mucosa D. Intermittent longitudinal mucosal ulcers and fissures

(c) D. Ulcerations tend to be linear with transverse fissures in Crohn's disease. These skip lesions are common with Crohn's disease.

The most accurate method of diagnosing thrombophlebitis of the lower leg is A. impedance plethysmography. B. physical exam findings. C. Doppler ultrasound. D. venography.

(c) D. While impedance plethysmography, physical exam findings, and Doppler ultrasound are useful in diagnosing thrombophlebitis, venography is the most accurate method for diagnosis in the lower leg.

Clinical Therapeutics/Hematology A 6-year-old male presents with hemarthrosis of the left knee. Coagulation studies reveal the following results: PT 12.5 seconds (normal range 12-14 seconds), INR 1.0, aPTT 58 seconds (normal range 18-28 seconds), platelet count 430,000/microliter (normal range 150,000-450,000/microliter), and bleeding time 4 minutes (normal range 2-12 minutes). Which of the following is the best treatment option for this patient? A. Desmopressin acetate B. Corticosteroids C. Vitamin K D. Cryoprecipitate

(c) D.Hemophilia A presents with a prolonged aPTT and normal platelet count and function. Hemophilia A is treated with factor VIII concentrate or cryoprecipitate.

DIAG STUDIES In suspected subarachnoid hemorrhage with a negative head CT, which of the following studies should be used to help establish the diagnosis of subarachnoid hemorrhage? A. complete blood count B. lipid profile C. lumbar puncture D. electrocardiogram

(c). C. Although 95% of subarachnoid hemorrhages show blood on head CT, the remaining do not show evidence of hemorrhaging. A lumbar puncture should then be performed and the fluid examined for red blood cells or xanthochromia.

In suspected subarachnoid hemorrhage with a negative head CT, which of the following studies should be used to help establish the diagnosis of subarachnoid hemorrhage? A. complete blood count B. lipid profile C. lumbar puncture D. electrocardiogram

(c). C. Although 95% of subarachnoid hemorrhages show blood on head CT, the remaining do not show evidence of hemorrhaging. A lumbar puncture should then be performed and the fluid examined for red blood cells or xanthochromia.

63. 3. DIAGNOSIS A 59-year-old male complains of pain in his right leg for the last few months. He is normally able to walk two miles a day, but has noted pain when he climbs hills, which is relieved with rest or resuming walking on flat ground. He also complains of foot pain at rest. He does not smoke and denies injury to his back. He has been generally healthy with obesity and mildly elevated triglycerides. The most likely cause of the pain in this patient is A. sciatica. B. diabetic neuropathy. C. deep vein thrombosis. D. intermittent claudication.

(u) C. Deep vein thrombosis does not cause intermittent pain, but rather continuous aching pain not relieved by rest. (c) D. Symptoms of intermittent claudication and arterial occlusive disease include pain with exercise that is relieved by rest; pain in the feet at rest indicates severe circulatory compromise. The history of obesity and elevated triglycerides is consistent with the peripheral vascular disease. Although this patient could also be at risk for diabetes, the most likely immediate problem is vascular in nature.

patient with hx of colorectal cancer for whom colonoscopy is no indicated (meastases) which is the best alternative choice

*******CT scan of thorax, abdomen, and pelvis colonic wall thickening, enlarged lymph nodes, liver metastases, ascites, lung secondaries********* *CT colonoscopy* double-contrast barium enema mass lesion in the colon and/or as a characteristic "apple core" lesion Fecal occult blood tests Sigmoidoscope Fecal Immunochemical Tests Stool DNA

Pt pregnant woman with kidney stone treatment

******Stent placement****** Treatment of stones in pregnancy ranges from conservative management (eg, bed rest, hydration, analgesia) to more invasive measures (eg, stent placement, ureteroscopy with stone manipulation, percutaneous nephrostomy).

Tumor marker for pancreatic cancer CA 19-9 is a carbohydrate antigen

*****CA (carbohydrate antigen) 19-9 is a carbohydrate antigen **** ****CA 19-9 is used to monitor recurrence of pancreatic carcinoma.***** AFP is used to monitor recurrence of hepatocellular carcinoma and testicular cancer. CEA is used to monitor recurrence of colon carcinoma. CA 19-9 is used to monitor recurrence of pancreatic carcinoma. ( CA-125 is used to monitor recurrence of ovarian carcinoma.

Gastrointestinal/Nutritional : Diagnosis, Choledocholithiasis Common bile duct obstruction Proximal inflamation Obstructive jaundice Dilated hepatic bile ducts Dx = Utz Tx : ERCP 1 . Passage of stone through the cystic duct can obstruct CBD 2. Si/Sx = obstructive jaundice, ↑conjugated bilirubin, hypercholesterolemia, ↑ alkaline phosphatase Dx = Utz ~ CBD >9-mm diameter (Utz first line for Dx) Tx : ERCP Tx = laparoscopic cholecystectomy, or ERCP

****Ultrasound of the abdomen is the best test for checking the extra-hepatic biliary tree for ductal dilatation and choledocholithiasis.******* (u) B. HIDA scan is usually ordered to assess gallbladder function. It is mostly ordered if initial ultrasound is normal and there is still a high index of suspicion for gallbladder disease. (u) C. Flat plate of the abdomen will only identify about 10 to 15% of gallstones. (u) D. ERCP is performed to remove gallstones that have become lodged in the common bile duct. It is not an initial study that is performed.

Pt who came from costa rica with tuberculosis complication

**adrenal gland insufficiency** Mycobacterium tuberculosis complex spreads to the adrenal glands hematogenously causing adrenal gland insufficiency .

Pre-Operative/Post-Operative Care : Diagnosis, Pernicious anemia Parenteral (intramuscular) administration of Vitamin B-12 (cobalamin) in the form of cyanocobalamin (CN-Cbl) and hydroxocobalamin (OH-Cbl). o High dose oral B12

*Antibodies to intrinsic factor or parietal cell antibodies* — may be present in pernicious anaemia *Vitamin B12 level — blood level may be low when deficient in B12* Schilling test, the intestines' ability to absorb vitamin B12 is measured.

Pt with GERD is at risk to develop

*Barrett's esophagus* Chronic GERD can lead to Barrett's esophagus, a condition in which any extent of the metaplastic columnar epithelium that predisposes to cancer development replaces the stratified squamous epithelium that normally lines the distal esophagus. Esophageal columnar metaplasia predisposes to the development of adenocarcinoma

Pre-Operative/Post-Operative Care : Diagnostic Studies, Carotid stenosis Si/Sx = carotid bruit, transient ischemic attacks amaurosis fugax (transient monocular blindness), accidents (CVAs) that result in permanent neurologic changes Tx= modification of risk factors important, anticoagulation, and use of antiplatelet agents (aspirin, dipyridamole) intended to prevent thrombosis. Surgical therapy is carotid endarterectomy or Carotid angioplasty and stenting pts usually placed on postoperative aspirin therapy asymptomatic carotid stenosis <70% antiplatelet therapy and cardiovascular risk reduction adjunct carotid endarterectomy asymptomatic carotid stenosis ≥70%: good surgical candidate carotid endarterectomy plus antiplatelet therapy and cardiovascular risk reduction *asymptomatic carotid stenosis ≥70%: poor surgical candidate carotid artery stenting plus antiplatelet therapy and cardiovascular risk reduction*

*Dx= duplex ultrasonography* = elevated blood flow velocities along with visualization of details of the plaque within the carotid arteries *Dx = computed tomography angiography (CTA)* of head, neck, and chest visualization of the narrowed carotid artery lumen; expressed as a specific percent stenosis

Men 2 related disorder =Medullary thyroid cancers (MTC) carcinoma

*Medullary thyroid cancers (MTC)* MEN 1 = Parathyroid tumors, pancreatic tumors, and pituitary tumors. MEN 2a = Medullary thyroid cancers (MTC), pheochromocytoma, and parathyroid tumors. MEN 2b = Medullary thyroid cancers, pheochromocytoma and neuromas. Familial Medullary Thyroid Carcinoma (FMTC) is medullary thyroid cancer

Pulmonology : Diagnostic Studies, Pleural effusion 1st Tests To Order *PA and lateral CXR blunting of the costophrenic angles* *pleural ultrasound fluid in the pleural space* LDH and protein in pleural fluid and serum

*↑Fluid in the pleural space, transudative or exudative* *Presents on CXR with blunting of the costophrenic angle and causes dullnessto percussion, decreased tactile fremitus, and decreased breath sounds on the exam* *Gold standard = thoracentesis* ○ Both diagnostic and therapeutic• *Tx Thoracentesis*

A 42 year-old male with a history of constipation presents with complaints of severe pain with defecation described as feeling like he is "tearing apart." He has also noted occasional small amounts of blood on toilet paper. External examination of the rectum is unremarkable and an internal rectal exam cannot be performed due to severe pain when attempted. Which of the following is the most likely diagnosis? A. Proctitis B. Anal fissure C. Rectal prolapse D. Internal hemorrhoids

. (c) B. Anal fissures are easily diagnosed from history alone with the classic finding of severe pain upon defecation. Constipation is also a common cause of the trauma that leads to development of a fissure.

Urology/Renal: Clinical Intervention, Renal artery stenosis Renal Artery Stenosis Causes are atherosclerotic plaques and fibromuscular dysplasia

1st antihypertensive therapy + lifestyle modification + statin + antiplatelet agent 2nd renal artery stenting + continuation of medical therapy + post-stent clopidogrel 3rd surgical reconstruction of the renal arteries Renal angioplasty and stenting of the renal artery will dilated the narrowed area interfering with blood supply to the kidney or Renal artery bypass surgery.

Gastrointestinal/Nutritional : Diagnosis, Peptic ulcer disease Peptic ulcers • SijSx = epigastric pain Perforated ulcers present with sudden upper abd pain, shoulder pain, GI bleed Peptic Ulcer Disease o Most common cause of Upper gastrointestinal bleeding (UGIB) o H pylori are the major cause of peptic ulcer disease. o RFs: smoking, H. pylori, NSAIDS, ASA, steroids o Duodenal > gastric o Gastric: pain greater with food o Duodenal: pain decreases food o Proton pump inhibitors, endoscopy Complications: perforation, gastric outlet syndrome

1st Tests To Order Helicobacter pylori breath test or H Pylori stool antigen test Upper Endoscopy with biopsy peptic ulcer CBC microcytic anemia stool heme test changes color if occult blood is present

Gastrointestinal/Nutritional: Diagnostic Studies, Barrett Esophagus Barrett's esophagus (1) Chronic GERD ↑ metaplasia from squamous to columnar epithelia in the lower esophagus (2) Requires close surveillance with endoscopy and aggressive Tx as 30% progress to adenoCA (3) Multiple FDA-approved Tx, including radiofrequency ablation, photodynamic Tx, cryotherapy, endoscopic mucosal resection Treatment nondysplastic Barrett esophagus 1st proton-pump inhibitor adjunct radiofrequency ablation 2nd antireflux surgery with low-grade dysplasia 1st radiofrequency ablation with or without endoscopic mucosal resection with high-grade dysplasia 1st radiofrequency ablation with or without endoscopic mucosal resection plus proton-pump inhibitor 2nd esophagectomy

1st Tests To Order Test Result upper GI endoscopy with biopsy to assess distal esophagus epithelium metaplastic changes;***Transformation of normal squamous epithelium to columnar epithelium*** Barium esophagogram identifies hiatal hernia and reflux

o Neurology/Neurosurgery: Diagnostic Studies, hemorrhagic stroke Stroke is an acute neurologic deficit caused by cerebrovascular etiology. It is further subdivided into ischemic stroke and hemorrhagic stroke. Ischemic stroke is lack of blood perfusion due to occlusion or critical stenosis of a cerebrospinal artery, and hemorrhagic stroke is due to rupture of a cerebrospinal artery, resulting in intraparenchymal, subarachnoid, and intraventricular hemorrhage.

1st Tests t non-infused head CT Result hyperdense lesion Most important test, as differentiates hemorrhagic from ischemic stroke. image May be combined with CT angiography. MRI brain

Gastrointestinal/Nutritional: Clinical Intervention, Gastroesophageal reflux • SifSx = position-dependent (supine worse) substernal or epigastric burning pain, regurgitation, dysphagia, hoarse voice • Dx proton-pump inhibitor trial symptom improvement esophagogastroduodenoscopy (EGD) may show esophagitis (erosion, ulcerations, strictures) or Barrett's esophagus ambulatory pH monitoring pH <4 more than 4% of the time is abnormal esophageal manometry may suggest achalasia, esophageal spasm, or other motor disorders

1st line treatment Lifestyle modification; weight loss; avoidance of instigating foods such as c(e.g., chocolate, caffeine, alcohol, acidic and/or spicy foods); avoid eating right before going to sleep. o 2nd line treatment H2 blockers o 3rd line treatment If H2 blockers fail, try proton pump inhibitors (PPIs) Acute Tx Line 1st standard-dose proton-pump inhibitor plus lifestyle changes Ongoing Patient Group Tx proton-pump inhibitor-responsive 2nd Surgery (open Evidence or laparoscopic ) is reserved mainly for people who have had a good response to proton-pump inhibitors (PPIs) but who do not wish to take long-term medical treatment.

Neurology/Neurosurgery : Clinical Intervention, Subarachnoid hemorrhage Occurrence of a sudden, severe headache is characteristic of SAH. It is the most important clue to diagnosis and is often described as "the worst ever headache". Dx: CT scan Lumbar puncture

1st line: Goal is to decrease intracranial pressure (ICP) Give ccb nimodipine, Nicardipine to decrease the chance of vasospasm, raise the head of the bed, and administer IV fluids. o 2nd line treatment for an SAH Surgical evacuation of blood via burr holes ● Supportive: bed rest, stool softener, lower ICP. surgical clipping or coil embolization ● +/- lower BP gradually (ex. Nicardipine, Nimodipine, labetalol)

Femoral Hernias diagnosis giFemoral Hernias 1. More common in women 2. Si/Sx = bulge above or below the inguinal ligament, ↑ risk of incarceration 3. Dx = clinical and/or surgical clinical diagnosis groin ultrasound scan CT of groin herniography MRI of groin 4. Tx =surgical repair should not be delayed irreducible, strangulated, or obstructed hernia, or uncertain viability of bowel open repair small, asymptomatic hernia: watchful waiting large or symptomatic hernia open mesh repair recurrent or bilateral hernia laparoscopic inguinal herniorrhaphy or open preperitoneal repair with mesh placement

3. Dx = clinical and/or surgical clinical diagnosis groin ultrasound scan CT of groin herniography MRI of groin

Patient with exarcerbation of ulcerative colitis when should get colonoscopy done Longstanding ulcerative colitis is associated with an increased risk of colon cancer. Patients should receive an initial screening colonoscopy eight years after the onset of pancolitis and 12 to 15 years after the onset of left-sided disease; follow-up colonoscopy should be repeated every two to three years.

????????

pt with previous hypovolimic shock is given IV fluid 09% sodium chloride for what acid-base imbalance is this patient is D/C discharge discontinue ??????????

????????????????????? Hypovolemia is characterized by sodium depletion (salt depletion)

Urology/Renal: Diagnosis, Hydrocele

A hydrocele is a fluid-filled mass that is nontender to palpation. Diagnosis is readily made by transillumination. A hydrocele is a nontender scrotal mass that transilluminates

Endocrinology: Scientific Concepts, Addison disease Causes= autoimmune (most common), granulomatous dz, infarction, HIV, o Idiopathic atrophy of the adrenal glands o Iatrogenic due to surgical removal of both glands o Infections (eg. TB/Histoplasmosis) o Decreased cortisol from adrenal cortex and oftentimes decreased aldosterone o Hyperpigmentation due to increased levels of ACTH causing increased Melanocyte Stimulating Hormone (3) Si/Sx = fatigue, anorexia, nausea/vomit, constipation, diarrhea, salt craving (pica), hypotension, hyponatremia, hyperkalemia ( 4) Dx = hyperpigmentation, increased ACTH, decreased cortisol response to ACTH

A rare, chronic endocrine system disorder in which the adrenal glands do not produce sufficient steroid hormones (glucocorticoids and mineralocorticoids). o Primary adrenal insufficiency occurs when the adrenal glands are damaged and cannot produce enough cortisol. The adrenal hormone aldosterone may also be lacking. Causes= autoimmune (most common), granulomatous dz, infarction, HIV, o Idiopathic atrophy of the adrenal glands o Iatrogenic due to surgical removal of both glands o Infections (eg. TB/Histoplasmosis) o Decreased cortisol from adrenal cortex and oftentimes decreased aldosterone o Hyperpigmentation due to increased levels of ACTH causing increased Melanocyte Stimulating Hormone (3) Si/Sx = fatigue, anorexia, nausea/vomit, constipation, diarrhea, salt craving (pica), hypotension, hyponatremia, hyperkalemia

Tumor Markers

A. AFP is used to monitor recurrence of hepatocellular carcinoma and testicular cancer. (c) B. CEA is used to monitor recurrence of colon carcinoma. (u) C. CA 19-9 is used to monitor recurrence of pancreatic carcinoma. (u) D. CA-125 is used to monitor recurrence of ovarian carcinoma. Alpha-fetoprotein (AFP) Liver cancer CA19-9 Cancer types: Pancreatic cancer, gallbladder cancer, bile duct cancer, and gastric cancer CA-125 Ovarian cancer Carcinoembryonic antigen(CEA) Colorectal cancer Prostate-specific antigen (PSA) Prostate cancer BRCA1 and BRCA2 gene mutations Ovarian cancer

When to stop Blood thinners

ASA 14 days, if indicated for TIA/CVA/MI: 7d Persantine 7 days NSAIDS 3-7 days Stop warfarin five days before the procedure Clopidogrel 4-7 days Herbal 14 day

Cardiovascular: Diagnostic Studies, Abdominal aneurysm screening "

Abdominal ultrasound í Monitor aneurysm growth. If your aneurysm is large, you may need an ultrasound every 6 to 12 months. If your aneurysm is small, you may need one every 2 to 3 years. " CT and MRA í if a view more detailed than an ultrasound is needed. Assesses the aneurysm's relation to the blood vessels of the kidney or other organs. Especially before surgery. CT is used to watch the growth of a thoracic aortic aneurysm. " TTE or a TEE í may be done to diagnose thoracic aortic aneurysm. " Angiogram í Assess size of the aneurysm and if there are aortic dissections, blood clots, or other blood vessel involvement.

Pt with a hx of arrhythmia ( atrial fibrillation) with abdominal pain out of proportion diagnostic

Acute mesenteric ischemia

Pre-Operative/Post-Operative Care : Diagnosis, Lactic acidosis ● Lactic acidosis is a medical condition characterized by the buildup of lactate (especially L-lactate) in the body, which results in an excessively low pH in the bloodstream. It is a form of metabolic acidosis, in which excessive acid accumulates due to a problem with the body's metabolism of lactic acid.

Although blood gas sampling is not always essential for the diagnosis of acidosis, a low pH (in either a venous or arterial sample) does support the diagnosis. If the pH is low (under 7.35) and the bicarbonate levels are decreased (<24 mmol/L), metabolic acidemia is present, and metabolic acidosis is presumed. GI loss of bicarbonate serum anion gap: <12 mEq/L urine anion gap: low (<0 mEq/L) Renal loss of bicarbonate - renal tubular acidosis type 1 and 2 serum anion gap: <12 mEq/L

Dermatology: Diagnostic Studies, Melanoma Malignant melanoma Seen in Lightly pigmented , of individuals with increased sun exposure Excision , chemo if metastasis likely High rate metastasis~ #1

An excisional biopsy is indicated for suspected cases of melanoma. Excisional Biopsy, then a sentinel lymph node biopsy diagnose with ABCDEs • Asymmetry = malignant; symmetry = benign Border = irregular; benign = smooth • Color = multicolored; benign =one color • Diameter >6 mm; benign= <6 mm • Elevation = raised above skin; benign = flat • Enlargement = growing; benign = not growing

Gastrointestinal/Nutritional: History and Physical, Anal fissure

Anal Fissure o Patient will be complaining of rectal pain and bleeding which occurs with or shortly after defecation and onstipation o PE will show fissure located in the posterior midline o Diagnosis is made by visual inspection

o Cardiovascular: Diagnostic Studies, Arterial embolization "

Angiogram í examines the blood vessels for abnormalities " Doppler ultrasound í watches blood flow " MRI í takes images of the body to locate blood clots

Gastrointestinal/Nutritional : Health Maintenance, Anorexia nervosa

Anorexia Si/Sx a. By definition, anorexic pts are 85% below their expected body weight because they do not eat enough, b. Amenorrhea occurs 2° to weight loss Key Diagnostic Factors Frequency weight loss fear of gaining weight disturbed body image calorie restriction bingeing and/or purging misuse of laxatives, enemas, and/or diuretics amenorrhea decreased subcutaneous fat

Question about cholelithiasis . Pt with US showing stones but no signs of obstruction cholelithiasis Stone formation in GB Cholicky pain worse with food DX RUQ US TxElective cholecystectomy

Answer Elective cholecystectomy

Patient with chronic post-pandrial abdominal pain and N/V what diagnosis technique would you order

Answer Mesenteric angiogram

Question: What is the surgical procedure used in the treatment of chronic anal fissures? Answer: An open or closed partial lateral internal sphincterotomy.

Answer: An open or closed partial lateral internal sphincterotomy. Anal Fissure o Patient will be complaining of rectal pain and bleeding which occurs with or shortly after defecation o PE will show fissure located in the posterior midline o Diagnosis is made by visual inspection o Treatment is stool softeners, protective ointments, sitz baths

Question: What is quadruple therapy for peptic ulcer disease?

Answer: Bismuth subsalicylate, metronidazole, tetracycline, and a PPI.

Question: What is the preferred method of prophylactic endoscopic treatment of esophageal varices to help prevent future bleeding?

Answer: Variceal band ligation as endoscopic variceal ligation decreases the risk of first bleeding and the mortality rate.

o Pre-Operative/Post-Operative Care: Diagnosis, Respiratory acidosis When alveolar gas exchange units are unable to sufficiently excrete carbon dioxide, this leads to an increase in the arterial carbon dioxide levels above the normal range of 35 to 45 mmHg (4.7-6.0 kPa). With the increase in carbon dioxide, hydrogen ions accumulate, causing the arterial pH to fall below the normal range (<7.35). o Primary increase in Pco2 with or without compensatory increase in HCO3−; pH is usually low but may be near normal. o ABG and serum electrolytes o Diagnosis of cause usually clinical

Arterial blood gas (ABG) analysis for suspected respiratory acidosis. Acidemia is documented by the presence of a decreased pH (< 7.35) on ABG analysis. The presence of an increased partial pressure of arterial carbon dioxide (PaCO2) (>45 mm Hg) indicates a respiratory etiology of the acidemia. Most common abnormal serum electrolyte finding in chronic respiratory acidosis is the presence of a compensatory increase in serum bicarbonate concentration.

Pathophysiology of Addison's disease

Autoimmune attack to the adrenal glands A rare, chronic endocrine system disorder in which the adrenal glands do not produce sufficient steroid hormones (glucocorticoids and mineralocorticoids). o Primary adrenal insufficiency occurs when the adrenal glands are damaged and cannot produce enough cortisol. The adrenal hormone aldosterone may also be lacking. Causes= autoimmune (most common), granulomatous dz, infarction, HIV, o Idiopathic atrophy of the adrenal glands o Iatrogenic due to surgical removal of both glands o Infections (eg. TB/Histoplasmosis) o Decreased cortisol from adrenal cortex and oftentimes decreased aldosterone o Hyperpigmentation due to increased levels of ACTH causing increased Melanocyte Stimulating Hormone

Dermatology: Diagnosis, Third-degree burn V. Burns A. Partial Thickness 1. 1 o and 2° burns are limited to epidermis and superficial dermis 2. Si/Sx = skin is red, blistered, edematous, skin underneath blister is pink or white in appearance, very painful 3. lnfxn may convert to full-thickness burns B. Full Thickness 1. 3° and 4° burns affect all layers of skin and subcutaneous tissues 2. Si/Sx =skin initially is painless, dry, white, charred, cracked, insensate 3. 4° burns also involve muscle and bone 4. All full-thickness burns require surgical excision and skin grafting. 5. Percentage of body surface area (BSA) affected (Figure 2.3 and Table 2.7) 6. Tx =Resuscitation, monitor fluid status, transfer to burn center. a. Consider any facial burns or burning of nasal hairs as a potential candidate for acute respiratory distress syndrome (ARDS) and airway compromise b. Fluid resuscitation (1) Parkland formula=% BSA x weight (kg) x 4, formula used to calculate volume of crystalloid needed (2) Give half of fluid in first 8 hrs; remainder given over next 16 hrs c. CXR to rule out inhalation injury d. labs ~ PT /PTT, CBC, type and cross, arterial blood gas (ABGs), electrolytes, UA e. Irrigate and debride wound, topical antibiotics (silver sulfadiazine, mafenide, Polysporin), tetanus prophylaxis, and stress ulcer prophylaxis f. Transfer to burn center if pt is very young <1 0 yrs old or >50 yrs old. Burns >20% BSA, full-thickness burns >5% BSA, coexisting chemical or electrical injury, facial burns, genitalia, perineum, hands, feet, or preexisting medical problems g. Make pt NPO until bowel function returns; pt will have extremely t protein and caloric requirements with vitamin supplementation

B. Full Thickness 1. 3° and 4° burns affect all layers of skin and subcutaneous tissues 2. Si/Sx =skin initially is painless, dry, white, charred, cracked, insensate 3. 4° burns also involve muscle and bone 4. All full-thickness burns require surgical excision and skin grafting. 5. Percentage of body surface area (BSA) affected Full thickness burn. o This type of burn destroys the outer layer of skin (epidermis) and the entire layer beneath (or dermis).

A patient with hypovolemic shock would most likely exhibit which of the following signs? A. increased CVP; decreased BP; increased pulse rate B. decreased CVP; decreased BP; increased pulse rate C. increased CVP; increased BP; decreased pulse rate D. decreased CVP; increased BP; decreased pulse rate

B. Hypovolemic shock is a condition with a decrease in the amount of circulating blood volume in the intravascular system. A decrease in the amount of circulating volume will result in a decrease in the CVP pressure which is an indirect measurement of the amount of blood in the right ventricle. Less blood in the vascular system means decreased blood pressure. Since there is less blood in the circulation, the body will attempt to compensate for this by increasing the number of contractions (pulse rate) and the force of those contractions due to increased sympathetic stimulation.

Neurology/Neurosurgery: Diagnosis, Idiopathic facial paralysis or Bell palsySigns and symptoms of Bell's palsy come on suddenly and may include: Rapid onset of mild weakness to total paralysis on one side of your face — occurring within 24 hs to 72 hs. Facial droop and difficulty making facial expressions, such as closing your eye or smiling, drooling, pain around the jaw or in or behind your ear on the affected side, increased sensitivity to sound on the affected side, headache, a decrease in your ability to taste Changes in the amount of tears and saliva you produce

Bell palsy remains a clinical diagnosis of exclusion. The diagnostic history should reflect unilateral facial palsy of acute onset, affecting all branches in equal fashion, whose deficits are fully evolved within 72 hours. Ipsilateral postauricular pain, mild to moderate otalgia, hypo- or dysgeusia, hyperacusis or phonophobia, and subjective tongue and facial hypoesthesia are commonly reported.Onset of clinical recovery is nearly always demonstrated within 4 to 6 months of symptom onset.

Neurology/Neurosurgery: Clinical Therapeutics, Idiopathic facial paralysis or Bell palsy

Bell's palsy will resolve completely without treatment in approximately 60% of all cases between 4 or 6 months. Tx Line corticosteroid Prednisone eye protection antiviral therapy valacyclovir acyclovir surgical decompression

Urology/Renal : Diagnosis, Urinary retention

Blockage of urinary flow, which can occur at any level in the urinary tract. May affect one or both kidneys, depending on the level of obstruction. A number of underlying conditions can result in obstructive uropathy, most commonly urolithiasis and benign prostatic hyperplasia. Diagnostic Tests urinary dipstick normal or positive nitrites, leukocyte esterase, and/or blood in presence of infection; microscopic hematuria in renal colic renal ultrasound hydronephrosis affecting the upper urinary tract serum BUN and creatinine normal or elevated CBC normal or elevated WBC if infection present; low hemoglobin and hematocrit if bleeding CT pyelogram stones in the urinary tract identified as causing obstruction prostate specific antigen elevation indicative of prostatic disease

A 25 year-old female presents with right lower quadrant pain, right flank pain, nausea, and vomiting. Her temperature is 39.6 degrees C. There is right CVA tenderness and RLQ tenderness. Pelvic exam is unremarkable. Urinalysis reveals pH 7.0, trace protein, negative glucose, negative ketones, positive blood, and positive nitrates. Specific gravity is 1.022. Microscopic shows 102 RBCs/HPF, 50-75 WBCs/HPF, rare epithelial cells, and WBC casts. The most likely diagnosis is A. acute salpingitis. B. nephrolithiasis. C. acute pyelonephritis. D. appendicitis.

C. Acute pyelonephritis presents with flank pain, fever, and generalized muscle tenderness. Urinalysis shows pyuria with leukocyte casts.

Hematology: Diagnosis, Hemophilia Hemophilia (1) X-linked deficiency of factor VIII (hemophilia A) or X-linked deficiency of factor IX (hemophilia B = Christmas dz) (2) Si/Sx = hemarthroses (bleeding into joint), ecchymoses with minor trauma, ↑ PTT, nml PT, nml bleeding time (3) Dx = ↓ factor VIII (hemophilia A) and factor IX (hemophilia B) levels (4) Tx = recombinant factor VIII or factor IX concentrate

CBC usually normal; low Hb if bleeding has been severe or prolonged activated partial thromboplastin time (aPTT) usually prolonged; may not be prolonged in mild cases (factor levels >30%) plasma factor VIII and IX assay decreased or absent factor VIII or IX levels

Subdural hematoma Causes = trauma, coagulopathy, common in ederly Sx may start 1-2 wks after trauma • Dx = CT Findings Crescent-shaped, concave hyperdensity that does not cross the midline. Cross sutures lines suture lines • Px worse than epidural due to the risk of concurrent brain injury Tx Evacuate hematoma via burr holes

CT Findings Crescent-shaped, concave hyperdensity that does not cross the midline. Cross sutures lines

Gastrointestinal/Nutritional : Diagnostic Studies, Diverticular abscess

CT with oral and IV contrast shows diverticular abscess presence, thickening of bowel wall, mass, abscess, streaky mesenteric fat; may show gas in the bladder in cases of fistula If the abscess is found Percutaneous cath drainage if accessible ● CT scan - test of choice with oral or IV contrast ● Leukocytosis ● Positive guaiac Dx: CT scan of the abdomen with contrast is the imaging of choice TX: percutaneous cath drainage of abscess. Open Surgery if non accessible

Reynolds pentad

Charcot's triad (right upper quadrant pain, jaundice, and fever) hypotension and an altered mental status

Patient presents with a Charcot's cholangitis triad right upper quadrant abdominal pain; fever, usually with chills; and jaundice It occurs as a result of ascending cholangitis (an infection of the bile duct in the liver). Plus increase pancreatic enzymesCholangitis Choledocholithiasis + infection Charcot triad: RUQ pain+fever w/chills+jaundice Charcot's triad (right upper quadrant pain, jaundice, and fever) with shock (hypotension, tachycardia) and an altered mental status Dx RUQ US Tx: Emergency ERCP

Cholangitis is characterized by a history of biliary pain, fever, chills, and jaundice associated with episodes of abdominal pain.Cholangitis is a clinical diagnosis made by the triad of right upper quadrant pain, fever and jaundice. A gallstone typically causes obstruction leading to bacterial overgrowth. Patients can become ill quickly requiring broad-spectrum antibiotics and relief of the obstruction. The ultrasound should demonstrate intra and extrahepatic biliary dilation.

Gastrointestinal/Nutritional: Clinical Intervention, Internal hemorrhoid

Clinical Intervention, Internal hemorrhoid o Rubber Band Ligation office procedure used to treat internal hemorrhoids, a tight band is placed around the base of hemorrhoid to cut off its blood supply. o Sclerotherapy This procedure involves injecting a chemical into hemorrhoid. The chemical causes hemorrhoid to shrink and stops it from bleeding. o Hemorrhoidectomy or Hemorrhoidopexy large, prolapsed internal hemorrhoids. Hemorrhoidopexy is less painful recovery vs. hemorrhoidectomy.

Endocrinology: Diagnosis, Hypoparathyroidism

Clinical manifestations: ○ Signs of hypocalcemia: carpopedal spasms, Trousseau's & Chvostek's signs, increased DTR (deep tendon reflexes) history of progressive severe muscle cramps, extremity paresthesias, and lethargy which began shortly after a thyroidectomy for a malignant thyroid lesion. likely diagnosis? Hypoparathyroidism o Blood tests: Low calcium, low PTH, high phosphorus, low magnesium. o Facial muscle twitching.

Gastrointestinal/Nutritional : Diagnostic Studies, Colon cancer and tumor markers

Colon CA Biopsy of the lesion on colonoscopy/sigmoidoscopy c. Obtain preoperative carcinoembryonic antigen (CEA) levels to follow disease, if the CEA is elevated, it should be negligible after resection allowing it to be followed postoperatively for recurrence of disease. 2. Screening a. Age >50 yrs without risk factors (strong family Hx, ulcerative colitis, etc.)~ yearly stool occult blood tests, flexible sigmoidoscopy q3-S yrs or colonoscopy q10 yrs or barium enema qS-1 0 yrs b. Colonoscopy/barium enema if polyps found c. Pts with risk factors require more frequent and full colonoscopies 3. Dx a. Endoscopy or barium enema-biopsy not essential b. Obtain preoperative CEA to follow disease; these levels will be elevated before any physical evidence of disease 4. Surgical = resection and regional lymph node dissection 5. Adjuvant Tx for metastatic dz = FOlFOX (Fluorouracil (5-FU) + leucovorin or levamisole and oxaliplatin) 6. Follow-Up a. Hx, physical, and CEA level q3 mos for 3 yrs, then follow-up every 6 mos for 2 yrs b. Colonoscopy at 6 mos, 12 mos, 3 yrs, and 5 yrs c. CT and MRI for suspected recurrences

Pt with a history of colon polyps in the rectum and sigmoid colon what do you use Colonoscopy and sigmoidoscopy

Colonoscopy and sigmoidoscopy are screening tests that use a thin flexible tube with a camera at the end to look at the colon but differ in the areas they can see. A colonoscopy examines the entire colon, while a sigmoidoscopy covers only the lower part of the colon, also known as the rectum and sigmoid colon.

Gastrointestinal/Nutritional : Clinical Intervention, Hemorrhoids PPP 163

Conservative treatment: high-fiber diet, increased fluids. Warm sitz bath and topical rectal corticosteroids (+/- analgesics like lidocaine) may be used for pruritus and discomfort/thrombosis ● Procedures: if failed conservative management, debilitating pain, strangulation or stage IV. Options include rubber band ligation (MC used), sclerotherapy or infrared coagulation ● Hemorrhoidectomy: for all stage IV or those not responsive to the aforementioned therapies.

A 25-year-old truck driver presents with a 1-day history of throbbing rectal pain. Your examination shows a large thrombosed external hemorrhoid. Which one of the following is the preferred initial treatment for this patient? A Elliptical excision of the thrombosed Hemorrhoid B Infrared coagulation C Rubber band ligation of the hemorrhoid D Stool softeners and a topical analgesic

Correct Answer ( A )

A 65-year-old Norwegian man presents with hematemesis. His past medical history includes alcohol abuse, tobacco use, Helicobacter pylori infection, heavy intake of smoked fish and diabetes mellitus. Endoscopic examination reveals a large pyloric mass. Which of the following is the most common cause of this type of gastric cancer? A Diabetes mellitus B Helicobacter pylori C Smoked fish diet D Tobacco use

Correct Answer ( B ) Adenocarcinoma is the main type, a. Risk factors include male gender, African-American or Asian race, age >70-years-old, family history, tobacco, and alcohol use, salted, smoked, and nitrate-preserved foods, H.pylori infection (main cause, up to 65-80% of all gastric cancers), Symptoms include nausea, vomiting, heartburn, indigestion, loss of weight and appetite, bloating, central or upper abdominal pain, hematochezia (blood per rectum), hematemesis (vomiting of blood), melena (black tarry stool) and anemia associated fatigue. Metastatic spread may reveal palpable left supraclavicular or umbilical nodes (Virchow ad Sister Mary Joseph nodes, respectively). Diagnosis is confirmed by endoscopy with biopsies, but other modalities, like barium roentgenogram (barium swallow, or upper GI series) or computed tomography are also used. After cancer has been staged, treatment options include surgery (most common), chemotherapy and radiotherapy. Surgical resection of the tumor and surrounding lymph nodes

A patient with alcohol abuse and cirrhosis presents with acute upper gastrointestinal bleeding, hypotension and tachycardia. His past medical history also includes portal hypertension. You suspect ruptured esophageal varices as the source of bleeding. In addition to cardiopulmonary stabilization and blood transfusion, which of the following is the most appropriate emergent interventional treatment? A Distal splenorenal shunt B Endoscopic band ligation C Large volume paracentesis D Transjugular intrahepatic portosystemic shunt

Correct Answer ( B ) Explanation: Esophageal varices are dilated submucosal veins which commonly occur in the lower third of the esophagus. They are mainly due to portal hypertension secondary to cirrhosis. They typically present with massive upper gastrointestinal bleeding. Emergency care is based on hemorrhage control, maintaining cardiopulmonary function and correcting concurrent cirrhosis-induced coagulopathies. Blood transfusions are often necessary. Therapeutic endoscopy, with variceal ligation, banding or sclerotherapy, is considered by many to be the definitive emergent treatment. Terlipressin, a vasopressin analogue, and octreotide, a somatostatin analogue, can also be used in the emergent care of bleeding esophageal varices. Refractory cases may require balloon tamponade.Transjugular intrahepatic portosystemic shunt (D) and distal splenorenal shunt (A) are two procedures used for treating underlying portal hypertension. They are not used in the emergent treatment of variceal bleeding, but reserved to treat such patients once they are stabilized. Large volume paracentesis

In a patient with chronic abdominal pain, which additional finding suggests a diagnosis of irritable bowel syndrome? A Decreased hemoglobin B Improvement with defecation C Nocturnal or progressive abdominal pain D Weight loss .

Correct Answer ( B ) Explanation: Irritable bowel syndrome (IBS) Symptoms include chronic abdominal pain described as crampy and varying in intensity, accompanied by altered bowel habits. Patients complain of diarrhea, constipation, both diarrhea, and constipation, or normal bowel movements that alternate with diarrhea and constipation. Symptoms usually improve with defecation. The Rome III criteria confirm a diagnosis of IBS in patients with recurrent abdominal pain at least three days per month for the past three months associated with at least two other criteria. These additional criteria include improvement with defecation, onset associated with a change in the frequency of stool and onset associated with a change in the appearance or form of stool. Management of IBS includes psychological support, dietary measures and adjunctive pharmacologic treatment aimed at reducing symptoms

Which of the following is recommended in the treatment of an acute anal fissure? A Antibiotics B Antifungal cream C Sitz bath D Surgery

Correct Answer ( C ) An anal fissure is a tear in the anal mucosa that can cause severe anal pain and rectal bleeding and results from forceful dilation of the anal canal, most commonly during defecation. Most occur posteriorly in the midline. the anus is exquisitely tender and may demonstrate internal anal sphincter spasm. Evaluation can occur with gentle spreading of the anus, but some require anoscopy to assess their extent. Most acute anal fissures can be managed conservatively with dietary fiber, prevention of constipation, adequate hydration, and stool softeners. Sitz baths may provide symptomatic improvement. Surgical treatment (D) is indicated in chronic anal fissures greater than 1 month but not in the treatment of acute anal fissures.

You examine a 17-year-old girl due to anorexia nervosa. She has low self-esteem with intense fear of gaining weight. She restricted her diet for the past three months. On physical exam, her BMI is 18 with heart rate of 35 beats per minute at rest. You decide to admit her for inpatient management. You plan to start nutritional therapy but worry about the risk of refeeding syndrome. Which laboratory finding is most consistent with refeeding syndrome? A Hyperkalemia B Hypermagnesemia C Hypophosphatemia D Thrombocytopenia

Correct Answer ( C ) Explanation: The core features of anorexia nervosa are restriction of energy intake, intense fear of gaining weight, and distorted perception of body weight.

A 48-year-old man presents with a 4-week history of rectal pain associated with minimal rectal bleeding. On examination there is a small tear of the anorectal mucosa at the 6 o'clock position. Which of the following is the most appropriate initial treatment? A Botulinum toxin B Nitroglycerin C Sitz baths, psyllium fiber, and bulking agents D Sphincterotomy

Correct Answer ( C ) Explanation: This patient has classic findings for acute rectal fissure. Typically, the patient reports severe pain during a bowel movement, with the pain lasting several minutes to hours afterward. The pain recurs with every bowel movement, and the patient commonly becomes afraid or unwilling to have a bowel movement, leading to a cycle of worsening constipation, harder stools, and more anal pain. patients note bright-red blood on the toilet paper or stool. A fissure is a small cut or split in the anoderm. IA sentinel tag or "pile" is frequently observed protruding from the anus

An obese man presents with a 1-month history of dyspepsia. He denies dysphagia, odynophagia, vomiting, or weight loss. His exam reveals no concerning findings. You start him on antacids and order a fecal occult blood test (FOBT). He returns 1-month later unchanged. His FOBT is negative. Which of the following would be most likely used to diagnose gastroesophageal reflux disease (GERD) in this patient? A Esophagogastroduodenoscopy B HIDA scan C High resolution manometry D Proton pump inhibitor trial

Correct Answer ( D )

An overweight 29-year-old roofer presents with acute groin swelling after lifting an 80-pound sack of shingles. Examination reveals a minimally tender mass in the right scrotum. There is no mass in the proximal thigh or abdomen. Which of the following types of hernia do you most likely suspect? A Direct inguinal B Epigastric C Femoral D Indirect inguinal

Correct Answer ( D ) Explanation: The inguinal canal is formed by the inguinal ligament and the lower abdominal muscles. It is a tube from the abdominal cavity into the scrotum, allowing the testicles a passageway to descend through. It has two openings called the deep (internal) and superficial (external) inguinal rings. Hernia is the general term used to describe the passage of tissue or organ from its cavity of origin into a different body cavity. Hernias are common in the inguinal region. A direct inguinal hernia is passage of the abdominal contents straight through a weakened portion of the abdominal wall, usually occurring medial to the inferior epigastric vessels and superior to the inguinal ligament. An indirect inguinal hernia is the passage of abdominal contents through the internal inguinal ring, through the inguinal canal, and out through the external inguinal ring into the scrotum. Indirect inguinal hernias are more common than the direct type. Some risk factors that predispose a patient to have an inguinal hernia are obesity, heavy lifting, coughing, straining, and chronic lung disease.

A 45-year-old woman presents for evaluation of right upper quadrant pain. The pain was initially intermittent but has become more constant without radiation. She does not have fever. Her examination is notable for tenderness in the right upper quadrant without a Murphy's sign. She undergoes a right upper quadrant ultrasound as seen above. Which of the following is the most likely diagnosis? A Cholangitis B Cholecystitis C Choledocholithiasis D Cholelithiasis

Correct Answer ( D ) Explanation: The ultrasound images display gallstones within the gallbladder consistent with cholelithiasis. This patient has biliary colic, and although the name includes colic, the pain may be a steady constant pain. The physical examination reveals tenderness in the right upper quadrant classically without rebound, guarding or a Murphy's sign.Cholangitis (A) is a clinical diagnosis made by the triad of right upper quadrant pain, fever and jaundice. A gallstone typically causes obstruction leading to bacterial overgrowth. Patients can become ill quickly requiring broad spectrum antibiotics and relief of the obstruction. The ultrasound should demonstrate intra and extrahepatic biliary dilation. Cholecystitis (B) is classically associated with a positive Murphy's sign on physical examination. On ultrasound images, typical findings include a thickened gallbladder wall and pericholecystic fluid. Choledocholithiasis (C) is the presence of a gallstone in the common bile duct. A proximal stone may be visualized on ultrasound although stones in the distal duct may be challenging to identify. Ultrasound may only reveal dilation of the common bile duct as a secondary indication of the stone.

A 34-year-old man, with no past medical history, presents to the emergency department complaining of burning epigastric pain radiating to the back for the last 3 months. The pain started a couple of hours after eating and occurs at night. The patient does not take any medications. Vitals are T 37°C, HR 82, BP 138/62 RR 18, and oxygen saturation 100% on room air. Physical examination is significant for brown, heme-positive stool. Once the diagnosis is confirmed, which of the following is the most appropriate to treat this condition? A Antacids B Aspirin C Carafate D Clarithromycin, amoxicillin, and a proton-pump inhibitor (PPI)

Correct Answer ( D ) Explanation: This patient presents with symptoms suggestive of peptic ulcer disease (PUD). The most common etiologic agent of PUD is infection with Helicobacter pylori and eradication of the bacteria leads to more rapid ulcer healing, prevents relapse and decreases the rate of complications. The second most common cause of PUD is the use of non-steroidal anti-inflammatory drugs (NSAIDs). Initial treatment of presumptive PUD (definitive diagnosis can only be made on endoscopy) includes lifestyle changes including smoking cessation, reduction or elimination of alcohol use, and stopping NSAID and aspirin use. the current recommendation is to test for H. pylori with antibody detection, urea breath test or fecal antigen tests. Once H.pylori is confirmed as the cause, triple therapy, which consists of clarithromycin, amoxicillin (or metronidazole ) and a PPI once a day for 10-14 days, should be initiated.

A 45-year-old man presents to the clinic with several weeks of dull, gnawing epigastric pain that is usually relieved by eating. He has been taking omeprazole for five weeks, but has had no improvement. His only other daily medication is ibuprofen for knee pain. An abdominal exam is normal and fecal occult blood testing is negative. The next best step in management includes which of the following diagnostic tests? A Abdominal CT B Barium upper gastrointestinal series C H. pylori fecal antigen assay D Upper endoscopy with biopsy

Correct Answer ( D ) Explanation: Upper Endoscopy is the next best step in management of this patient's epigastric pain because it will aid in visualization of a gastric or peptic ulcer and allow for biopsy if necessary. Based on the location and description of this patient's pain, it is likely that he has peptic ulcer disease (PUD).

Which of the following conditions is characterized by ataxia, oculomotor dysfunction, and altered mental status? A Korsakoff syndrome B Normal pressure hydrocephalus C Vitamin B12 deficiency D Wernicke encephalopathy

Correct Answer ( D ) Explanation: Wernicke encephalopathy is characterized by oculomotor dysfunction (most commonly nystagmus), cerebellar dysfunction, and altered mental status. The result of thiamine deficiency, it is most frequently seen in chronic alcoholics, but can also be seen in post-bariatric surgery patients

o Gastrointestinal/Nutritional: History and Physical, Crohn disease

Crohn's Dz (Inflammatory Bowel Dz) a. Can affect any part of Gl from mouth to rectum, but usually includes the intestines and often spares the rectum (in contrast to ulcerative colitis) b. Si/Sx = abd pain, diarrhea, malabsorption, fever, stricture causing obstruction, fistulae; c. Dx = colonoscopy with Bx of affected areas ~ transmural inflammation, noncaseating granulomas, cobblestone mucosa, skip lesions, creeping fat on gross dissection is pathognomonic d. Tx (1) First line for the colonic disease is aminosalicylate (e.g., mesalamine, sulfasalazine) (2) Steroids for acute exacerbation/breakthrough flairs (3) Immunotherapy (a) Azathioprine and mercaptopurine to try to prolong disease remission (b) Anti-tumor necrosis factor (TN F) therapy with infliximab, adalimumab, certolizumab, anti-TNF Abs

Pt 28 y/o with abdominal pain, diarrhea , n/v + perianal fistula and oral ulcers?

Crohn's disease Can affect any part of Gl from mouth to rectum, but usually includes the intestines and often spares the rectum (in contrast to ulcerative colitis) b. Si/Sx = abd pain, diarrhea, malabsorption, fever, stricture causing obstruction, fistulae; see below for extraintestinal manifestations Crohn's disease Ulcerations tend to be linear with transverse fissures in Crohn's disease. These skip lesions are common with Crohn's disease.

Gastrointestinal/Nutritional : Diagnostic Studies, Acute cholecystitis

Cystic duct obstruction + Murphy sign Constant( vs colicky) pain Fever, leukocytosis ● Ultrasound → initial test of choice (thickened gallbladder >3mm) ● HIDA scan → Gold standard ● Labs: leukocytosis with left shift, elevated bilirubin, elevated LFTs and alkaline phosphatase TX: Cholecystectomy

Which of the following treatments will most benefit the diabetic patient with two vessel coronary disease? A. Stent placement B. Percutaneous balloon angioplasty C. Medical management D. Coronary artery bypass graft

D. CABG is the treatment of choice in a diabetic with two or three vessel disease.

o Gastrointestinal/Nutritional: Diagnostic Studies,Boerhaave syndrome Boerhaave syndrome is a transmural perforation of the esophagus to be distinguished from Mallory-Weiss syndrome, a nontransmural esophageal tear also associated with vomiting. Boerhaave syndrome is reserved for the 10% of esophageal perforations which occur due to vomiting.treatment includes immediate antibiotic therapy to prevent mediastinitis and sepsis, surgical repair of the perforation,[10] and if there is significant fluid loss it should be replaced with IV fluid therapy since oral rehydration is not possible

DIAGNOSIS Boerhaave syndrome o Many patients present with a pleural effusion. Thoracentesis with examination of the pleural fluid can aid in diagnosis. Undigested food particles and gastric juices usually are found. o CXR The most common finding is a unilateral effusion, usually on the left. This corresponds with the fact that most perforations occur in the left posterior aspect of the esophagus. Other findings may include pneumothorax, hydropneumothorax, pneumomediastinum, subcutaneous emphysema, or mediastinal widening. o Esophagraphy helps to confirm the diagnosis. It typically shows extravasation of contrast into the pleural cavity. o CT can reveal decisive criteria for diagnosis, it is helpful in patients too ill to tolerate esophagrams, and it localizes collections of fluid for surgical drainage.

Urology/Renal : Diagnostic Studies, Nephrolithiasis

DX: ● *acute, severe flank pain urinary frequency/urgency hematuria groin pain testicular pain fever tachycardia hypotension costovertebral angle and ipsilateral flank tenderness* ● *UA: microscopic or gross hematuria, nitrites (if infectious)* ○ Obtain culture if infectious ●* Noncontrast CT scan of the abdomen/pelvis is the preferred imaging modality.* ● Renal Ultrasound → detects stones or complications (ex. hydronephrosis) ○ Used if CT is contraindicated

Gastrointestinal/Nutritional: Clinical Intervention, Cholecystitis Cholecystitis Cystic duct obstruction positive Murphy's sign Constant RUQ pain Fever leukocytosis Dx :RUQ US, HIDA scan Tx cholecystetomy

Definitive therapy Tx = laparoscopic cholecystectomy

o Pre-Operative/Post-Operative Care: Diagnosis, Atelectasis o The most common test used to diagnose atelectasis is a chest x ray. Atelectasis is the collapse or closure of a lung resulting in reduced or absent gas exchange. It may affect part or all of a lung. It is usually unilateral. It is a condition where the alveoli are deflated down to little or no volume, as distinct from pulmonary consolidation, in which they are filled with liquid. It is often called a collapsed lung, although that term may also refer to pneumothorax. It's a breathing (respiratory) complication after surgery. Atelectasis is also a possible complication of other respiratory problems, including cystic fibrosis, inhaled foreign objects, lung tumors, fluid in the lung, respiratory weakness and chest injuries.

Diagnosis Clinically significant atelectasis is generally visible on chest X-ray; findings can include lung opacification and/or loss of lung volume. Post-surgical atelectasis will be bibasal in pattern. Chest CT or Bronchoscopy may be necessary if the cause of atelectasis is not clinically apparent. Direct signs of atelectasis include displacement of interlobar fissures and mobile structures within the thorax, overinflation of the unaffected ipsilateral lobe or contralateral lung, and opacification of the collapsed lobe. Chest X-ray. A chest X-ray usually can diagnose atelectasis. CT scan. CT is more sensitive than plain X-ray in detecting atelectasis because it can measure lung volumes in all or part of a lung. Oximetry. Bronchoscopy. A flexible, lighted tube threaded down your throat

Pre-Operative/Post-Operative Care: Diagnosis, Delirium tremens Alcohol withdrawal has four phases- (1) Tremor-occurs (2) Seizure-occurs (3) Hallucinosis (4) DTs-at approximately 72 hrs after the last drink, the autonomic instability that defines DTs begins with dangerous tachycardia and hypertension and can be accompanied by each of the other three phases (tremor, seizure, hallucinosis}--the autonomic instability is also best Tx with benzodiazepines Delirium tremens (DTs) is a rapid onset of confusion usually caused by withdrawal from alcohol. When it occurs, it is often three days into the withdrawal symptoms and lasts for two to three days. Physical effects may include shaking, shivering, irregular heart rate, and sweating. People may also see or hear things other people do not. Occasionally, a very high body temperature or seizures may result in death. Alcohol is one of the most dangerous drugs from which to withdraw. o Basic laboratory investigations include a complete blood count, liver function tests, a urine drug screen, and determination of blood alcohol and electrolyte levels.

Diagnosis is mainly based on physical symptoms, effects may include tremors, shaking, shivering, irregular heart rate, fever, hallucination, sweating, and seizures with delirium tremens it is important to rule out other associated problems such as electrolyte abnormalities, pancreatitis, and alcoholic hepatitis.Basic laboratory investigations include a complete blood count, liver function tests, a urine drug screen, and determination of blood alcohol and electrolyte levels.

Gastrointestinal/Nutritional : Diagnosis, Celiac disease

Diagnosis, Celiac disease Si/Sx: male patient presents with complaints of diarrhea with marked flatulence and weight loss for the past 6-8 months. In addition, he states that his stools are loose and soft with an oily appearance and foul smelling.Steatorrhea is usually present in mild disease of celiac sprue due to fat malabsorption. ● Small Bowel Biopsy: definitive dx ● stool for 72-hour fecal fat ● (+) anti-endomysial antibodies IgA Ab and transglutaminase AB

Hematology: Diagnosis, Disseminated intravascular coagulation Disseminated intravascular coagulation is characterized by bleeding from many sites as all coagulation factors are consumed and then broken down, leading to decreased fibrinogen level and platelet count, prolonged PT and PTT, and presence of fibrin split products. Disseminated intravascular coagulation (DIC) is an acquired syndrome characterized by activation of coagulation pathways, resulting in the formation of intravascular thrombi and depletion of platelets and coagulation factors. Thrombi may lead to vascular obstruction/ischemia and multiorgan failure. Spontaneous bleeding may occur. Generalized bleeding, evidenced by at least 3 unrelated sites, is highly suggestive of DIC. DIC can be triggered by major trauma, organ destruction, sepsis or severe infection, severe obstetric disorders, some malignancies, major vascular disorders, and severe toxic or immunologic reactions.

Diagnosis: The patient is now 6 hours postoperative and has blood oozing from the suture line and IV sites. There is bloody urine in the Foley bag. Laboratory evaluation demonstrates a platelet count of 10,000/microliter, prolonged prothrombin level, and the presence of fibrin split products. ○ Increased thrombin formation: decreased fibrinogen, increased PTT/PT/INR, severe thrombocytopenia ○ Peripheral smear → fragmented RBCs (schistocytes), platelet count decreased prothrombin time (PT) often prolonged fibrinogen decreased, D-dimer/fibrin degradation products elevated

Cardiovascular: Diagnosis, Intermittent Claudication

Diagnostic Tests 1st Tests To Order ankle brachial index (ABI) duplex ultrasound

Peripheral vascular (artery) Disease (PVD) clinical intervention

Diagnostic Tests 1st Tests To Order ankle brachial index (ABI) duplex ultrasoundLifestyle modifications include smoking cessation and increasing moderate exercise Lifestyle modifications include smoking cessation and increasing moderate exerciseb. Pharmacotherapy is Cilostazol (vasodilator and platelet inhibitory properties) or second-line pentoxifylline c. Minimally invasive therapy includes percutaneous balloon angioplasty (PTA) and/or atherectomy-best results f d. Treatment of iliac disease now involves PTA plus the placement of endoluminal stents e. Indications for surgical intervention are severe rest pain, tissue necrosis, nonhealing infxn, and intractable claudication f. Surgical treatment includes local endarterectomy with or without patch angioplasty and bypass procedures

o Gastrointestinal/Nutritional: Scientific Concepts, Inguinal hernia

Direct Inguinal Hernia: Hesselbach's Triangle: Medial- Rectus Abdominis Muscle, Inferior - Inguinal ligament, Lateral - Inferior epigastric vessels.A direct inguinal hernia is passage of the abdominal contents straight through a weakened portion of the abdominal wall, usually occurring medial to the inferior epigastric vessels and superior to the inguinal ligament. An indirect inguinal hernia is the passage of abdominal contents through the internal inguinal ring, through the inguinal canal, and out through the external inguinal ring into the scrotum. Indirect inguinal hernias are more common than the direct type. An indirect inguinal hernia is typically elliptic that does not reduce easily.

Pre-Operative/Post-Operative Care : Diagnosis, Pneumonia

Dx (1) Dx = CXR shows infiltrates pneumonia in the presence of appropriate Si/Sx (2) Sputum Gram stain and culture demonstrates causative organism

Pre-Operative/Post-Operative Care: Diagnostic Studies, Chronic renal disease The first diagnostic tests to order are a serum creatinine and GFR, urine microalbumin, urinalysis to assess for hematuria and proteinuria, and a renal ultrasound to evaluate for kidney size, mass lesions, urinary tract obstruction, and, with a duplex examination, renal arterial blood flow. A urine microalbumin would be indicated in patients with diabetes and CKD if there was no evidence of proteinuria on urine dipstick.

Dx 1st Tests To Order serum creatinine elevated: >1.1mg/dL urinalysis hematuria and/or proteinuria urine microalbumin microalbuminuria (30 to 300 mg/day) renal US small kidney size; the presence of obstruction/hydronephrosis; kidney stones estimation of GFR <60 anemia from chronic lack of EPO, diffuse osteopenia

Gastrointestinal/Nutritional : Clinical Intervention, Esophageal varices

Dx esophagogastroduodenoscopy (EGD) dilated veins in lower esophagus CBC microcytic anemia and/or thrombocytopenia Tx Medications Beta-blocker, Synthetic hormones octrectide, Antibiotics, and Blood transfusion Medical procedure Rubber band ligation, Embolization, Sclerotherapy, Transjugular intrahepatic portosystemic shunt, and Balloon tamponade Supportive care IV fluids

Neurology/Neurosurgery : Diagnosis, Cerebrovascular accident

Dx = CT if positive, Dx is made Rule out hemorrhagia; if negative, still need MRI to rule out) **MRI is gold standard* ECG may exclude arrhythmia or ischemia Rule out Sz ~ EEG, , loss of bowel/bladder control and tongue injury

Gastrointestinal/Nutritional: Diagnosis, Choledocholithiasis Common bile duct obstruction Proximal inflammation Obstructive jaundice Dilated hepatic bile ducts Tx: ERCP • SifSx = RUQ pain worse with fatty meals, jaundice 1 . Passage of stone through the cystic duct, can obstruct CBD 2. Si/Sx = obstructive jaundice, ↑conjugated bilirubin, hypercholesterolemia, ↑ alkaline phosphatase Dx = Utz ~ CBD >9-mm diameter (Utz first line for Dx) • Labs~ ↑LFTs, ↑ bilirubin

Dx = Utz ~ CBD >9-mm diameter (Utz first line for Dx) • Labs~ ↑LFTs, ↑ bilirubin Tx = laparoscopic cholecystectomy, or ERCP is performed to remove gallstones that have become lodged in the common bile duct. It is not an initial study that is performed.

Hematology: Diagnostic Studies, Iron deficiency anemia Si/Sx = tachycardia, fatigue, pallor all from anemia, smooth tongue, brittle nails, esophageal webs, and term "pica" to describe craving and chewing substances such as ice all from iron deficiency Dx = .↓ serum iron, ↓serum ferritin, ↑ total iron-binding capacity (TIBQ), The peripheral smear will demonstrate Hypochromic, microcytic red blood cells and few target cells. (5) Tx = iron sulfate; should achieve baseline hematocrit within 2 mos

Dx = ↓ serum iron, ↓serum ferritin, ↑ total iron-binding capacity (TIBQ), The peripheral smear will demonstrate Hypochromic, microcytic red blood cells and few target cells. o Iron Panel: Low Iron, Low Ferritin, High TIBC o Si/Sx = tachycardia, fatigue, pallor all from anemia, smooth tongue, brittle nails, esophageal webs, and pica all from iron deficiency

Gastrointestinal/Nutritional: Diagnostic Studies, Zollinger Ellison syndrome Zollinger-EIIison syndrome (Gastrinoma) The signs and symptoms are due to the hypersecretion of gastrin. The MC presenting symptoms are abdominal pain , diarrhea , and gastroesophageal reflux disease (GERD) Steatorrhea may also be a presenting feature due to the inactivation of pancreatic enzymes by excess acid. Patients presenting with peptic ulcer disease refractory to medical therapy, associated with diarrhea and/or GI bleeding in the absence of nonsteroidal anti-inflammatory drug use or Helicobacter pylori infection, should all be evaluated for ZES. a. Dx =clinically, fasting serum gastrin. measurement of the levels of gastrin in blood serum after the patient has fasted for 12 hours, so that presence of food is not a factor. It is markedly increased in certain conditions such as Zollinger-Ellison syndrome and G cell hyperplasia. +secretin test. b. Tx = surgical resection of the tumor and aggressive PPI therapy.

Dx =clinically, fasting serum gastrin. measurement of the levels of gastrin in blood serum after the patient has fasted for 12 hours. It is markedly increased in certain conditions such as Zollinger-Ellison syndrome +secretin test. stimulation test Initial measurement of gastrin levels, an injection of secretin, post measurement of gastrin levels. If you have Zollinger-Ellison, your gastrin levels will increase even more. Serum gastrin levels can be elevated for many reasons; however significant elevations in a fasting state as well as with the secretin stimulation test are confirmatory for Zollinger-Ellison syndrome which is highly suspect in this case

Endocrinology: Diagnosis, Pheochromocytoma Pheochromocytoma (1) Si/Sx = HTN (episodic or chronic), diaphoresis, palpitations, tachycardia, headache, nausea/vomit, flushing, dyspnea, diarrhea (3) Dx =↑ urinary catecholamines, CT scan of the adrenal showing neoplasm (4) Tx (a) Surgical excision after preoperative administration of a-blockers (b) Ca2 + channel blockers for hypertensive crisis (c) Phenoxybenzamine or phentolamine (a-blockers) for inoperable dz

Dx =↑ urinary catecholamines, CT scan of the adrenal showing neoplasm

83. 3. DIAGNOSIS 52-year-old female presents with diffuse abdominal pain accompanied by distention and visible peristalsis. Auscultation reveals hyperactive bowel sounds. Percussion is tympanic throughout. Palpation reveals mild diffuse tenderness without masses. The most likely diagnosis is A. intra-abdominal abscess. B. intestinal obstruction. C. paralytic ileus. D. cholecystitis.

EXPLANATIONS: (c) B. Intestinal obstruction without complications is suggested by crampy pain, abdominal distention, hyperactive bowel sounds, visible peristalsis, and minimal tenderness.

31. 2. DIAG STUDIES The serum creatine phosphokinase-mB (CPK-MB) rises to a peak after an acute myocardial infarction after how many hours? A. 4 - 6 B. 8 - 12 C. 18 - 20 D. 48 - 72

EXPLANATIONS: (c) C. CPK-MB is found mainly in cardiac muscle. It begins to rise in 4 to 6 hours, peaks at 18 hours, and returns to normal in 48 hours.

DIAGNOSIS Marked hypersecretion, gastric hyperacidity, and persistent ulcers are associated with which of the following? A. Zollinger-Ellison syndrome B. Peutz-Jeghers syndrome C. hyperthyroidism D. hypercalcemia

EXPLANATIONS: (c) A. Gastrin-producing islet cell tumors stimulate hypersecretion of gastric acid with resultant severe peptic ulcers as indicated in Zollinger-Ellison syndrome.

75. 3. DIAGNOSIS A 58-year-old chemotherapy patient presents with fever, chills, productive cough, and disorientation. The patient appears very ill. Vital signs include: T 102° F P 138/min R 24/min BP 70/40 mm Hg Laboratory results include: WBC 2.1 x 103/mm3 Na 140 mEq/L CI 90 mEq/L HCO3 - 15 mEq/L Glucose 140 mg/dL Besides sepsis, the most likely diagnosis is A. lactic acidosis. B. hyperglycemia. C. hyperchloremia. D. blast crisis.

EXPLANATIONS: (c) A. Lactic acidosis is the most common cause of anion gap acidosis. In a patient with inadequate tissue perfusion, lactic acid builds up due to anaerobic metabolism, leading to accumulation of an "unmeasured ion."

DIAGNOSIS Chest x-ray of a 63-year-old smoker reveals a hilar mass, mediastinal widening, and cavitation. Which of the following is the most likely diagnosis? A. squamous cell carcinoma B. adenocarcinoma C. mesothelioma D. large cell tumor

EXPLANATIONS: (c) A. Squamous cell carcinoma commonly produces a hilar mass, mediastinal widening, and cavitation

DIAG STUDIES A 35-year-old male presents to the emergency department with a 4-hour history of abdominal pain, nausea, and vomiting. He denies diarrhea. Examination reveals the following: Temperature 101° F (38.3° C) Pulse 100 beats/min Respiratory rate 20/min Blood pressure 110/65 mm Hg The patient is in moderate distress and slightly diaphoretic. He has poor oral hygiene and the odor of alcohol is noted. Examination of the heart and lungs are unremarkable except for tachycardia. Abdominal examination reveals hypoactive bowel sounds, diffuse tenderness with more marked tenderness in the epigastric area, without guarding, rebound, masses, or organomegaly. Along with CBC, which of the following diagnostic studies would be most appropriate? A. liver function tests, amylase, and abdominal films B. ESR, urinalysis, and electrolytes C. electrolytes, ECG, and upper GI D. ECG, barium enema, and sonogram

EXPLANATIONS: (c) A. The clinical presentation suggests acute pancreatitis, which is best evaluated by liver function tests, amylase, and abdominal films.

Which of the following electrocardiographic findings is the hallmark of pericarditis? A. ST elevation B. prolonged Q-T interval C. atrial fibrillation D. tall peaked T waves

EXPLANATIONS: (c) A. The hallmark of pericarditis is ST segment elevation throughout the precordium along with PR segment depression.

A 72-year-old male presents to the clinic for evaluation of a bump on his nose. The patient is a lifelong resident of Florida and an avid golfer. On examination, a 1 cm round, firm, pearly nodule with fine telangiectasias is noted. Which of the following is the most likely diagnosis? A. basal cell carcinoma B. squamous cell carcinoma C. seborrheic keratosis D. actinic keratosis

EXPLANATIONS: (c) A. The lesion of basal cell carcinoma is typically firm, round, and pearly or waxy. It is the most common cutaneous neoplasm in humans, with 85% of them occurring on the head or neck. Margin telangiectasis is classically associated with basal cell carcinomas. Risk factors include fair skin, excessive sun exposure, and male gender.

Post-infarction syndrome (Dressler's syndrome) occurs after acute myocardial infarction presenting as A. ventricular aneurysm. B. pericarditis and pleuritis. C. cardiac tamponade. D. pleural effusion and rash.

EXPLANATIONS: (c) B. Dressler's syndrome is the occurrence of pericarditis and pleuritis several days to weeks following an MI.

Which of the following is the pathophysiologic process of a transudative pleural effusion? A. increased fluid production due to increased hydrostatic pressure B. decreased lymphatic clearance of fluid from the pleural space C. infection in the pleural space D. bleeding into the pleural space

EXPLANATIONS: (c) A. A transudative pleural effusion occurs in the setting of normal capillary integrity and suggests the absence of local pleural disease. Chronic heart failure accounts for 90% of transudates. Hypoalbuminemia, cirrhosis, and acute atelectasis are also causes of a transudate.

Which of the following is the selected method for the prevention of venous thromboembolism in a 38-year-old male undergoing an inguinal hernia repair? A. early ambulation B. elastic stockings C. intermittent pneumatic compression D. low-molecular weight heparin

EXPLANATIONS: (c) A. Early ambulation is recommended for prophylaxis of venous thromboembolism in low-risk, minor procedures when the patient is under 40 years of age and there are no clinical risk factors.

Which of the following is the selected method for the prevention of venous thromboembolism in a 38-year-old male undergoing an inguinal hernia repair? A. early ambulation B. elastic stockings C. intermittent pneumatic compression D. low-molecular weight heparin

EXPLANATIONS: (c) A. Early ambulation is recommended for prophylaxis of venous thromboembolism in low-risk, minor procedures when the patient is under 40 years of age and there are no clinical risk factors. (u) B. Elastic stockings are indicated for patients at moderate risk of venous thromboembolism in ages 40-60 with minor procedures with additional thrombosis risk factor, or major operations for patients under age 40 without additional clinical risk factors. (u) C. Intermittent pneumatic compression is indicated in patients undergoing a major operation plus an increased risk of bleeding. (u) D. Low molecular weight heparin is indicated in patients undergoing orthopedic surgery, neurosurgery, or trauma with an identifiable risk factor for thromboembolism

The most effective preventive strategy to prevent recurrence of renal lithiasis is which of the following? A. increase in hydration B. early treatment of urinary tract infection C. limitation of calcium intake D. use of probenecid

EXPLANATIONS: (c) A. Keeping the urine dilute is the most effective strategy to prevent crystal accumulation in the urine and the development of urinary stones.

Which of the following conditions is most suggestive of an asymptomatic abdominal aortic aneurysm? A. abdominal mass B. hypertension C. chest pain D. syncope

EXPLANATIONS: (c) A. Symptomatic abdominal aortic aneurysm presents with pulsating upper abdominal mass.

Post-infarction syndrome (Dressler's syndrome) occurs after acute myocardial infarction presenting as A. ventricular aneurysm. B. pericarditis and pleuritis. C. cardiac tamponade. D. pleural effusion and rash.

EXPLANATIONS: (c) B. Dressler's syndrome is the occurrence of pericarditis and pleuritis several days to weeks following an MI.

A 20-year-old male presents with a mass in the groin. On inspection with the patient standing a symmetric, round swelling is noted at the external ring. When the patient lies down the mass disappears. The patient denies any trauma. The most likely diagnosis is A. an indirect inguinal hernia. B. a direct inguinal hernia. C. an obturator hernia. D. a femoral hernia.

EXPLANATIONS: (u) A. An indirect inguinal hernia is typically elliptic that does not reduce easily. (c) B. A direct inguinal hernia is symmetrical, round and disappears easily with the patient lying down.

134. 5. CLIN INTERVENTION The treatment of choice for acute arterial embolism of an extremity is A. intravenous heparin. B. embolectomy. C. amputation. D. aspirin.

EXPLANATIONS: Ref 23 (c) B. Embolectomy within 4 to 6 hours is the treatment of choice

Cardiovascular: Diagnostic Studies, Ventricular arrhythmia

Electrocardiogram (ECG) An electrocardiogram, also called an ECG or EKG, is the most common tool used to diagnose ventricular tachycardia. Holter monitor. This portable ECG device is carried in your pocket or worn on a belt or shoulder strap Medications. Anti-arrhythmic medications amiodarone , lidocaine Implantable cardioverter-defibrillator. Surgery. Open-heart surgery intravenous antiarrhythmic medications amiodarone lidocaine 2nd synchronized cardioversion chronic implantable cardioverter defibrillator (ICD)

Gastrointestinal/Nutritional : Clinical Intervention, Thrombosed hemorrhoid

Elliptical excision of the thrombosed Hemorrhoid / Hemorrhoidectomy

Urology/Renal: History and Physical, Epididymitis

Epididymitis a. Si/Sx = unilateral testicular pain, dysuria, occasional urethral discharge, fever, leukocytosis in severe cases, painful and swollen epididymis b. Dx = history and physical, labs ~ UA can be negative or show pyuria, urine Cx should be obtained, swab for Neisseria gonorrhoeae and Chlamydia c. Tx =antibiotics and NSAIDs

pt CT scans epidural hematoma epidural hematoma Dx = CT ~ biconcave disk not crossing sutures Middle meningeal artery This is a medical emergency! Tx Evacuate hematoma via burr holes

Epidural Hematoma CT Findings Convex, lens-shaped hyperdensity that may cross the midline not crossing sutures Epidural Hematoma Epidural hematomas occur when an artery is injured and arterial blood accumulates between the dura and the calvarium. Do not cross suture lines because of the tight adherence of the dura to the calvarium and thus have a biconvex or elliptical appearance. The middle meningeal artery is classically involved, especially with a skull fracture. Subdural Hematoma CT Findings Crescent-shaped, concave hyperdensity that does not cross the midline. Cross sutures lines Midline shift and compression of lateral ventricle may also be present Tearing of bridging veins during rapid or sudden changes in velocity thereby causing an accumulation of venous blood below the dura but above the arachnoid membrane (i.e., the "subural space"). Cross suture lines since bleeding is below the dura, which is tightly attached to the calvarium, thus giving the "crescent shape" appearance on head CT. Can result in mass effect leading to uncal and/or tonsillar herniation if left untreated. Occur more frequently in elderly patients due to reduced brain volume and "stretched" bridging veins.Associated with a "lucid interval", which means that a patient can be conscious and appear "normal" right after an injury, but as the blood accumulates the headache will worsen and mental status will decline as the intracranial pressure rises.

52 year-old male with history of hypertension and hyperlipidemia presents with an acute myocardial infarction. Urgent cardiac catheterization is performed and shows a 90% occlusion of the left anterior descending artery. The other arteries have minimal disease. Ejection fraction is 45%. Which of the following is the treatment of choice in this patient? A. Coronary artery bypass grafting (CABG) B. Streptokinase C. Percutaneous coronary intervention (PCI) D. Warfarin (Coumadin)

Explanation c) C. Immediate coronary angiography and primary percutaneous coronary intervention has been shown to be superior to thrombolysis. Percutaneous coronary intervention is a better, less invasive alternative to CABG for single vessel coronary artery disease.

Which subset of the asymptomatic general population should undergo one-time screening for abdominal aortic aneurysm? A. 65 year-old female with chronic kidney disease B. 65 year-old healthy male who has smoked since age 20 C. 74 year-old male who has diabetes mellitus D. 74 year-old female who has smoked since age 15

Explanations (c) B. Data support the use of abdominal ultrasound to screen 65- to 74-year old men, but not women, who have a history of smoking. Repeated screening does not appear to be needed if the aorta shows no enlargement.

135. Clinical Intervention/Cardiology An unresponsive patient is brought to the ED by ambulance. He is in ventricular tachycardia with a heart rate of 210 beats/min and a blood pressure of 70/40 mmHg. The first step in treatment is to Answers A. administer IV adenosine. B. DC cardiovert. C. administer IV lidocaine. D. apply overdrive pacer.

Explanations (c) B. The first step in treatment of unstable ventricular tachycardia with a pulse is to cardiovert using a 100 J countershock. Adenosine is used to treat PSVT.

148. Diagnosis/Endocrinology A 43 year-old male is found to have an elevated serum calcium on routine pre-employment laboratory testing. Further laboratory testing demonstrated: Calcium 11.3 mg/dL (8.8 - 10.2 mg/dL) Ionized Calcium 6.2 mg/dL (4.6 - 5.3 mg/dL) Phosphorus 2.1 mg/dL (3.0 - 4.5 mg/dL). What is the most likely diagnosis? A. Adrenal insufficiency B. Hyperparathyroidism C. Osteoporosis D. Paget's disease

Explanations (c) B. The presence of elevated total and ionized calcium are consistent with hyperparathyroidism.

A 25 year-old female presents with a three-day history of chest pain aggravated by. coughing and relieved by sitting. She is febrile and a CBC with differential reveals leukocytosis. Which of the following physical exam signs is characteristic of her problem? Answers A. Pulsus paradoxus B. Localized crackles C. Pericardial friction rub D. Wheezing

Explanations (c) C. Pericardial friction rub is characteristic of an inflammatory pericarditis.

38. Diagnostic Studies/Gastrointestinal/Nutritional Which of the following laboratory tests, if positive, would be most indicative of Crohn's disease? A. Antineutrophil cytoplasmic antibodies (ANCA) B. Antiendomysial antibodies (AEA) C. Antinuclear antibodies (ANA) D. Anti-Saccharomyces cerevisiae antibodies (ASCA)

Explanations (c) D. About 60-70% of patients with Crohn's disease are positive for Anti-Saccharomyces cerevisiae antibodies (ASCA) which are directed at the cell walls of S. cerevisiae while only 10-15% of ulcerative colitis is found to be positive.

222. History & Physical/Cardiology Which of the following is an expected finding in a patient with a diagnosis of an arterial embolism? A. lower extremity edema B. stasis dermatitis C. palpable cord D. pulselessness

Explanations (c) D. Pulselessness is a sign of acute ischemia secondary to arterial embolism

140. Diagnosis/Pulmonology A 28 year-old man presents to the emergency department complaining of sudden onset of shortness of breath associated with sharp right-sided chest pain increased with breathing. On physical examination, respirations are 20 per minute and blood pressure is 120/76 mm Hg. Auscultation of the chest reveals absent breath sounds over the right apex with normal heart sounds. Percussion of the right apex is noted to be hyperresonant. Which of the following is the most likely diagnosis? A. Hemothorax B. Pneumothorax C. Pulmonary embolus D. Foreign body aspiration

Explanations (c) B. This patient most likely has a spontaneous pneumothorax which is supported by the presenting symptoms of sudden onset of dyspnea and pleuritic chest pain as well as the physical exam findings of absent breath sounds and hyperresonance to percussion.

93. Clinical Therapeutics/Cardiology A 48 year-old male patient presents with a recent onset of anterior chest pain. The pain increases with deep breathing and coughing. He says he slept in the recliner last night as his discomfort worsens when he lies down. He also complains of feeling feverish although did not have a thermometer to check his temperature. An electrocardiogram (ECG) shows ST elevations in the precordial and limb leads. Additionally you note PR depression predominantly over leads I, II and III. Past medical history is significant for pharyngitis diagnosed as mononucleosis about 10 days ago. What is the most appropriate next step in the evaluation of this patient? A. Streptokinase B. Pericardiocentesis C. Indomethacin D. Cardiac catheterization

Explanations (c) C. Acute inflammatory pericarditis presents with anterior pleuritic chest pain that is worse supine than upright. ECG reveals diffuse ST segment elevation with associated PR depression. Viral infections (including EpsteinBarr) are the most common cause of acute pericarditis. Treatment for viral pericarditis is generally symptomatic. ASA or other NSAIDs are usually effective.

Which of the following is the most common cause of arterial embolization? A. Rheumatic heart disease B. Myxoma C. Atrial fibrillation D. Venous thrombosis

Explanations (c) C. Atrial fibrillation is present in 60-70% of patients with arterial emboli and is associated with left atrial appendage thrombus

170. Scientific Concepts/Endocrinology A 72 year-old male presents complaining of weakness, fatigue, anorexia, nausea and weight loss. On PE his BP is 94/52 mm Mercury and he has hyperpigmentation of his skin folds and mucous membranes. Which of the following is responsible for the patient's skin hyperpigmentation? A. Vitamin D deficiency B. Iron deficiency C. Excess ACTH secretion D. Excess thyroid hormone secretion

Explanations (c) C. The patient described most likely suffers from Addison's disease. The hyperpigmentation of primary adrenocortical insufficiency is due to ACTH having intrinsic MSH (melanocyte-stimulating hormone) activity.

180. History & Physical/Cardiology A 28 year-old patient presents with complaint of chest pain for two days. The patient describes the pain as constant and sharp. It is worse with lying down, better with sitting up and leaning forward. Vital signs are BP 120/80, HR 80, regular, RR 14 and Temperature 100.1 degrees F. Which of the following would you expect to find on physical examination? A. lower extremity edema B. carotid bruit C. pericardial friction rub D. splinter hemorrhages

Explanations (c) C. This patient has signs and symptoms of pericarditis. A pericardial friction rub is characteristic of acute pericarditis.

Several weeks following an acute MI, a patient presents with pericarditis, pleuritis, myalgias, fever, arthralgias, leukocytosis, and increased erythrocyte sedimentation rate. Which of the following is the most likely diagnosis? A. Levine syndrome B. Tietze's syndrome C. Leriche syndrome D. Dressler syndrome

Explanations (c) D. Dressler Syndrome occurs post MI as a type of post-inflammatory pericarditis.

A 76 year-old male presents after returning from a Safari in Africa. Seven days ago he experienced chest pressure lasting one hour that did not respond to three sublingual nitroglycerin tablets. There was no ability to have lab work or an EKG. The pain has not returned. If the patient had a non-STEMI myocardial infarction, which of the following studies will still be positive? A. Electrocardiogram B. Myoglobulin C. CK-MB index D. Troponin I

Explanations (c) D. Troponin I levels will stay positive for at least one week following myocardial infarction and is the preferred enzyme to measure in this setting.(u) CK-MB index has improved sensitivity for myocardial muscle damage that occurs with acute myocardial infarction but it returns to baseline within 2-3 days after injury

37. History & Physical/Gastrointestinal/Nutritional A 68-year-old male presents with jaundice, weight loss, and boring abdominal pain which radiates to the back. The gallbladder is palpable on physical examination. This finding is most consistent with which of the following? A. Pancreatic Tumor B. Hemorrhagic pancreatitis C. Cholecystitis D. Cholelithiasis

Explanations (c) A. A large palpable gallbladder resulting from pressure from a tumor in the pancreatic head is known as Courvoisier's sign.

171. Scientific Concepts/Cardiology Which of the following is the most common complication that occurs in the setting of acute pericarditis? A. Pericardial effusion B. Left ventricular failure C. Superior vena cava syndrome D. Subclavian steal syndrome

Explanations (c) A. Accumulation of transudate, exudate or blood in the pericardial sac can occur due to pericardial inflammation.

203. Diagnosis/Cardiology A 70-year-old female with a history of hypertension, diabetes, and hypothyroidism presents with a complaint of sudden onset of left lower extremity pain. Examination reveals a cool left lower extremity with a mottled appearance. Dorsalis pedis and posterior tibialis pulses are absent. Which of the following is the most likely diagnosis? A. Acute arterial occlusion B. Thromboangiitis obliterans C. Deep vein thrombosis D. Peripheral neuropathy

Explanations (c) A. Acute arterial occlusion presents with sudden onset of extremity pain, with absent or diminished pulses. The extremity will be cool to the touch and have a mottled appearance.

Diagnosis/Gastrointestinal/Nutritional A 20-year-old male presents with a mass in the groin. On examination with the patient standing, a mass is noted that extends into the scrotum. The patient denies any trauma. The most likely diagnosis is A. an indirect inguinal hernia. B. a direct inguinal hernia. C. an obturator hernia. D. a femoral hernia.

Explanations (c) A. An indirect inguinal hernia is caused by a patent processus vaginalis and the hernial contents may be felt in the ipsilateral scrotum.

68. Clinical Therapeutics/Cardiology A 76-year-old active female with a history of hypertension and hypothyroidism presents with complaints of palpitations and dyspnea on exertion. On examination, vital signs are BP 120/80 mmHg, HR 76 bpm, irregular, RR 16. Heart examination reveals an irregularly, irregular rhythm without murmur. Lungs are clear to auscultation and extremities are without edema. Which of the following is the most important medication to initiate for chronic therapy in this patient? A. Warfarin (Coumadin) B. Verapamil (Calan) C. Amiodarone (Cordarone) D. Digoxin (Lanoxin)

Explanations (c) A. Anticoagulation is necessary in all patients with atrial fibrillation to prevent thromboembolic events unless there is a contraindication.

History & Physical/Psychiatry/Behavioral Medicine Early clues to impending delirium tremens include A. agitation and decreased cognition. B. visual hallucinations and diaphoresis. C. autonomic hyperactivity and dehydration. D. mental confusion and sensory hyperacuity.

Explanations (c) A. Anxiety, decreased cognition, tremulousness, increasing irritability, and hyperactivity are common early clues to impending delirium tremens.

225. Diagnosis/Cardiology A 24-year-old male comes to the clinic with a one-week history of pain and swelling that involves the entire right upper extremity. He exercises frequently and has noticed the pain worsening while lifting weights. Examination shows enlarged cutaneous veins over the right anterior chest wall with a palpable cord. His right hand appears dusky. Which of the following is the most likely diagnosis? A. Axillary-subclavian venous thrombosis B. Thromboangiitis obliterans C. Superficial thrombophlebitis of the cephalic vein D. Brachial artery occlusion

Explanations (c) A. Axillary-subclavian venous thrombosis can occur in someone who strenuously exercises, has had a central venous catheter or history of venous thrombosis.

128. Clinical Therapeutics/Cardiology A 16-year-old male presents with a complaint of syncope after basketball practice today. Physical examination reveals a systolic murmur along the left sternal border that increases with Valsalva maneuver. An electrocardiogram reveals left ventricular hypertrophy. Echocardiogram shows asymmetric left ventricular hypertrophy with a hypercontractile left ventricle. Which of the following is the initial medication of choice in this patient? A. Metoprolol (Lopressor) B. Losartan (Cozaar) C. Lisinopril (Zestril) D. Hydrochlorothiazide (Diuril)

Explanations (c) A. Beta-blockers are the initial drug of choice in a symptomatic patient with hypertrophic cardiomyopathy.

107. Diagnosis/Urology/Renal A 65 year-old male with a 60 pack-year smoking history presents with painless hematuria for two days. He also complains of frequency and dysuria. He denies a history of recent upper respiratory tract infection. Which of the following is the most likely diagnosis? Answers A. Bladder cancer B. Wegener's granulomatosis C. IgA nephropathy D. Benign prostatic hypertrophy

Explanations (c) A. Bladder cancer is associated with smoking and presents with painless hematuria.

History & Physical/Urology/Renal Which of the following is most frequently associated with bladder cancer? A. Hematuria B. Dysuria C. Urgency D. Frequency

Explanations (c) A. Bladder cancer usually presents with painless hematuria.

Diagnosis/Gastrointestinal/Nutritional A 30-year-old patient presents with weight loss, diarrhea, and steatorrhea. Labs reveal that the anti-endomysial antibody (AEA) is positive. What is the most likely diagnosis? A. Celiac sprue B. Ulcerative colitis C. Whipple's disease D. Zollinger-Ellison syndrome

Explanations (c) A. Celiac sprue is not only characterized by these classic symptoms. The antiendomysial antibody has a 90-95%sensitivity and 90-95% specificity for celiac sprue.

165. Diagnosis/Dermatology A 32-year-old male with past history of type 2 diabetes mellitus presents with complaints of pain and increasing redness and swelling of his left lower leg associated with fever and chills. Examination reveals a temperature of 101 degrees F with local tenderness, erythema with indistinct flat borders and moderate edema involving the left calf. Inguinal lymph nodes are non-palpable and there is no evidence of lymphatic streaking. Which of the following is the most likely diagnosis? A. Cellulitis B. Erysipelas C. Necrotizing fasciitis D. Mucocutaneous candidiasis

Explanations (c) A. Cellulitis is characterized by an expanding, erythematous, edematous and tender lesion often involving the lower extremity. The borders of the area involved are not elevated and not demarcated. Diabetes mellitus is one of the risk factors for the development of this disease.

Clinical Therapeutics/Hematology A 21-year-old male with a diagnosis of type 1 von Willebrand disease undergoes dental extraction of his wisdom teeth. The patient comes to the clinic with continued oozing of the dental sockets despite packing. Treatment should begin with which of the following? A. DDAVP B. Factor VIII C. vWF concentrate D. FFP

Explanations (c) A. DDAVP (Desmopressin = Clotting promoter and antidiuretic) causes the release of vWF and factor VIII from storage sites significantly which is needed to complete hemostasis. Factor VIII is indicated for patients with Hemophilia A. Fresh frozen plasma is indicated in Coumadin overdosage and vWF concentrate is indicated in type 2 and 3 patients with von Willebrand disease.

Diagnosis/Gastrointestinal/Nutritional A 68-year-old male with a history of alcohol abuse presents with coffee-ground emesis. He denies vomiting prior to this episode. What is the most likely cause of his bleeding? A. Erosive gastritis B. Gastric neoplasm C. Mallory-Weiss tear D. Zenker diverticulum

Explanations (c) A. Drugs and alcohol are the most common causes of upper gastrointestinal bleeding

125. Clinical Therapeutics/Pulmonology A 55-year-old man with a history of chronic bronchitis presents with two days of increased dyspnea and cough with worsening purulent sputum production. He is currently using inhaled albuterol as needed. In addition to systemic corticosteroids, what pharmacologic agent is warranted at this time for treatment of this patient? A. Antibiotic B. Inhaled corticosteroid C. Long acting beta-agonist D. Theophylline

Explanations (c) A. Empiric antibiotic treatment is indicated in the treatment of acute exacerbations of COPD if there are sputum changes suggestive of bacterial infection, such as increased quantity and purulence

194. Diagnostic Studies/Gastrointestinal/Nutritional A 65-year-old patient with known history of alcohol and tobacco abuse presents with solid food dysphagia. The patient also has a 24 lb weight loss over the past 6 months. Which of the following is the most appropriate intervention? A. endoscopy with biopsy B. chest x-ray C. barium esophagogram D. CT scan of the thorax

Explanations (c) A. Endoscopy with biopsy establishes the diagnosis of esophageal cancer with a high degree of reliability.

132. Clinical Intervention/Cardiology Following an acute anterolateral myocardial wall infarction two days ago, a patient suddenly develops hemodynamic deterioration without EKG changes occurring. What complication can explain this scenario? Answers A. Free wall rupture B. CVA C. Atrial fibrillation D. Sick sinus syndrome

Explanations (c) A. Free wall rupture is a complication that occurs within 72 hours of infarction. It is seen mainly in Q wave transmural and lateral wall infarctions.

187. Diagnosis/Gastrointestinal/Nutritional A 35-year-old female 2 months status-post gastric bypass surgery develops abdominal cramps, diarrhea, palpitations, sweating, and nausea shortly after eating. Which of the following is the most likely diagnosis? A. Dumping syndrome B. Inflammatory bowel disease C. Surgical Adhesions D. Zollinger-Ellison syndrome

Explanations (c) A. Gastric bypass puts this patient at risk for dumping syndrome. Her symptoms represent both cardiovascular and gastrointestinal reactions which occur after eating. The underlying effect is due to the stomach's inability to regulate its rate of emptying.

189. Health Maintenance/Cardiology A 40 year-old G3P3003 female presents complaining of dull aching discomfort of her lower extremities, which is worse in the evening. The patient currently works as a waitress. Examination reveals dilated, tortuous veins beneath the skin in the thigh and leg bilaterally. Which of the following is the best initial approach to prevent progression of disease and complications in this patient? A. Compression stockings B. Warfarin (Coumadin) therapy C. Sclerotherapy D. Clopidogrel (Plavix)

Explanations (c) A. Graduated compression stockings can be used in patients with early varicosities to prevent progression of the disease and when used with leg elevation complications from varicose veins can be avoided.

40. Diagnostic Studies/Cardiology A 36 year-old male complains of occasional episodes of "heart fluttering". The patient describes these episodes as frequent, short-lived and episodic. He denies any associated chest pain. Based on this information, which one of the following tests would be the most appropriate to order? Answers A. Holter monitor B. Cardiac catheterization C. Stress testing D. Cardiac nuclear scanning

Explanations (c) A. Holter monitoring is a non-invasive test done to obtain a continuous monitoring of the electrical activity of the heart. This can help to detect cardiac rhythm disturbances that can correlate with the patient symptoms. Cardiac catheterization is an invasive procedure done to assess coronary artery disease. Stress testing and cardiac nuclear scanning are non-invasive testing maneuvers done to assess coronary artery disease.

1. History & Physical/Cardiology When performing a pre-participation sports physical in the adolescent population, a murmur with which of the following qualities indicates a risk for sudden death during exercise? A. Increases with the Valsalva maneuver B. Increases with squatting maneuver C. Associated with a mid-systolic click D. Mid-systolic without radiation to the carotids

Explanations (c) A. Hypertrophic cardiomyopathy (HCM) is a known cause of sudden death during or just after physical exertion and competitive sports. The murmur associated with HCM is worsened by conditions that cause reduced ventricular volume such as the Valsalva maneuver, sudden standing, and tachycardia.

80. Diagnosis/Cardiology A 23 year-old male presents with syncope. On physical examination you note a medium-pitched, mid-systolic murmur that decreases with squatting and increases with straining. Which of the following is the most likely diagnosis? A. Hypertrophic cardiomyopathy B. Aortic stenosis C. Mitral regurgitation D. Pulmonic stenosis

Explanations (c) A. Hypertrophic cardiomyopathy is characterized by a medium- pitched, mid-systolic murmur that decreases with squatting and increases with straining.

Diagnosis/Endocrinology An adult presents with a three-month history of progressive severe muscle cramps, extremity paresthesias, and lethargy which began shortly after a thyroidectomy for a malignant thyroid lesion. Which of the following is the most likely diagnosis? A. Hypoparathyroidism B. Hypothyroidism C. Hyperparathyroidism D. Hyperthyroidism

Explanations (c) A. Hypocalcemia secondary to hypoparathyroidism is commonly seen as a complication of thyroidectomy.

Scientific Concepts/Urology/Renal Which of the following is the most common cause of acute epididymitis in men under the age of 40? A. Chlamydia trachomatis B. Ureaplasma urealyticum C. Pseudomonas aeruginosa D. Escherichia coli

Explanations (c) A. In men under the age of 40, acute epididymitis is typically caused by Chlamydia trachomatis and Neisseria gonorrhoea

94. Health Maintenance/Dermatology A 67-year-old male presents with worsening lower extremity edema. He denies chest pain, shortness of breath or history of congestive heart failure. On examination, the patient's lungs are clear to auscultation bilaterally and his cardiac exam reveals a regular rate and rhythm. His lower extremity physical examination is significant for 1+ pitting edema to the mid-shin, multiple varicosities, and hyperpigmentation without skin breakdown. His ankle/brachial index (ABI) is within normal limits. You explain your concern to the patient about the development of stasis ulcers. Which of the following is the most important preventative step for this patient? A. Compression stockings B. Immobilization of the affected leg C. Diuretic therapy D. Referral to a vascular surgeon

Explanations (c) A. In the setting of venous insufficiency, compression stockings are the best preventative tool to developing stasis ulcers by reducing edema.

Diagnosis/Hematology A 35-y/o female presents with fatigue and dyspnea on exertion. She has a history of increasingly heavy menstrual periods for the last 8 months. She denies change in stool.Examination reveals HR 102 bpm, TEMP 97.6 F, RESP 20, BP 100/60 mmHg,pallor & and cheilosis.Peripheral smear demonstrates a hypochromic microcytic anemia and few target cells.Which of the following is the most likely diagnosis? A. Iron deficiency anemia B. Vitamin B12 deficiency C. Thalassemia D. Sickle cell anemia

Explanations (c) A. Iron deficiency anemia is characterized by pallor and cheilosis and is commonly associated with menorrhagia. The peripheral smear will demonstrate hypochromic microcytic anemia and few target cells.

A mean corpuscular volume (MCV) of less than 80 cubic microns is a manifestation of which of the following diagnoses? A. Iron deficiency B. Vitamin B12 deficiency C. Folate deficiency D. G6PD deficiency

Explanations (c) A. Iron deficiency is associated with microcytic anemia.

121. Clinical Therapeutics/Gastrointestinal/Nutritional Initial pharmacologic treatment of acute hepatic encephalopathy consists of A. lactulose. B. omega-3-fatty acids. C. neomycin. D. mannitol. metabolism.

Explanations (c) A. Lactulose acts as an osmotic laxative decreasing ammonia absorption and decreases ammonia production by directly affecting bacterial

72. Diagnosis/Cardiology A 58-year old male presents for a six-week follow-up after an acute anterior wall myocardial infarction. He denies chest pain and shortness of breath. Electrocardiogram shows persistent ST-segment elevation in the anterior leads. An echocardiogram reveals a sharply delineated area of scar that bulges paradoxically during systole. Which of the following is the most likely diagnosis in this patient? A. Left ventricular aneurysm B. Postinfarction ischemia C. Ischemic cardiomyopathy D. Constrictive pericarditis

Explanations (c) A. Left ventricular (LV) an aneurysm develops in about 10-20 percent of patients following acute myocardial infarctions, especially anterior wall myocardial infarctions. LV an aneurysm is identified by ST-segment elevation that is present beyond 4-8 weeks after the acute infarct and a scar that bulges paradoxically during systole on echocardiogram.

208. History & Physical/Pulmonology A 55-year-old female presents to the emergency department with complaints of dyspnea, chest pain and coughing with hemoptysis. Past medical history includes breast cancer 5 years ago, currently in remission. Vital signs are Temp. 98.6 degrees F, BP 150/90 mmHg, P 110 bpm, RR 20. Physical examination shows her right leg swollen with pain on palpation of deep veins. Which of the patient's history or examination findings is most suggestive of a pulmonary embolus (PE)? A. Leg swelling and pain with palpation of deep veins B. Heart rate > 100 C. Hemoptysis D. Past history of cancer

Explanations (c) A. Leg swelling and pain with palpation of the deep veins are consistent with a DVT and increase the likelihood of a PE.

Scientific Concepts/Gastrointestinal/Nutritional Gallstones usually result in biliary symptoms by causing inflammation or obstruction following migration into the common bile duct or A. cystic duct. B. pancreatic duct. C. duodenal ampulla. D. common hepatic duct.

Explanations (c) A. Obstruction of the cystic duct by gallstones causes the typical symptom of biliary colic. Once obstructed the gallbladder distends and becomes edematous and inflamed. Gallstones can also migrate into the common bile duct through the cystic duct leading to a condition known as choledocholithiasis.

Diagnostic Studies/Gastrointestinal/Nutritional A 45 year-old female presents with a ten pound weight loss and recurrent greasy stools mixed with diarrhea. The patient notes that these symptoms are worse with certain foods. Which of the following laboratory tests should initially be ordered? A. anti-endomysial antibodies B. anti-mitochondrial antibodies C. anti-glomerular basement membrane antibodies D. anti-phospholipid antibodies

Explanations (c) A. Patients with celiac sprue disease are likely to have anti-endomysial antibody formation.

219. Clinical Intervention/Cardiology A 68 year-old patient presents after a syncopal episode. The patient has a history of coronary artery disease and ischemic cardiomyopathy. Echocardiogram shows an ejection fraction of 20%. Electrophysiology study reveals inducible sustained ventricular tachycardia from the left ventricle. Which of the following is the most appropriate therapy in this patient? A. implantable defibrillator B. metoprolol (Lopressor) C. radiofrequency ablation D. warfarin (Coumadin)

Explanations (c) A. Patients with symptomatic ventricular tachycardia (VT) or sustained VT and left ventricular dysfunction are at increased risk for sudden cardiac death. An implantable defibrillator is the treatment of choice.

176. Clinical Intervention/Endocrinology Radioactive iodine (I131) is most successful in treating hyperthyroidism that results from A. Grave's disease. B. subacute thyroiditis. C. Hashimoto's thyroiditis. D. papillary thyroid carcinoma.

Explanations (c) A. Radioactive iodine (I131) is an excellent method to destroy overactive thyroid tissue of Grave's disease.

139. Clinical Intervention/Endocrinology Radioactive iodine is most successful in treating hyperthyroidism that results from Answers A. Grave's disease. B. subacute thyroiditis. C. Hashimoto's thyroiditis. D. papillary thyroid carcinoma.

Explanations (c) A. Radioactive iodine is an excellent method to destroy overactive thyroid tissue of Grave's disease.

Clinical Intervention/Endocrinology Radioactive iodine is most successful in treating hyperthyroidism that results from Answers A. Grave's disease. B. subacute thyroiditis. C. Hashimoto's thyroiditis. D. papillary thyroid carcinoma.

Explanations (c) A. Radioactive iodine is an excellent method to destroy overactive thyroid tissue of Grave's disease.

Clinical Therapeutics/Gastrointestinal/Nutritional A 48 year-old male presents with complaints of heartburn that occurs approximately 45 minutes after eating about three times a week that is relieved by antacids. He claims to have followed advice about elevating the head of the bed, avoiding spicy foods, and losing weight, but continues to have heartburn. Which of the following is the most appropriate next step? A. Ranitidine (Zantac) B. Sucralfate (Carafate) C. Metoclopramide (Reglan) D. Misoprostol (Cytotec)

Explanations (c) A. Ranitidine, an H2 receptor blocker, is indicated for the treatment of mild, intermittent symptoms of gastroesophageal reflux disease.

A 66 year-old woman with type 2 diabetes comes to the office because she has had a painful red patch that has been spreading on her right lower leg over the past 3 days. Her temperature is 100.4 degrees F. Physical examination of her right lower leg reveals a 3 cm tender, warm, erythematous, and edematous plaque. Which of the following pathogens is most likely causing this patient's symptoms? A. Staphylococcus aureus B. Haemophilus influenzae C. Proteus mirabilis D. Escherchia coli

Explanations (c) A. S. aureus and group A beta-hemolytic streptococci are the most common causes of cellulitis.

121. Diagnosis/Endocrinology A 53-year-old female who is well known for the practice presents to the office complaining of increasing fatigue, constipation, and a weight gain of 10 lb (4.5 kg) over the past year. She also states others have noticed a recent hoarseness to her voice, and she is bothered by "Charley horses" in her legs that wake her up at night. Her past medical history is unremarkable except for a history of hyperthyroidism treated by radioactive iodine 5 years ago. She is currently taking no medications and has no known drug allergies. Which of the following is the most likely cause of the patient's symptoms? A. Hypothyroidism B. Hypoparathyroidism C. Vocal cord paralysis D. Radiation thyroiditis .

Explanations (c) A. The current symptoms, along with the past treatment of hyperthyroidism with radioactive iodine, would indicate hypothyroidism

17. Clinical Intervention/Gastrointestinal/Nutritional Which of the following is the most important intervention in acute pancreatitis? A. IV fluid administration B. Antibiotic administration C. Calcium replacement D. Antiemetics

Explanations (c) A. The mainstay of management in acute pancreatitis is fluid resuscitation. Isotonic solutions are best to maintain renal perfusion and urine output > 100 ml/hour.

164. History & Physical/Endocrinology A 30-year-old female with diabetes mellitus type 1 presents for her annual exam with complaints of intermittent nausea with occasional vomiting, diarrhea and abdominal pain worsening over the past six months. She states she has been irritable lately and feels anxious today. Her menses ceased two months ago. On physical examination, she is mildly hypotensive. Urine pregnancy test is negative. Which of the following are you also likely to find on her physical examination? A. Hyperpigmentation over the knuckles B. Increased axillary hair C. Hypopigmented skin lesions D. Weight gain

Explanations (c) A. The patient exhibits signs and symptoms consistent with Addison disease (chronic adrenal insufficiency) which can be a complication of diabetes. Skin examination tends to show hyperpigmentation in areas of non-sun-exposed skin, as well as sun-exposed areas. This effect is particularly clear in areas of skin creasing such as knuckles, elbows, and posterior neck.

23. History & Physical/Gastrointestinal/Nutritional A 68-year-old woman presents with intermittent crampy abdominal pain and vomiting of 6 hours' duration. She feels bloated but denies rectal bleeding. Examination of the abdomen reveals no palpable mass and you note an upper midline scar from previous abdominal surgery. An abdominal radiograph demonstrates dilated loops of small bowel with air-fluid levels. Which of the following is the most likely cause of this patient's symptoms? A. Adhesions B. Sigmoid volvulus C. Incisional hernia D. Intussusception

Explanations (c) A. The patient's surgical history, presenting symptoms and abdominal imaging are consistent with small bowel obstruction. Peritoneal adhesions are the most common cause of small bowel obstruction.

History & Physical/Gastrointestinal/Nutritional A 68-year-old woman presents with intermittent crampy abdominal pain and vomiting of 6 hours' duration. She feels bloated but denies rectal bleeding. Examination of the abdomen reveals no palpable mass and you note an upper midline scar from previous abdominal surgery. An abdominal radiograph demonstrates dilated loops of small bowel with air-fluid levels. Which of the following is the most likely cause of this patient's symptoms? A. Adhesions B. Sigmoid volvulus C. Incisional hernia D. Intussusception

Explanations (c) A. The patient's surgical history, presenting symptoms and abdominal imaging is consistent with small bowel obstruction. Peritoneal adhesions are the most common cause of small bowel obstruction.

34. Diagnostic Studies/Endocrinology A patient presents 5 days status-post thyroidectomy with muscle cramping and altered mental status. On physical exam, you elicit a positive Chvostek sign. Which of the following is the expected laboratory result for this condition? A. Decreased serum calcium B. Decreased serum phosphate C. Elevated parathyroid hormone D. Elevated serum calcium

Explanations (c) A. The patient's symptoms are consistent with hypoparathyroidism. Expected lab results include low serum calcium, high serum phosphate, low urine calcium, normal alkaline phosphatase, and low PTH levels.

18. Diagnosis/Gastrointestinal/Nutritional A 40-year-old female complains of acute right upper quadrant pain radiating to the back and low-grade fever. Laboratory evaluation indicates the presence of urinary bilirubin and an elevation of serum alkaline phosphatase. Which of the following is the most likely diagnosis? A. cholecystitis B. viral hepatitis C. Gilbert's syndrome D. Dubin-Johnson syndrome

Explanations (c) A. The presence of urinary bilirubin indicating conjugated hyperbilirubinemia coupled with the elevation of serum\ alkaline phosphatase suggests biliary obstruction that may lead to cholecystitis.

65. Diagnostic Studies/Cardiology Which of the following is a non-invasive quick method of evaluating a patient with suspected lower extremity arterial insufficiency? A. Ankle-Brachial Index B. Striker Tonometry C. CT Angiography D. Lower extremity arteriography

Explanations (c) A. The single most useful index is the ankle pressure. This can be obtained with an Ankle-Brachial Index (ABI) which the severity of signs and symptoms of arterial insufficiency are correlated to the findings on the ABI. It is a non-invasive study that can be performed in an office setting.

134. Clinical Intervention/Cardiology A 48 year-old male presents to the ED with complaints of chest pressure, dyspnea on exertion, and diaphoresis that has been present for the last one hour. Electrocardiogram reveals normal sinus rhythm at 92/minute along with ST segment elevation in leads V3-V5. Initial cardiac enzymes are normal. What is the next most appropriate step in the management of this patient? Answers A. Coronary artery revascularization B. Admission for medical management C. Administer lidocaine D. Administer nitrates

Explanations (c) A. The standard of care for the management of acute ST-segment elevation MI is coronary artery revascularization. This patient is diagnosed with an ST-segment elevation MI based upon his history and EKG findings. Cardiac enzymes are normal because of the early presentation of this patient.

199. Clinical Intervention/Cardiology A 52 year-old patient with episodes of syncope has an electrocardiogram which shows a consistently prolonged PR interval with a missing QRS every two beats. Which of the following is the most effective management? A. Permanent pacing B. Beta-blocker C. ACE Inhibitor D. Defibrillation .

Explanations (c) A. This is consistent with ECG findings of a Mobitz type II AV block. Since the patient is symptomatic this type of AV block requires a permanent pacing to prevent total AV disassociation.

212. Diagnostic Studies/Cardiology A 45 year-old male presents to the Emergency Department complaining of sudden onset of tearing chest pain radiating to his back. On examination the patient is hypertensive and his peripheral pulses are diminished. Electrocardiogram shows no acute ST-T wave changes. Which of the following is the diagnostic study of choice in this patient? A. Computed tomography (CT) scan B. Transthoracic echocardiogram C. Magnetic resonance imaging (MRI) D. Cardiac catheterization

Explanations (c) A. This patient has signs and symptoms of acute aortic dissection for which CT scan is the diagnostic study of choice.

153. Clinical Intervention/Cardiology A 66 year-old female with a history of coronary artery disease presents with a new onset of dizziness and fatigue for two weeks. She recalls nearly passing out on one occasion. Examination is unremarkable except for bradycardia. Electrocardiogram (ECG) reveals a heart rate of 50 with a normal PR interval followed by a normal QRS. There are several non-conducting P waves and no lengthening of the PR interval. Which of the following interventions is the therapy of choice? A. Permanent pacemaker B. Radio-frequency ablation C. Maze procedure D. Automatic Implantable Cardioverter Defibrillator

Explanations (c) A. This patient has symptomatic second degree type II heart block and requires a pacemaker.

116. Diagnosis/Cardiology A 16-year-old athlete with no past medical history collapses after running 50 yards down the field. He is unresponsive, pulseless and cyanotic. Which of the following is the most likely cause of this student's collapse? A. Hypertrophic cardiomyopathy B. Myocardial infarction C. Pulmonary embolism D. Reactive airway disease

Explanations (c) A. This presentation is consistent with hypertrophic cardiomyopathy which may initially be difficult to diagnose. Infants but not older children frequently present with signs of CHF. Older children may be asymptomatic, with sudden death as the initial presentation

Diagnostic Studies/Gastrointestinal/Nutritional Which of the following diagnostic tests is considered to be the best initial test to order in a patient with suspected gallbladder disease? Answers A. Ultrasound B. Hepatic iminodiacetic acid (HIDA) scan C. Flat plate of the abdomen D. Endoscopic retrograde cholangiopancreatography (ERCP)

Explanations (c) A. Ultrasound of the abdomen is the best test for checking the extra-hepatic biliary tree for ductal dilatation and choledocholithiasis. (u) B. HIDA scan is usually ordered to assess gallbladder function. It is mostly ordered if initial ultrasound is normal and there is still a high index of suspicion for gallbladder disease. (u) C. Flat plate of the abdomen will only identify about 10 to 15% of gallstones. (u) D. ERCP is performed to remove gallstones that have become lodged in the common bile duct. It is not an initial study that is performed.

A 36-year-old female presents for a refill of her oral contraceptives. She admits to smoking one pack of cigarettes per day. She should be counseled with regard to her risk of Answers A. venous thrombosis. B. varicose veins. C. atherosclerosis. D. peripheral edema.

Explanations (c) A. Women over age 35 who smoke are at increased risk for the development of venous thrombosis

56. Clinical Intervention/Pulmonology A patient presents with increasing shortness of breath. On examination there are decreased breath sounds on the right with hyperresonance to percussion. There is tracheal deviation to the left. Of the following which intervention is indicated? A. Open thoracotomy B. Needle decompression C. Thoracostomy tube D. Mechanical ventilation

Explanations (c) B. A tension pneumothorax is a medical emergency. If the tension in the pleural space is not relieved the patient is likely to die. Initial treatment of choice is insertion of a large bore needle into the pleural space.

210. Scientific Concepts/Cardiology Which of the following is the most common cause for acute myocardial infarction? A. Occlusion caused by coronary microemboli B. Thrombus development at a site of vascular injury C. Congenital abnormalities D. Severe coronary artery spasm

Explanations (c) B. Acute myocardial infarction occurs when a coronary artery thrombus develops rapidly at a site of vascular injury. In most cases, infarction occurs when an atherosclerotic plaque fissures, ruptures, or ulcerates and when conditions favor thrombogenesis so that a mural thrombus forms at the site of rupture and leads to coronary artery occlusion.

14. Diagnosis/Cardiology A 56 year-old male with a known history of polycythemia suddenly complains of pain and paresthesia in the left leg. Physical examination reveals the left leg to be cool to the touch and the toes to be cyanotic. The popliteal pulse is absent by palpation and Doppler. The femoral pulse is absent by palpation but weak with doppler. The right leg and upper extremities have 2+/4+ pulses throughout. Given these findings what is the most likely diagnosis? A. Venous thrombosis B. Arterial thrombosis C. Thromboangiitis obliterans D. Thrombophlebitis

Explanations (c) B. Arterial thrombosis has occurred and is evidenced by the loss of the popliteal and dorsalis pedis pulse. This is a surgical emergency. Venous occlusion and thrombophlebitis do not result in loss of arterial pulse.

197. Diagnostic Studies/Cardiology A 63 year-old patient was admitted with an acute non-ST elevation myocardial infarction 3 days ago confirmed by elevated CK, CK-MB, troponin I and troponin T. He begins to experience recurrent chest pain. Which laboratory study is most appropriate to evaluate his recurrent chest pain? A. Creatine kinase B. CK-MB C. Troponin D. Myoglobin (Mb)

Explanations (c) B. Cardiac-specific markers of myocardial infarction include quantitative determinations of CK-MB, troponin I and T. Troponins are more sensitive and specific than CK-MB. All tests should become positive as early as 4-6 hours after onset of a myocardial infarction and should be abnormal by 8-12 hours. Troponins may remain elevated for 5-7 days or longer. CK-MB generally normalizes within 24 hours, thus being more helpful for evaluation of reinfarction. (u) C. As troponins remain elevated for 5-7 days or longer, they are not useful in the evaluation of a patient with chest pain who already had elevated levels.

80. Diagnosis/Cardiology A patient presents with chest pain. ECG done in the emergency department reveals ST segment elevation in leads II,III, and AVF. This is most consistent with a myocardial infarction in which of the following areas? A. anterior wall B. inferior wall C. posterior wall D. lateral wall

Explanations (c) B. Inferior wall myocardial infarction is characterized by ST segment elevation in leads II, III, and AVF.

121. Diagnosis/Cardiology A 65 year-old male 5 days status-post myocardial infarction is evaluated for anterior chest pain. The pain improves with sitting up. The patient has associated fever, leukocytosis and a pericardial friction rub. Which of the following is the most likely diagnosis? A. Tako-tsubo cardiomyopathy B. Dressler syndrome C. Rupture of papillary muscles D. Recurrent myocardial infarction

Explanations (c) B. Pericarditis may occur 2-5 days after infarction due to an inflammatory reaction to transmural myocardial necrosis. This is known as postmyocardial infarction pericarditis or Dressler syndrome.

57. Diagnosis/Cardiology A 60-year-old male with hypertension is brought to the emergency department 30 minutes after the sudden onset of severe chest pain that radiates to his back and arms. His blood pressure is 180/80 mmHg in his left arm; no blood pressure reading can be obtained from the right arm. ECG shows sinus tachycardia with left ventricular hypertrophy. A high pitched decrescendo diastolic murmur is heard along the left mid-sternal border. Which of the following is the most likely diagnosis? A. Acute myocardial infarction B. Aortic dissection C. Pulmonary embolism D. Right subclavian arterial embolus

Explanations (c) B. This is a classic presentation for aortic dissection.

83. Clinical Intervention/Cardiology Patient who was recently diagnosed with viral pericarditis now complains of severe dyspnea and non-productive cough with pain over the precordial region. His vital signs reveal a heart rate 130bpm and respiratory rate 26. Blood pressure is 130/105 mmHg but fluctuates with inspiration resulting in a 20 mmHg decline in the systolic pressure.Which of the following is the most appropriate therapy for this patient? A. Serial echocardiography B. Urgent pericardiocentesis C. Surgery for pericardial window D. Furosemide (Lasix) bolus

Explanations (c) B. This patient has signs and symptoms consistent with pericardial tamponade. Urgent pericardiocentesis is required.

111. Diagnostic Studies/Hematology A 33 year-old female 2 years status-post gastric bypass surgery presents to the office with complaints of generalized fatigue and paresthesias in her hands. Examination reveals pallor, mild glossitis and decreased vibratory sense. Which of the following laboratory tests is most diagnostic for her condition? A. Serum ferritin B. Serum vitamin B12 C. RBC with indices D. Reticulocyte count

Explanations (c) B. Vitamin B12 deficiency leads to neurologic symptoms of paresthesias, glossitis, and pallor. Gastric bypass surgery aids in malabsorption.

156. Diagnosis/Cardiology A 60-year-old female with a history of radiation therapy for the treatment of cancer presents with progressive dyspnea and fatigue. On examination the patient has lower extremity edema, significant ascites, and an elevated jugular venous pressure that does not fall with inspiration. Heart examination reveals a pericardial knock. Echocardiogram shows rapid early filling and reduced mitral inflow velocities with inspiration. Which of the following is the most likely diagnosis in this patient? A. Pulmonary hypertension B. Atrial myxoma C. Constrictive pericarditis D. Tako-Tsubo cardiomyopathy

Explanations (c) C. Constrictive pericarditis is associated with TB, radiation therapy, cardiac surgery, or following viral pericarditis.There is evidence of right-sided heart failure, a positive Kussmaul sign, and a septal bounce and reduced mitral inflow velocities with inspiration on echocardiogram.

22. Clinical Intervention/Pulmonology A 22 year-old patient complains of sudden onset of chest pain accompanied by shortness of breath. The patient appears dyspneic. On examination, the trachea is deviated to the left, breath sounds are faint on the right, and the right chest is hyperresonant to percussion. The preferable treatment for this patient would be A. a tracheostomy. B. insertion of a chest tube with underwater seal, left 2nd intercostal space. C. needle thoracotomy right 2nd intercostal space D. a lung scan for pulmonary embolus and begin heparin sodium (Heparin) therapy.

Explanations (c) C. Decreased breath sounds and hyperresonance are noted on the side of the pneumothorax; tracheal deviation to the opposite side indicates development of a tension pneumothorax. Treatment consists of inserting a chest tube on the side of the pneumothorax and connecting to an underwater seal.

Clinical Intervention/Urology/Renal Which of the following is the most appropriate intervention for a stage I testicular seminoma? Answers A. Watchful waiting B. Chemotherapy initially C. Orchiectomy and radiation D. Orchiectomy and chemotherapy

Explanations (c) C. Inguinal orchiectomy followed by retroperitoneal radiation therapy cures about 98% of patients with stage I seminoma.

217. Diagnosis/Cardiology A 58 year-old male presents for evaluation after an episode of chest pain noted last night after an argument with his wife. The pain lasted about 20 minutes and was described as a tightness and burning sensation in his chest behind the sternum and radiated to the left shoulder and upper arm. He states the pain improved after he rested. The patient states this is the third such episode during stressful situations this month. Which of the following is the most likely diagnosis for this patient? A. Anginal equivalant B. Tietze syndrome C. Stable angina D. Unstable angina

Explanations (c) C. The diagnosis of angina pectoris depends principally on the history. Stable angina is generally of short duration and subsides completely without residual discomfort. Attacks following a heavy meal or brought on by anger last 12-20 minutes. Attacks lasting more than 30 minutes are unusual and suggest the development of unstable angina, myocardial infarction or an alternative diagnosis.

Which of the following is considered to be the first EKG evidence of acute myocardial infarction? A. ST segment depression B. ST segment inversion C. Peaking of T waves D. Q wave formation

Explanations (c) C. The initial EKG of a patient experiencing an acute MI may not reveal any significant changes at all. When an infarction pattern does occur, the first change that is seen is the peaking of the T waves.

Which of the following mechanisms leads to a primary pneumothorax? Answers A. Penetrating or blunt trauma forces B. Underlying lung cancer C. Pressure of air in the pleural space exceeds room air pressure D. Rupture of subpleural apical blebs due to high negative intrapleural pressures

Explanations (c) D. A primary spontaneous pneumothorax is thought to result from a rupture of subpleural apical blebs secondary to high negative intrapleural pressures.

186. Diagnosis/Cardiology A 55 year-old male presents with complaint of sudden ripping chest pain that radiates into the abdomen. On examination the patient is found to have diminished peripheral pulses and a diastolic murmur. EKG reveals left ventricular hypertrophy. Which of the following is the most likely diagnosis? A. acute myocardial infarction B. pulmonary embolism C. acute pericarditis D. aortic dissection

Explanations (c) D. Aortic dissection is characterized by a ripping or tearing type pain with radiation to the neck, back or abdomen. Left ventricular hypertrophy is often seen on EKG secondary to longstanding hypertension. A diastolic murmur is often present secondary to aortic insufficiency.

12. Clinical Therapeutics/Cardiology A hospitalized patient is found with confirmed pulseless ventricular tachycardia. IV access is obtained following the second shock given. Which of the following medications is to be administered immediately? A. Amiodarone B. Magnesium C. Atropine D. Epinephrine

Explanations (c) D. Epinephrine should be given as soon as IV access is obtained before or after the second shock.

Which of the following valvular heart abnormalities will most likely be seen on echocardiography as a complication of acute myocardial infarction? A. Aortic stenosis B. Aortic regurgitation C. Mitral stenosis D. Mitral regurgitation

Explanations (c) D. In patients with acute myocardial infarction, an echocardiogram can show the severity of mitral regurgitation and the presence of ventricular septal defect if one is present. Acute inferior wall myocardial infarction is associated with acute mitral regurgitation due to necrosis of the posterior papillary muscle which is supplied by the right coronary artery.

Which of the following conditions would cause a positive Kussmaul's sign on physical examination? Answers A. Left ventricular failure B. Pulmonary edema C. Coarctation of the aorta D. Constrictive pericarditis

Explanations (c) D. Kussmaul's sign is an increase rather than the normal decrease in the CVP during inspiration. It is most often caused by severe right-sided heart failure; it is a frequent finding in patients with constrictive pericarditis or right ventricular infarction.

50. Diagnosis/Cardiology The acute onset of congestive heart failure associated with a loud systolic murmur along the left sternal border in a patient with myocardial infarction suggests which of the following? A. Pericarditis B. Cardiac rupture C. Ventricular aneurysm D. Papillary muscle rupture

Explanations (c) D. Papillary muscle rupture occurs in 1% of patients with a myocardial infarction. It presents 2 to 7 days after the infarction with sudden appearance of pulmonary edema and a loud systolic murmur.

222. History & Physical/Cardiology Which of the following is an expected finding in a patient with a diagnosis of an arterial embolism? A. lower extremity edema B. stasis dermatitis C. palpable cord D. pulselessness

Explanations (c) D. Pulselessness is a sign of acute ischemia secondary to arterial embolism.

158. Clinical Intervention/Pulmonology A 21 year-old male presents to the ED with increasing dyspnea and pleuritic chest pain of sudden onset after getting hit in the left side of the chest during a bar fight. Examination reveals moderate respiratory distress with absence of breath sounds and hyperresonance to percussion on the left, with tracheal deviation to the right. Which of the following is the most appropriate next step? Answers A. order a V/Q scan B. order a chest x-ray C. administer a sclerosing agent D. insert large bore needle into left 2nd ICS stat

Explanations (c) D. Simple aspiration by insertion of a needle into the involved side will decompress the tension pneumothorax until a chest tube can be inserted.

165. Diagnosis/Cardiology The 35 year-old patient presents after a syncopal episode while throwing a football with his son. Examination reveals regular heart rate and EKG is normal. There were no symptoms prior to the episode. Right radial pulse is decreased. Which of the following is the most likely explanation for the syncope? A. Carotid sinus hypersensitivity B. Vasovagal episode C. Cardiac dysrhythmia D. Subclavian steal syndrome

Explanations (c) D. Subclavian steal syndrome occurs if the subclavian artery is occluded proximal to the origin of the vertebral artery which results in reversal in the direction of blood flow in the ipsilateral vertebral artery. Exercise of the ipsilateral arm may increase demand on the vertebral flow which produces a "subclavian steal".

127. Clinical Intervention/Cardiology A 34 year-old male presents with an acute onset of fatigue and dyspnea. He has experienced repeated episodes of near-syncope and an unresolved chest discomfort described as a "fluttering" sensation over the past 3 hours. His electrocardiogram reveals no definable p waves and his R-R interval is irregular. His blood pressure is 88/60 mmHg. Which of the following is most appropriate for this patient? A. Initiate warfarin (Coumadin) therapy to an INR target of 2.0 B. Consult for radiofrequency ablation therapy C. Transfer to cardiac catherization lab D. Sedate for synchronized cardioversion

Explanations (c) D. The patient has symptoms and ECG findings consistent with atrial fibrillation. Urgent cardioversion is indicated with shock or severe hypotension, pulmonary edema, or ongoing myocardial infarction or ischemia.

1. History & Physical/Cardiology A 23 year-old male with recent upper respiratory symptoms presents complaining of chest pain. His pain is worse lying down and better sitting up and leaning forward. Electrocardiogram shows widespread ST segment elevation. Which of the following is the most likely physical examination finding in this patient? A. Elevated blood pressure B. Subungual hematoma C. Diastolic murmur D. Pericardial friction rub

Explanations (c) D. This patient has symptoms consistent with acute pericarditis and would most likely have a pericardial friction rub on examination.

202. Diagnosis/Pulmonology On a frontal chest radiograph view, you notice a visceral pleural line with a radiolucent area devoid of vascular and pulmonary markings on the right side only. Which of the following is the most likely diagnosis? A. Asthma B. Emphysema C. Pneumonia D. Pneumothorax

Explanations (c) D. Unilateral findings of a visceral pleural line with no vascular pulmonary markings in between this line in the chest wall are almost always indicative of a pneumothorax. Certain conditions like emphysema can result in a pneumothorax but these findings are not diagnostic of emphysema.

135. Clinical Intervention/Cardiology An unresponsive patient is brought to the ED by ambulance. He is in ventricular tachycardia with a heart rate of 210 beats/min and a blood pressure of 70/40 mmHg. The first step in treatment is to Answers A. administer IV adenosine. B. DC cardiovert. C. administer IV lidocaine. D. apply overdrive pacer.

Explanations (u) A. Adenosine is used to treat PSVT. (c) B. The first step in treatment of unstable ventricular tachycardia with a pulse is to cardiovert using a 100 J countershock.

53. Clinical Intervention/Gastrointestinal/Nutritional A patient presents complaining of vague anal discomfort. On examination, the patient is noted to have a few small external hemorrhoids and edema in the anal region. Which of the following is the most appropriate intervention? A. proctoscopy followed by a hemorrhoidectomy B. increased dietary fiber and sitz baths C. hemorrhoidal banding D. inject a sclerosing agent

Explanations (u) A. Hemorrhoidectomy should be used for permanently prolapsed internal hemorrhoids. (c) B. Most hemorrhoids respond well to conservative treatment such as fiber and sitz baths. (u) C. Banding and injection of sclerosing agents are used if mild prolapse, enlargement, or intermittent bleeding is present

185. Clinical Intervention/Cardiology A 45 year-old female presents with complaint of lower extremity discomfort. The patient admits to dull aching of the left lower extremity. The discomfort is worse after standing for long periods of time. Examination reveals dilated, tortuous and elongated veins on the medial aspect of the left leg. Pedal pulses are +2/4 bilaterally. There are no skin changes or lower extremity edema noted. Which of the following is the most appropriate initial treatment of choice in this patient? A. heparin B. compression stockings C. furosemide (Lasix) D. thrombectomy

Explanations (u) A. Heparin is used in the treatment of deep vein thrombosis not varicose veins. (c) B. This patient has signs and symptoms of varicose veins. Initial treatment with compression stockings may prolong or avoid the need for surgery. (u) C. This patient has no signs of edema or venous insufficiency requiring diuretic therapy. (u) D. Thrombectomy is indicated in a patient with an arterial thrombus, this patient has intact pulses and no pallor.

Which of the following is the most appropriate study for diagnosing Hirschsprung disease? A. Rectal biopsy B. Stool leukocyte test C. CT of the abdomen and pelvis D. Fecal occult blood test

Explanations (c) A. A rectal biopsy showing the absence of ganglion cells in both the submucosal and muscular layers of the involved bowel is the most appropriate diagnostic study for Hirschsprung disease.

While awaiting operative removal of pheochromocytoma, which of the following classes of medications are used for control of hypertension? A. alpha-adrenergic blocker B. beta-adrenergic blocker C. ACE inhibitor D. diuretic

Explanations (c) A. Alpha-adrenergic blockers are used preoperatively to control hypertension in a patient with pheochromocytoma that occurs from unopposed alpha stimulation when the tumor is manipulated.

25 year-old female presents to the ED with an open fracture of the left fibula sustained from an auto accident. The patient has no neurological findings. In addition to stabilization of the patient which of the following should be immediately initiated? A. Antibiotic therapy B. Apply a bi-valve cast C. Reduce the fracture D. Surgical debridement

Explanations (c) A. Antibiotic therapy should be started immediately along with tetanus if needed. Surgically debriding the injury should be initiated immediately after antibiotics are begun.

A 59 year-old otherwise healthy female develops acute dyspnea and chest pain one week post total abdominal hysterectomy. Echocardiogram demonstrates normal heart size with normal right and left ventricular function. Lung scan demonstrates two segmental perfusion defects. Which of the following is the next step in the management of this patient? A. Anticoagulation B. Embolectomy C. Thrombolysis D. Inferior vena cava filter

Explanations (c) A. Anticoagulation is the treatment of choice in patients with pulmonary embolism with normal ventricular function and no absolute contraindications. (u) B. Embolectomy is not indicated as initial treatment of a pulmonary embolism in patients with normal ventricular function. (h) C. Thrombolysis is contraindicated in patients within 10 days of having major surgery. (u) D. An inferior vena cava filter is considered in patients with contraindications to anticoagulation therapy or failed anticoagulation therapy.

A 48 year-old female presents to the clinic complaining of hematuria. The patient states that she was found to have hematuria during an insurance physical examination. The patient denies dysuria or frequency. She also denies pain in the abdomen, flank or meatus. She denies any history of previous nephrolithiasis. Urinalysis reveals the urine to be yellow and slightly hazy with a positive dipstick for hemoglobin. Microscopic reveals 5-7 RBCs/HPF without WBCs, bacteria, casts, or crystals. What is the next diagnostic study this patient should undergo? A. CT urography B. Intravenous pyelogram C. Abdominal ultrasound D. Cystoscopy

Explanations (c) A. CT urography with and without contrast should be done to evaluate the upper and lower urinary tract for neoplasms, and benign conditions such as urolithiasis. This has replaced IVP for imaging of the upper tracts. Abdominal ultrasound will not help in this scenario and the role of renal ultrasound in evaluation of hematuria is unclear. Cystoscopy will help to assess the bladder and urethra but will not help with evaluation of the upper urinary tract.

Which of the following laboratory markers are helpful in establishing the diagnosis of ovarian cancer in a 55 year-old post-menopausal women? A. Ca-125 B. CEA C. LDH D. AFP

Explanations (c) A. Ca-125 is one of the best tumor markers in epithelial ovarian cancer. (u) B. CEA (carcinoembryonic antigen) is a tumor marker used for colon cancer.

Which of the following is the most common radiographic presentation of lung abscess? A. cavitation B. pleural thickening C. hilar mass D. hyperinflation

Explanations (c) A. Cavitation is seen with lung abscess or progressive primary tuberculosis.

What is the term for blue discoloration about the umbilicus? A. Cullen's sign B. Murphy's sign C. Rovsing's sign D. Turner sign

Explanations (c) A. Cullen's sign is a blue discoloration about the umbilicus and can occur in hemorrhagic pancreatitis and results from hemoperitoneum.

A 65 year-old patient with known history of alcohol and tobacco abuse presents with solid food dysphagia. The patient also has a 24 lb weight loss over the past 6 months. Which of the following is the most appropriate intervention? A. endoscopy with biopsy B. chest x-ray C. barium esophagogram D. CT scan of the thorax

Explanations (c) A. Endoscopy with biopsy establishes the diagnosis of esophageal cancer with a high degree of reliability.

A 62 year-old male is brought to the emergency department with acute hematemesis. The patient denies a previous history of vomiting. His wife states he has chronic liver disease. Physical examination reveals a distended abdomen without rebound, guarding or organomegaly. There is a fluid wave. Which of the following is the most likely diagnosis? A. Esophageal varices B. Mallory-Weiss tear C. Arteriovenous malformation D. Perforated duodenal ulcer

Explanations (c) A. Esophageal varices are dilated submucosal veins that develop in a patient with underlying portal hypertension. The most common cause of portal hypertension is cirrhosis.

A 26 year-old gravida 0 sexually active female presents to the emergency room complaining of colicky pain in her lower abdomen for the past 12 hours. She passed out earlier in the day while trying to have a bowel movement. Her last menstrual period was 6 weeks ago. She has noted vaginal spotting over the last 24 hours. Vital signs show Temp 37 degrees C, BP 96/60mmHg, P 110, R 16, Oxygen Sat. 98%. Abdominal exam is positive for distension and tenderness. Bowel sounds are decreased. Pelvic exam shows cervical motion and adnexal tenderness. Which of the following is the most likely diagnosis? A. Ectopic pregnancy B. Appendicitis C. Crohn's disease D. Pelvic inflammatory disease

Explanations (c) A. High suspicion for ectopic pregnancy should be maintained when any possible pregnant woman presents with vaginal bleeding or abdominal pain.

Congenital absence of ganglionic nerve cells innervating the bowel wall is seen in which of the following conditions? A. Hirschsprung's disease B. Meckel's diverticulum C. Chagas disease D. Hashimoto's hypothyroidism

Explanations (c) A. Hirschsprung disease, also termed congenital aganglionic megacolon, results from a lack of ganglion cells in the bowel wall.

78 year-old male with history of coronary artery disease status post CABG and ischemic cardiomyopathy presents with complaint of progressive dyspnea and orthopnea. He also complains of lower extremity edema. The patient denies fever, chest pain, or cough. On physical examination, vital signs are BP 120/68, HR 75 and regular, RR 22, afebrile. You note the patient to have an S3 heart sound, jugular venous distention, and 2+ lower extremity edema. The patient is admitted and treated. Upon discharge from the hospital, the patient should be educated to monitor which of the following at home? A. Daily weights B. Daily spirometry C. Daily blood glucose D. Daily fat intake

Explanations (c) A. Home monitoring of daily weights can alert the health care provider to the early recognition of worsening heart failure.

An adult presents with a three month history of progressive severe muscle cramps, extremity paresthesias and lethargy which began shortly after a thyroidectomy for a malignant thyroid lesion. Which of the following is the most likely diagnosis? A. Hypoparathyroidism B. Hypothyroidism C. Hyperparathyroidism D. Hyperthyroidism

Explanations (c) A. Hypocalcemia secondary to hypoparathyroidism is commonly seen as a complication of thyroidectomy.

25 year-old male presents to the ED with left calf pain and cramping, as well as nausea and vomiting. He admits to "partying with cocaine all night". He describes his urine as a dark brown color. Serum creatinine kinase (CK) is 1325 IU/L (Normal Range 32-267 IU/L). Which of the following is the initial mainstay of therapy for this condition? A. IV rehydration B. Fasciotomy C. Toradol (Ketorlac) D. Hydrotherapy

Explanations (c) A. IV rehydration with crystalloids for 24 to 72 hours is the mainstay of therapy for rhabdomyolysis.

The first step in the treatment of a patient with an intestinal obstruction and no comorbid diseases is A. nasogastric decompression. B. invasive hemodynamic monitoring. C. abdominal exploration. D. administration of antibiotics.

Explanations (c) A. Nasogastric decompression is indicated in all but mild cases of obstruction to prevent distal passage of swallowed air and minimize distension.

Which of the following pathophysiological processes is believed to initiate acute appendicitis? A. Obstruction B. Perforation C. Hemorrhage D. Vascular compromise

Explanations (c) A. Obstruction of the appendiceal lumen by lymphoid hyperplasia, a fecalith or foreign body initiates most cases of appendicitis.

In patients with diabetic retinopathy, what clinical intervention is most successful in preserving vision? A. Panretinal laser photocoagulation B. Iridectomy C. Radial keratotomy D. Vitrectomy

Explanations (c) A. Panretinal laser photocoagulation is indicated for preservation of vision in patients with diabetic retinopathy.

Lab results for a post-operative oliguric patient reveals an increased BUN to creatinine ratio. The patient has a low fractional excretion of sodium (less than 1%). Which of the following is the most likely diagnosis? A. prerenal azotemia B. acute tubular necrosis C. acute glomerulonephritis D. obstructive uropathy

Explanations (c) A. Patients who have prerenal azotemia with otherwise normal kidneys will have severe sodium retention in order to help to save fluid. The amount of sodium in the urine is therefore very low.

Which of the following is first-line treatment for a symptomatic bradyarrhythmia due to sick sinus syndrome? A. Permanent pacemaker B. Radiofrequency ablation C. Antiarrhythmic therapy D. Anticoagulation therapy

Explanations (c) A. Permanent pacemakers are the therapy of choice in patients with symptomatic bradyarrhythmias in sick sinus syndrome.

A patient with renal artery stenosis is unresponsive to medical therapy. Which of the following is the next most appropriate intervention? A. Stenting of the renal artery B. Nephrectomy C. Radioactive iodine D. Lifestyle modifications

Explanations (c) A. Stenting of the renal artery will dilated the narrowed area interfering with blood supply to the kidney.

28 year-old male presents with burns sustained from hot grease splashed on his left hand earlier this afternoon. The burn extends from his palm to the volar aspect of his wrist and has an erythematous base, covered by an intact blister. There are a few small scattered blisters over the dorsum of the left hand. Which of the following is the initial intervention of choice? A. Tetanus prophylaxis B. Admission to a burn unit C. Intravenous fluid administration D. Debridement of blisters

Explanations (c) A. Tetanus prophylaxis should be initially considered in all burn patients.

A 60 year-old male is brought to the ED complaining of severe onset of chest pain and intrascapular pain. The patient states that the pain feels as though "something is ripping and tearing". The patient appears shocky; the skin is cool and clammy. The patient has an impaired sensorium. Physical examination reveals a loud diastolic murmur and variation in blood pressure between the right and left arm. Based upon this presentation what is the most likely diagnosis? A. Aortic dissection B. Acute myocardial infarction C. Cardiac tamponade D. Pulmonary embolism

Explanations (c) A. The scenario presented here is typical of an ascending aortic dissection. In an acute myocardial infarction the pain builds up gradually. Cardiac tamponade may occur with a dissection into the pericardial space; syncope is usually seen with this occurrence. Pulmonary embolism is usually associated with dyspnea along with chest pain.

A 26 year-old female presents to clinic with a left arm that is swollen and non-tender with bluish discoloration along the upper arm and forearm. She is status post pacemaker insertion in the left upper chest for third degree heart block, one week ago. Pulses are present and the arm is warm, but not red. The pacemaker incision is healing well despite a hematoma and tenderness at the site. Which of the following statements would be appropriate patient education about this condition? A. Reassurance that the discoloration is an expected finding. B. Apply cold compresses to the site of the hematoma. C. Elevation of the involved extremity will increase the swelling. D. Aspirin should be taken to help manage pain.

Explanations (c) A. This is indicative of migratory ecchymosis and expected after insertion of a pacemaker.

Trousseau's sign is usually associated with which of the following? A. Hypocalcemia B. Hypokalemia C. Osteomalacia D. Hyperparathyroidism

Explanations (c) A. Trousseau's sign is associated with hypocalcemia.

Ulcerative colitis usually presents with which of the following? A. bloody diarrhea B. toxic megacolon C. fever and left quadrant pain D. alternating constipation and diarrhea

Explanations (c) A. Ulcerative colitis typically presents with episodic bloody diarrhea, lower abdominal cramps, and urgency to defecate. (u) B. Toxic megacolon is a complication of ulcerative colitis, but it is not a common presentation. (u) C. Left lower quadrant pain and a palpable mass accompanied by fever is classic for diverticulitis. (u) D. Irritable bowel syndrome usually presents with constipation, painless diarrhea with mucous, or alternating constipation and diarrhea.

Which of the following beta-adrenergic blocking agents has cardioselectivity for primarily blocking beta-1 receptors? A. Propranolol (Inderal) B. Timolol (Blocadren) C. Metoprolol (Lopressor) D. Pindolol (Visken)

Explanations (c) C. Metoprolol is selective for beta-1 antagonists

Patient with chronic rheumatoid arthritis on maintenance prednisone and methotrexate undergoes surgery. She develops hyponatremia, hypoglycemia, and hypotension. In addition to IV fluid therapy, which of the following is the best initial therapy? A. Epinephrine B. Fludrocortisone (Florinef) C. Hydrocortisone D. Glucagon

Explanations (c) C. The acute phase of adrenal crisis is treated with IV saline and hydrocortisone. (u) B. Fludrocortisone may be utilized in treating the convalescent phase, but not acute phase, of adrenal crisis.

Episodes of hemoptysis. Chest x-ray reveals a hilar nodule with hilar adenopathy and mediastinal widening. What is the most likely diagnosis? A. Large cell lung cancer B. Adenocarcinoma C. Bronchiectasis D. Squamous cell lung cancer

Explanations (u) A. Large cell lung cancer typically presents with peripheral masses. (u) B. Adenocarcinoma typically presents with peripheral masses. (u) C. Bronchiectasis presents on chest x-ray with no findings or prominent cystic spaces with or without air-liquid levels. (c) D. Squamous cell cancer is common in patients with a smoking history and presents with a central mass, hilar adenopathy and mediastinal widening

Which of the following is the most appropriate intervention for a stage I testicular seminoma? A. Watchful waiting B. Chemotherapy initially C. Orchiectomy and radiation D. Orchiectomy and chemotherapy

Explanations (u) A. Surveillance is an option in stage I disease of a nonseminoma testicular tumor. (u) B. Patients with stage IIC and stage III are treated with chemotherapy. (c) C. Inguinal orchiectomy followed by retroperitoneal radiation therapy cures about 98% of patients with stage I seminoma. (u) D. Chemotherapy is used for later stage tumors (II/III) and followed by surgery in stage III tumors

A 60 year-old patient returned from the recovery room to the floor following a subtotal gastrectomy. At 3 AM the next morning, the patient's temperature is 102° F (39° C) and pulse is 112/min. Which of the following is the most likely cause? A. wound infection B. atelectasis C. phlebitis D. shock

Explanations (u) A. Wound infection does not present this early. (c) B. Atelectasis is the most common pulmonary complication, affecting 25% of patients with abdominal surgery. It is more common in elderly and overweight patients and occurs within the first 12 to 24 hours postoperatively. (u) C. Phlebitis occurs more commonly after the second postoperative day. (u) D. In shock, the pulse is usually thready and the temperature is not elevated

Cardiovascular : History and Physical, Acute mitral stenosis

Fusion of the leaflet commissures reducing orifice area. Usually occurs as a consequence of rheumatic fever. Causes decreased filling of the left ventricle, while simultaneously increasing left atrial pressure, producing the syndrome of heart failure. As disease progresses, pulmonary hypertension and right heart failure occur. Si/Sx =hx of rheumatic fever, dyspnea, opening snap on auscultation, diastolic murmur, jugular vein distension, paroxysmal nocturnal hemoptysis, hoarseness, pulmonary edema, A-fib, peripheral edema and ascitese Dx = clinical, confirm with echocardiography f. Tx (1) ~-blockers to slow HR, enabling prolongation of flow of blood across the narrowed valve (3) Anticoagulants for embolus prophylaxis (4) Surgical valve replacement for uncontrollable dzMild disease can be treated symptomatically with diuretics. Moderate to severe disease requires mechanical correction of the valve obstruction by valvotomy, valve repair, or valve replacement.

Gastrointestinal/Nutritional : Diagnostic Studies, Helicobacter pylori

H.pylori screening ● Endoscopy with biopsy → gold standard to rule out malignancy ○ Positive rapid urease test: direct staining of the biopsy specimen for H. pylori ● urea breath test - can be used to test eradication after therapy; ● Positive H. Pylori stool antigen - >90% specific ● Positive serologic antibodies - only useful in confirming infection (not eradication!)

Pt with postpartum uterine bleeding which is the treatment

Hysterectomy

Gastrointestinal/Nutritional : Diagnostic Studies, Ileus Ileus: Inability of the intestine (bowel) to contract normally and move waste out of the body.

Ileus: Inability of the intestine (bowel) to contract normally and move waste out of the body. ● Plain abdominal radiographs → first-line imaging → will show distended loops of small and large bowel due to air as well as air in the rectum with no transition zone Tx: NPO and intravenous hydration plus analgesia plus nasogastric decompression

Patient with Erectile dysfunction, Numbness or tingling, Pain or discomfort (Buttock) and Pain or discomfort (Thigh)

Iliac Artery Stenosis Aortoiliac disease ~ claudication in hip and gluteal muscles, impotence Abdominal aorta→ left and right Common iliac artery→ left internal and external iliac artery and right nternal and external iliac artery

Gastrointestinal/Nutritional: Diagnostic Studies, Pyloric stenosis

In infantile hypertrophic pyloric stenosis (HPS), hypertrophy of the pyloric sphincter results in narrowing of the pyloric canal. Pyloric stenosis leads to progressive and projectile vomiting. DX: Ultrasound abdomen pyloric muscle thickness and a narrowed, elongated pyloric lumen *Ultrasound increased thickness of the pylorus**** GI series shows delayed gastric emptying and a string sign or railroad track sign of a markedly narrowed, elongated pyloric lumen. An upper GI series will reveal a narrowed distal stomach with double tract of barium.*****. Congenital Pyloric Stenosis 1 . nonbilious projectile vomiting in first 2 wks to 2 mos of life 2. More common in boys and in first-born children An olive-sized mass may be palpated in the right upper abdomen in pyloric stenosis and if found, is pathognomonic for pyloric stenosis, representing hypertrophied pyloric sphincter 4. If olive is not present, diagnosis made by Utz 5. Tx = intravenous fluid resuscitation plus pyloromyotomy

Gastrointestinal/Nutritional : History and Physical, Strangulated hernia are Incarcerated hernia with vascular compromise ~ ischemia Inguinal Hernias 1. Most common hernia; more common in men 2. Direct type = viscera protrudes directly through abd wall at Hesselbach's triangle (inferior epigastric artery, rectus sheath, and inguinal ligament), medial to inferior epigastric artery 3. Indirect type is more common (two-thirds are indirect), pass lateral to inferior epigastric artery into spermatic cord covered by cremasteric muscle 4. Inguinal Hernias:Si/Sx intermittent groin mass with bowel sounds that appear during Valsalva maneuvers 5. DDx =femoral hernias, which protrude below the inguinal ligament 6. Dx = physical exam some unable to completely differentiate until surgery 7. Tx =surgical repair with mesh placement

Incarcerated hernias: Hernias that are not reducible Strangulated hernias: Incarcerated hernias with vascular compromise ~ ischemia Strangulated hernias: Incarcerated hernias o Vascular compromise of hernia contents o Usually incarcerated prior to strangulation o ABX Surgical emergency Irreducible hernia: entrapped intestine with blood supply cut off ○ Ischemic, incarcerated hernia with systemic toxicity ○ Pain sudden onset and then is persistent ○ Severe painful BM causing patient to refrain ○ Can mimic symptoms of bowel obstruction (N/V)

Dermatology: Diagnosis, Inflammatory breast cancer

Inflammatory breast cancer symptoms may include persistent itching and the appearance of a rash or small irritation similar to an insect bite. The breast typically becomes red, swollen, tender and warm. The skin may appear pitted like an orange peel, and nipple changes such as inversion, flattening, or dimpling may occur. d'orange ("skin of an orange") painful skin changes around nipple with bloody discharge arises fast and describes anatomy with the appearance and dimpled texture of an orange peel. Peau d'orange is caused by cutaneous lymphatic edema, which causes swelling. Examples include the skin of the breast in inflammatory breast cancer

Patient with breast disease describing peau d'orange dz which is the diagnose

Inflammatory breast disease

Pt female present with an inguinal hernia that reduce easily and reproduce with valsalva manuver

Inguinal hernia Bulge or impulse detected in inguinal canal with Valsalva maneuver or coughing direct inguinal hernia ? hugo An indirect inguinal hernia is typically elliptic that does not reduce easily. (c) B. A direct inguinal hernia is symmetrical, round and disappears easily with the patient lying down.

A 30-year-old man with a history of ulcerative colitis presents with severe abdominal pain and distention. He is currently being treated for an acute colitis flare characterized by 20 bloody bowel movements daily, fevers to 102°F (38.9°C), and left upper quadrant pain. Physical exam reveals tachycardia and hypotension. Abdominal exam shows distention with signs of focal peritonitis in the left lower quadrant.

Toxic Megacolon

Cardiovascular: clinical intervention, Intermittent Claudication

Treatment Urgent assessment for revascularization or amputation Antiplatelet therapy (Cilostazol, ASA, clopidogrel) Analgesia Anticoagulation Heparin Endovascular revascularization and intra-arterial thrombolysis = transluminal angioplasty (PTA) +alteplase surgical revascularization include surgical thrombectomy and bypass amputation

Peripheral Vascular Disease (PVD) clinical intervention

Treatment a. Lifestyle modifications include smoking cessation and increasing moderate exercise b. Pharmacotherapy is Cilostazol (vasodilator and platelet inhibitory properties) c. Minimally invasive therapy includes percutaneous balloon angioplasty (PTA) and/or atherectomy e. Indications for surgical intervention are severe rest pain, tissue necrosis, nonhealing infxn, and intractable claudication f. Surgical treatment includes local endarterectomy with or without patch angioplasty and bypass procedures

Cardiovascular: Clinical Therapeutics, Intermittent Claudication

Treatment a. Lifestyle modifications smoking cessation and increasing moderate exercise b. Pharmacotherapy is Cilostazol (vasodilator and platelet inhibitory properties)

Neurology/Neurosurgery: Clinical Intervention, Carotid stenosis

Tx = modification of risk factors important, anticoagulation, and use of antiplatelet agents (aspirin, dipyridamole) intended to prevent thrombosis. *Surgical therapy is carotid endarterectomy or Carotid angioplasty and stenting pts usually placed on postoperative aspirin therapy* surgery called carotid endarterectomy (CEA) is one way to remove blockages to the normal flow of blood and to minimize your risk of a stroke o If high risk - Carotid angioplasty and stenting o Carotid endarterectomy a) Symptomatic pt (1) Carotid stenosis >70% (2) Multiple TIAs (risk of stroke 10% per year) (3) Pts who have experienced a CV A a b) aSx pt stenosis >75% is an accepted indication

o Gastrointestinal/Nutritional: Clinical Intervention, External hemorrhoids 1 . Varicosity in the lower rectum or anus caused by congestion in the veins of the hemorrhoidal plexus 2. Si/Sx = anal mass, bleeding, itching, discomfort 3. Presence or absence of pain depends on the location of hemorrhoid: internal hemorrhoid is generally not painful, whereas external hemorrhoid can be extremely painful 4. Thrombosed External Hemorrhoid a. Not true hemorrhoid, but subcutaneous external hemorrhoidal veins of the anal canal b. It is classically painful, tense, bluish elevation beneath the skin or anoderm 6. Dx = H&P inspection of the perianal area, ORE, anoscopy, and sigmoidoscopy Tx =conservative therapy consists of a high-fiber diet, Sitz baths, stool-bulking agents, stool softeners, cortisone cream, astringent medicated pads 8. Definitive Tx = rubber band ligation, Sclerotherapy, cryosurgery, and surgical hemorrhoidectomy

Tx =conservative therapy consists of a high-fiber diet, Sitz baths, stool-bulking agents, stool softeners, cortisone cream, astringent medicated pads 8. Definitive Tx = rubber band ligation, Sclerotherapy, cryosurgery, and surgical hemorrhoidectomy Stage 4 internal, external, or mixed internal and external hemorrhoids surgical hemorrhoidectomy

cutaneous burns clinical intervention

Tx Line suitable for outpatient care wound cleaning and topical antibiotic prophylaxis adjunct tetanus immunization adjunct opioid analgesic with suspected wound infection plus antibiotics ± surgical debridement inpatient care assessment for admission to a burn center adjunct fluid resuscitation adjunct supplemental oxygen and supportive care adjunct tetanus immunization adjunct surgery adjunct DVT prophylaxis adjunct intravenous opioid plus benzodiazepine ± nonpharmacologic therapy with suspected wound infection plus antibiotics ± surgical excision

Pre-Operative/Post-Operative Care : Clinical Intervention, Pneumonia Pneumonia 1 . Community-Acquired Pneumonia (CAP) c. Si/Sx =cough, dyspnea, tachypnea, fever, rales on exam Dx: (1) Dx = CXR (2) Sputum Gram stain and culture (3) Blood cultures ( 4) Check an HIV test in all pts with CAP as a matter of routine

Tx: (1) Empiric Tx is with either: (a) A third-generation cephalosporin (e.g., ceftriaxone/ cefotaxime) plus either doxycycline or a second-generation macrolide (e.g., clarithromycin or azithromycin) (b) Monotherapy respiratory fluoroquinolone (levofloxacin, moxifloxacin); OR (3) Narrow Tx based on culture results (b) Monotherapy with a respiratory fluoroquinolone( levofloxacin, moxifloxacin) OR (c) For patients in the ICU, may also give cephalosporin plus a fluoroquinolone (2) If the pt has been recently exposed to antibiotics, consider adding broader coverage for drug-resistant pathogens ( Pseudomonas) (Tx is with broad-spectrum gram-negative and gram-positive agents, typically vancomycin plus (ceftazidime or piperacillin-tazobactam or a carbapenem, etc.)

Gastrointestinal/Nutritional: Clinical Intervention, Chronic anal fissure Most acute anal fissures can be managed conservatively with dietary fiber, prevention of constipation, adequate hydration, and stool softeners. Sitz baths may provide symptomatic improvement.

Tx: An open or closed partial lateral internal sphincterotomy. Surgical treatment is indicated in chronic anal fissures greater than 1 month but not in the treatment of acute anal fissures.

o Gastrointestinal/Nutritional: Clinical Intervention, Obesity, bariatric surgery

Types a. Adjustable Gastric Band The Adjustable Gastric Band - often called the band - involves an inflatable band that is placed around the upper portion of the stomach, creating a small stomach pouch above the band and the rest of the stomach below the band.The common explanation of how this device works is that with the smaller stomach pouch, eating just a small amount of food will satisfy hunger and promote the feeling of fullness. b. The Roux-en-Y Gastric Bypass - often called gastric bypass - is considered the gold standard' of weight loss surgery. c. Sleeve Gastrectomy:The Laparoscopic Sleeve Gastrectomy - often called the sleeve - is performed by removing approximately 80 percent of the stomach. The remaining stomach is a tubular pouch that resembles a banana.

Gastrointestinal/Nutritional : Health Maintenance, Diverticulosis Colonic diverticulosis refers to herniation of mucosa and submucosa through the muscular layer of the colonic wall and may be the result of colonic smooth muscle over-activity. Diverticular disease may be defined as any clinical state caused by symptoms pertaining to colonic diverticula and includes a wide-ranging spectrum from asymptomatic to severe and complicated disease. Diverticulitis indicates inflammation of a diverticulum or diverticula and may be caused by infection. Other complications of diverticular disease include segmental colitis, lower gastrointestinal bleeding, infection, abscess, perforation, peritonitis, and fistula formation.

Usually asymptomatic; may have constipation or nonspecific abdominal symptoms. Symptomatic acute diverticulitis presents with fever, leukocytosis, and left lower quadrant pain. Contrast enema and colonoscopy are common diagnostic tests for the diverticular disease. **CT scan is the imaging modality of choice for acute diverticulitis.*** Treatment includes bowel rest, gradually increasing the fiber content and increasing hydration, antibiotics, and surgical intervention. Complications include bleeding, segmental colitis, perforation, abscess, fistulas, and obstruction. Manage acute bleeding by intravenous fluids or blood transfusion, and radiologically directed hemostasis (embolization).

Gastrointestinal/Nutritional: Clinical Intervention, Perianal abscess

Usually treated by incision and drainage. Cutting or puncturing the skin to drain an infection such as an abscess or boil. Surgical drain A tube inserted into a wound or surgical opening to remove pus, blood, or other fluids.Commonly presents with perianal pain with leukocytosis. Adjunctive antibiotics are unnecessary for uncomplicated cases. Antibiotics are indicated for patients with diabetes, immunocompromise, chronic debilitation, older age, history of a cardiac valvular disease, or associated extensive cellulitis.

Postoperative fever Mnemonic for causes: the SWs (1) Wind (lungs) (2) Water (urinary tract) (3) Wound (4) Walking (DVT) (5) Wonder drug (drug reaction)

Wind POD 1-2 the lungs, i.e. pneumonia, aspiration, and pulmonary embolism; atelectasis fever Water POD 3-5 urinary tract infection, possibly catheter-associated i.e. Foley catheter ) Wound POD 5-7 infection of the surgical incision(s), either superficial or deep (W)abscess POD 5-7 infection of an organ or space Walking POD 5+ (risk may persist for months post-operatively) deep vein thrombosis or pulmonary embolism Wonder drugs or "What did we do?" Anytime drug fever or reaction to blood products,

Pre-Operative/Post-Operative Care: Clinical Therapeutics, Post-operative fever

Wind POD 1-2 the lungs, i.e. pneumonia, aspiration, and pulmonary embolism; atelectasis fever Water POD 3-5 urinary tract infection, possibly catheter-associated i.e. Foley catheter ) Wound POD 5-7 infection of the surgical incision(s), either superficial or deep (W)abscess POD 5-7 infection of an organ or space Walking POD 5+ (risk may persist for months post-operatively) deep vein thrombosis or pulmonary embolism Wonder drugs or "What did we do?" Anytime drug fever or reaction to blood products,

When are stress tests needed before surgery?

You might need a stress test before surgery if you have a serious heart condition. This includes un-controlled heart failure or severe valve disease, chest pain or trouble breathing, or you get tired or winded more easily than you did before. You might also need a stress test before surgery if you have the following risks: You are having intermediate-risk surgery, such as knee or hip replacement, or high-risk surgery, such as bypass surgery for a blocked artery in your leg. And you have diabetes, kidney disease, or a history of coronary artery disease, heart failure or stroke. And you have a hard time walking a short distance or climbing stairs

One question about Zenker diverticulum describing pharyngeal mucosa protruding through submucosa/muscularly at proximal esophagus A Zenker's diverticulum, also pharyngoesophageal diverticulum, also pharyngeal pouch, also hypopharyngeal diverticulum, is a diverticulum of the mucosa of the esophagus, just above the upper sphincter of the esophagus. It is a pseudo diverticulum Si/Sx =dysphagia, regurgitation of solid foods, choking, cough, infection,leftsided neck mass, and bad breath 4. Dx = clinically + barium swallow 5. Tx = myotomy of cricopharyngeus muscle and removal of diverticulum

Zenker's diverticulum

Neurology/Neurosurgery : Diagnosis, Peroneal nerve palsy

Zenker's paralysis, also known as peroneal nerve paralysis or palsy, is a paralysis on common fibular nerve that affects patient's ability to lift the foot at the ankle.The peroneal nerve is part of the peripheral nerve system, and branches from the sciatic nerve in the leg. Injury to the peroneal nerve can cause foot drop, a distinctive way of walking due to an inability to bend the foot upward at the ankle.patients with L5 radiculopathy or sciatic neuropathy will have similar deficits as those with common peroneal neuropathy. The tibialis posterior, innervated by the tibial nerve, receives its main segmental innervation from L5Plain radiographs of the knee and ankle should be obtained to evaluate for concurrent fracture, mass lesion, or arthritis if the history suggests one of these etiologies. Lumbar MRI can provide evidence of L5 radiculopathy if radiographs are negative. Knee and ankle MRI can further elucidate a bony lesion or demonstrate intraneural ganglia.Plain radiographs of the knee and ankle should be obtained to evaluate for concurrent fracture, mass lesion, or arthritis if the history suggests one of these etiologies. Lumbar MRI can provide evidence of L5 radiculopathy if radiographs are negative. Knee and ankle MRI can further elucidate a bony lesion or demonstrate intraneural ganglia.

Gastrointestinal/Nutritional: History and Physical, cirrhosis Cirrhosis o Fibrosis/regenerative nodules o Causes: alcohol > hepatitis, autoimmune o ↑ HCC risk o Gynecomastia, palmar erythema o Hepatic encephalopathy: asterixis, confusion o Portal hypertension: caput medusae, splenomegaly, ascites ↑ Bleeding 1st Tests liver function tests deranged gamma-glutamyl transferase (GGT) elevated serum albumin reduced Tx: liver transplantation

a. Most commonly because of alcoholism or chronic hepatitis B virus (HBV) or hepatitis C virus (HCV) infxn; also 1 o biliary cirrhosis, other chronic hepatic dz (e.g., sclerosing cholangitis, Wilson's dz, hemochromatosis, etc.). Cirrhosis is the pathologic end-stage of any chronic liver disease and most commonly results from chronic hepatitis C and B, alcohol misuse, and nonalcoholic fatty liver disease. The main complications of cirrhosis are related to the development of liver insufficiency and portal HTN and include ascites, variceal hemorrhage, jaundice, portosystemic encephalopathy, hepatorenal and hepato pulmonary syndromes, and the development of hepatocellular carcinoma. Timely referral for liver transplantation is the only curative treatment option for patients with decompensated cirrhosis.

Gastrointestinal/Nutritional: Diagnostic Studies, Bowel obstruction

abdominal X-ray To confirm a diagnosis of intestinal obstruction. However, some intestinal obstructions can't be seen with X-ray. in that case use CT o CT more detailed than a standard X-ray, and are more likely to show an intestinal obstruction. o Ultrasound When an intestinal obstruction occurs in children or pregnant, ultrasound is often the preferred type of imaging.

Pt with peptic ulcer disease evaluated with EGD that showing gastric ulcer that bleeds recently Management

active bleeding ulcer endoscopy ± blood transfusion Endoscopy as a diagnosis and stopping the bleeding.Epinephrine is injected into the bleeding site, together with cautery and/or clip application Endoscopic + epinephrine with cauterization and/or clip application plus proton-pump inhibitor (PPI) 2nd surgery Reserved for perforated ulcers and where endoscopic hemostasis of bleeding ulcers fails.

Urology/Renal: History and Physical, Bladder cancer A 64-year-old man presents with painless hematuria. He had a similar episode 1 year ago and was given antibiotics for a presumed urinary infection and his bleeding resolved. He has a decreased urinary stream and nocturia twice a night. He has smoked a pack of cigarettes daily for 45 years. Physical exam shows only moderate enlargement of the prostate. Urinalysis is positive for 10 to 15 RBCs and 5 to 10 WBCs per high-power field with no bacteria detected.

answer : Bladder cancer Painless Hematuria is the presenting symptom in 85-90% of patients with bladder cancer. Bladder cancer is the second most common urologic cancer and occurs more commonly in men than women. Cigarette smoking is a risk factor in over 60% of patients. The diagnosis and staging of bladder cancer are made by cystoscopy followed by transurethral resection of the bladder tumor. Hx:smoking history presents with painless hematuria for two days. He also complains of frequency and dysuria Dx: urinalysis = hematuria Diagnostic Test urinalysis hematuria is typical but may be absent; pyuria may also be seen, resulting in confusion with urinary infection CT urogram bladder tumors, upper tract tumors, and/or obstruction may be seen cystoscopy visualizes bladder tumors and enables pathologic diagnosis Tx: transurethral resection

Most concerning Sx with a pt with chronic epigastric pain options dyspepsia reflux regurgitation Odynophagia

answer Odynophagia

A 3 week-old male infant presents with recurrent regurgitation after feeding that has progressed to projectile vomiting in the last few days. The mother states that the child appears hungry all of the time and a palpable Olive-sized mass in the right upper abdomen

answer pyloric stenosis

Clinical Intervention/Neurology A 58-year-old female with a history of controlled hypertension is brought to the office after becoming incoherent during lunch 30 minutes ago. Examination reveals a right facial droop and right extremity weakness. A noncontrast head CT is negative for acute hemorrhage and an electrocardiogram reveals normal sinus rhythm. Which of the following is the most appropriate next step in the management of this patient? A. rtPA bolus and IV infusion B. Heparin bolus and IV infusion C. MRI of the brain D. lumbar puncture

c) A. This patient meets the qualifications for rtPA according to the NIH stroke scale. While Heparin is a good choice in the acute setting, the most effective treatment choice would be thrombolytics. A noncontrast CT scan already defined no bleed or obvious mass. Lumbar puncture is ideal in cases where the diagnosis is uncertain. The window of time is appropriate for acute treatment.

A 45 year-old female presents complaining of the worst headache of her life. Which of the following is the most appropriate initial diagnostic study? A. Magnetic resonance imaging (MRI) B. Computed tomography (CT) C. Electroencephalogram (EEG) D. Lumbar puncture (LP)

c) B. A CT scan is more sensitive in detecting cerebral hemorrhage in the first 24 to 48 hours. (u) D. A lumbar puncture is indicated in evaluation of suspected subarachnoid hemorrhage only if the CT scan does not establish the diagnosis.

1. Scientific Concepts/Cardiology Which of the following factors in patients with chronic venous insufficiency predisposes them to development of skin ulcers? A. Increased intravascular oncotic pressure B. Leakage of fibrinogen and growth factors into the interstitial space C. Decreased capillary leakage D. Inherited deficiency of protein C

c) B. Leakage of fibrinogen and growth factors into the interstitial space, leukocyte aggregation, and activation, and obliteration of the cutaneous lymphatic network can predispose a patient to skin ulcers

A 43 year-old male is found to have an elevated serum calcium on routine pre-employment laboratory testing. Further laboratory testing demonstrated: Calcium 11.3 mg/dL (8.8 - 10.2 mg/dL) Ionized Calcium 6.2 mg/dL (4.6 - 5.3 mg/dL) Phosphorus 2.1 mg/dL (3.0 - 4.5 mg/dL) What is the most likely diagnosis? A. Adrenal insufficiency B. Hyperparathyroidism C. Osteoporosis D. Paget's disease

c) B. The presence of elevated total and ionized calcium are consistent with hyperparathyroidism.

Clinical Therapeutics/Gastrointestinal/Nutritional A cirrhotic patient presents with progressive drowsiness and delirium. Physical examination reveals asterixis and tremor. Which of the following medications would you initiate? A. Empiric antibiotic therapy B. Lactulose (Kristalose) C. Bolus fluid challenge with sodium chloride D. Hydrochlorothiazide (Diuril)

c) B. This patient has hepatic encephalopathy. Increased ammonia levels contribute to the mental status changes associated with hepatic encephalopathy. Lactulose leads to a change in bowel flora so that fewer ammonia forming organisms are present and also helps in the acidification of colon contents which leads to a nonabsorbable ammonium ion creation

Diagnostic Studies/Gastrointestinal/Nutritional Which of the following is the diagnostic study of choice in the evaluation of Zollinger-Ellison syndrome? A. D-xylose absorption test B. Gastrin level C. 5-hydroxyindoleacetic acid assay (5-HIAA) D. Urea breath test

c) B. Zollinger-Ellison Syndrome (ZES) is characterized by elevated levels of serum gastrin resulting from gastrin-secreting tumors usually located in the duodenum or pancreas.

Diagnostic Studies/Urology/Renal Which of the following diagnostic findings in the urinary sediment is specific for a diagnosis of chronic renal failure? A. Hematuria B. Proteinuria C. Broad waxy casts D. Hyaline casts

c) C. Broad waxy casts in urinary sediment are a specific finding in chronic renal failure.

Which of the following is the most appropriate intervention for a stage I testicular seminoma? Answers A. Watchful waiting B. Chemotherapy initially C. Orchiectomy and radiation D. Orchiectomy and chemotherapy

c) C. Inguinal orchiectomy followed by retroperitoneal radiation therapy cures about 98% of patients with stage I seminoma.

History & Physical/Urology/Renal A 22-year-old male presents for a physical. On examination, you palpate a painless mass that cannot be separated from the right testicle. Which of the following is the most likely diagnosis? A. Spermatocele B. Varicocele C. Testicular cancer D. Epididymitis

c) C. Testicular cancer is the most common neoplasm in men 20-35 years of age. It commonly presents as a painless mass which cannot be separated away from the testicle. Any nodule within the testes warrants evaluation for malignancy.

147. Diagnostic Studies/Cardiology Which of the following diagnostic tests should be ordered initially to evaluate for suspected deep venous thrombosis of the leg? A. Venogram B. Arteriogram C. Duplex ultrasound D. Impedance plethysmography

c) C. Ultrasound is the technique of choice to detect deep venous thrombosis in the leg.

TIPS procedures involve the placement of a stent in the liver in order to shunt blood away from the portal vein into the hepatic vein, which bypasses the cirrhotic liver parenchyma. which of the following is complication? Answers A. Esophageal varices B. Worsening of cirrhosis C. Budd-Chiari syndrome D. Increased risk of encephalopathy

c) D. TIPS procedures involve the placement of a stent in the liver in order to shunt blood away from the portal vein into the hepatic vein, which bypasses the cirrhotic liver parenchyma. Its main complication is encephalopathy from the accumulation of toxic substances in the brain since the liver no longer acts as a filter.

74. 3. DIAGNOSIS A patient presents with fatigue, weakness and weight loss. On examination, his blood pressure is 90/60 mm Hg, and there is hyperpigmentation of skin creases, pressure areas, and nipples. Plasma ACTH level is elevated. The most likely diagnosis is which of the following? A. thyrotoxicosis B. hypothyroidism C. Cushing's disease D. Addison's disease

c) D. This is a classic presentation of Addison's disease, lack of inhibition of MSH leads to hyperpigmentation

Pt presenting with choledocholithiasis Common bile duct obstruction Proximal inflammation Obstructive Jaundice Dilated hepatic bile duct Dx = Utz ~ CBD >9-mm diameter (Utz first line for Dx) Tx = ERCP

choledocholithiasis Obstruction of the cystic duct by gallstones causes the typical symptom of biliary colic. Once obstructed the gallbladder distends and becomes edematous and inflamed. Gallstones can also migrate into the common bile duct through the cystic duct leading to a condition known as choledocholithiasis. Dx = Utz ~ CBD >9-mm diameter (Utz first line for Tx = ERCP

Urology/Renal: Clinical Intervention, Nephrolithiasis

confirmed stone: *hydration, pain control, and antiemetics* demonstrated bacteriuria antibiotic therapy surgical decompression *stones ≤10 mm medical expulsive therapy (MET)* *stones ≥10 mm or failed medical therapy surgical removal* o *Extracorporeal shock-wave lithotripsy for renal pelvis calculi.* o For most proximal and mid ureteral calculi <1 cm, ureteroscopic treatment with laser or other lithotripsy device. o *Prevention: increase fluid intake; combat UTIs; minimize calcium and vitamin D*intake for calcium-based stones.

Squamous cell CA in elderly, appears as erythematous firm nodules on sun-exposed areas that eventually ulcerate and crust, frequently preceded by actinic keratosis rough epidermal lesions on sun-exposed areas such as Lower Lip, ears, and nose. Metastasize more than basal ceLL not as much melanoma Squamous cell CA developing facial lesion He describes it as non-painful and non-pruritic but notes it to be extremely scaly. Examination reveals a one centimeter, firm nodule at the right temple with heavy keratinization. There is no fluctuance or skin discoloration. Tx: Excision, radiation

dx: Excisional Biopsy

Patient on sulfonamide with rash described red(target) lesion with purple center sparing the mouth

erythema multiforme minor, mucous membrane involvement is absent or mild.

Pt present with Drug induce Sjs or ten or erythema multiforme minor/ major which one spare mucous membrane

erythema multiforme minor, mucous membrane involvement is absent or mild.

Gastrointestinal/Nutritional: Diagnostic Studies, Gastric cancer

o **Upper GI endoscopy with biopsy** demonstrating carcinoma is required to confirm the diagnosis. o Presents with weight loss and abdominal pain, early satiety although patients with proximal or gastroesophageal junction tumors may present with dysphagia. **flag for a gastric malignancy Early satiety with weight loss**, Cancer that develops in any portion of the stomach and may spread to other organs.

Gastrointestinal/Nutritional: Health Maintenance, Average risk for colon cancer

o 5% lifetime incidence for people at average risk

Pre-Operative/Post-Operative Care: Diagnosis, Wound dehiscence

o Affects about 2% of midline laparotomy wounds. o Serious complication with a mortality of up to 30%. o Due to failure of wound closure technique. o It usually occurs between 7-10 days postoperatively. o Often, it is heralded by serosanguinous discharge from the wound. o It should be assumed that the defect involves the whole of the wound. o Initial management includes: opiate analgesia, sterile dressing to the wound, fluid resuscitation and early return to theatre for re-suture under general anaesthesia.

Gastrointestinal/Nutritional: Clinical Therapeutics, Peptic ulcer disease

o Avoidance of instigating factors such as smoking and NSAIDs H2 Blockers: ○ "tidine" ○ ie: cimetidine, ranitidine ● PPIs: ○ "prazole" ○ ie: omeprazole o H2 blockers or PPIs, mucosal protectors such as bismuth o If the patient is infected with H. pylori o Triple therapy: PPI + bismuth compound + two antibiotics for 14 days o For example: omeprazole + amoxicillin (or metronidazole) + clarithromycin + bismuth compound

o Cardiovascular: Clinical Intervention, CAD

o CABG indications: o 3 vessel CAD with decreased LV function o 2 vessel CAD with proximal LAD stenosis, left anterior descending artery (LAD), o Symptomatic 2 vessel CAD in diabetic patient o Left Main Artery CAD

Pre-Operative/Post-Operative Care: Diagnostic Studies, Pulmonary edema Pulmonary edema is typically caused by filling of alveoli in the lungs by fluid leaking out of the blood.Pulmonary Edema and ARDS (Figure 1 .4) a. Si/Sx = dyspnea, tachypnea, resistant hypoxia, diffuse alveolar infiltrate Differential for pulmonary edema c. Tx = 0xigen, diuretics, positive end-expiratory pressure (PEEP) ventilation

o CXR, Pulse OX, EKG, ABG o Echocardiogram/TEE o Pulmonary artery catheterization - lung capillaries (wedge pressure) (1) If pulmonary capillary wedge pressure <12 = ARDS (2) If pulmonary capillary wedge pressure > 1 S = cardiogenic

Gastrointestinal/Nutritional: History and Physical, Small bowel obstruction

o Crampy abdominal pain that comes and goes o Loss of appetite / Constipation / Vomiting o Inability to have a bowel movement or pass gas o Swelling of the abdomen o M/C due to intestinal adhesions formed after abdominal or pelvic surgery.

o Gastrointestinal/Nutritional: Diagnosis, Gastric ulcer Gastric Ulcer (GU) a. H. pylori found in 70% of GU ulcers; 1 0% caused by ulcerating malignancy b. Si/Sx =gnawing/burning pain in mid-epigastrium, worse with food intake; if ulcer erodes into artery can cause hemorrhage and peritonitis, maybe guaiac positive c. Dx = endoscopy with Bx to confirm not malignant, H. pylori testing as above d. Tx =proton pump inhibitors and antibiotics for H. pylori

o Dx = upper GI endoscopy with Bx to confirm not malignant or upper GI series o H.pylori screening

Cardiovascular: Diagnosis, Myocardial infarction

o EKG: o ST elevation or depression o Q waves o New LBBB o Isoenzymes: o Troponins 2-4 hours after MI, peak at 24-48 hours. persists for 7 days o CKMB 4 hours after MI, peak at 18 hours, resolves at 48 hours

Gastrointestinal/Nutritional: Diagnostic Studies, Gastric cancer

o Endoscopy with biopsy o Red flag for a gastric malignancy Early satiety with weight loss

Pre-Operative/Post-Operative Care : Diagnosis, Hemoperitoneum Hemoperitoneum (sometimes also hematoperitoneum) is the presence of blood in the peritoneal cavity. Focused assessment with sonography for trauma (commonly abbreviated as FAST) is a rapid bedside ultrasound examination performed by surgeons, emergency physicians and certain paramedics as a screening test for blood around the heart (pericardial effusion) or abdominal organs (hemoperitoneum) after trauma.[1]

o Focused assessment with sonography for trauma (FAST) o Paracentesis or Diagnostic peritoneal lavage o Computed tomography o Diagnostic laparoscopy or exploratory laparotomy

Pre-Operative/Post-Operative Care : Clinical Therapeutics, Hypovolemia

o IV Fluids Normal saline or lactated Ringer's (isotonic fluid = same consistency with blood) o Vasopressors (medications such as norepinephrine, epinephrine, dopamine, and vasopressin that increase the blood pressure) Fluids are the preferred treatment initially for hypovolemia. Type-specific packed cells are given when the patient's blood type is identified. Until then O negative packed cells are administered.

Gastrointestinal/Nutritional: Diagnostic Studies, Mesenteric ischemia

o Mesenteric angiography or CT angiography

Gastrointestinal/Nutritional: Diagnostic Studies, Cholelithiasis Cholelithiasis Stone formation in the GB Colicky pain worse with fatty food dx: RUQ US Tx: Tx Elective Cholecystectomy

o Most specific and sensitive test to diagnose Cholelithiasis RUQ ultrasound Dx: RUQ US The ultrasound images display gallstones within the gallbladder consistent with cholelithiasis. This patient has biliary colic, and although the name includes colic, the pain may be a steady constant pain. The physical examination reveals tenderness in the right upper quadrant classically without rebound, guarding or a Murphy's sign

Gastrointestinal/Nutritional : Diagnostic Studies, Small Bowel obstruction

o Patient with a history of pelvic surgery o Complaining of bilious vomiting o PE will show high pitched bowel sounds o Small Bowel Obstruction Abdominal X-ray→ first line imaging →air fluid levels, "stack of coins" or "string of pearls" sign or in step ladder pattern, dilated bowel loops ● Treatment is NGT, surgery: if there is strangulation emergency laparotomy plus fluid resuscitation plus preoperative antibiotic prophylaxis plus nasogastric decompression plus analgesia plus correction of the underlying cause plus surgery

Hematology: Diagnosis, Hodgkin Lymphoma Hodgkin's Lymphoma a. Occurs in a bimodal age distribution, young men and the elderly b. EBV infxn is present in up to 50% of cases c. Si/Sx = Pei-Ebstein B-symptoms-fever, night sweats, malaise, weight loss.; Sx worsen with alcohol intake d. Reed-Sternberg (RS) cells seen on Bx, appear as binucleated giant cells ("owl eyes") or mononucleated giant cell (lacunar cell) e. Tx depends on clinical staging (1) Stage I= 1 lymph node involved~ radiation (2) Stage II = ~2 lymph nodes on same side of diaphragm ~ radiation (3) Stage Ill =involvement on both sides of diaphragm ~ chemotherapy (4) Stage IV= disseminated to organs or extranodal tissue~ chemotherapy (5) Chemotherapy regimens (a) MOPP = mechlorethamine, Oncovin (vincristine), procarbazine, prednisone (b) ABVD = Adriamycin (daunorubicin), bleomycin, vincristine, dacarbazine

o Reed-Sternberg cells in lymph node biopsy o Next step after confirming presence of RSC CXR to see extent of involvement, o bone marrow biopsy o CT scan. o Mediastinal lymphadenopathy with a contiguous spread, symptoms worse with alcohol consumption. o B-symptoms-fever, night sweats, malaise, weight loss.

Gastrointestinal/Nutritional: Diagnosis, Perianal abscess A collection of pus in the tissue around the anus and rectum. o Signs and symptoms: Erythema, fluctuance, and swelling may be found in the perianal region on external examination or in the ischio-rectal fossa on digital rectal examination.

o Usually, a clinical evaluation -including a digital rectal exam, it is sufficient to diagnose an anal abscess.

Which of the following is the most consistent physical examination finding associated with a hydrocele? A. Tender and swollen testicle B. Palpable painless mass on the testicle C. Nontender scrotal mass that transilluminates D. Inability to get exam finger above scrotal mass

otherwise. (c) C. A hydrocele is a fluid-filled mass that is nontender to palpation. Diagnosis is readily made by transillumination, however evaluation is still warranted as 10% of testicular tumors may have an associated hydrocele.

Cardiovascular: Diagnosis, Pneumothorax

pneumothorax o Perform physical examination: o Tympanic or hyperresonate to percussion, decreased breath sounds, hypotension, distended neck veins o *Chest film (CXR upright or semi-upright)* o Tracheal deviation away from the affected side o Absent lung markings on the side of the pneumothorax (1) *Tension pneumothorax* (a) *Causes contralateral mediastinal shift, distended neck veins (central venous pressure [CVP]), hypotension, absent breath sounds, and hyperresonance to percussion on the affected side* (b) Tx = immediate chest tube or 14-gauge needle puncture of the affected side (2) Open pneumothorax ~ Tx = immediate closure of the wound with dressings and placement of a chest tube (3) Flail chest (a) Caused by multiple rib fractures that form a free-floating segment of the chest wall that moves paradoxically to the rest of the chest wall, resulting in an inability to generate sufficient inspiratory or expiratory pressure to drive ventilation (b) Tx =intubation with mechanical ventilation

Cardiovascular: Clinical Intervention, Chronic heart failure

refractory to optimal medical treatment LVEF <35%: no left bundle branch block 1st implantable cardiac defibrillator (ICD) 2nd cardiac transplantation LVEF <30%: left bundle branch block 1st cardiac resynchronization therapy (CRT) with biventricular pacemaker 2nd cardiac transplantation

Gastrointestinal/Nutritional: Diagnostic Studies, carcinoid tumor A carcinoid tumor is a specific type of neuroendocrine tumor. Carcinoid tumors most often develop in the GI tract, in organs such as the stomach or intestines, or in the lungs. However, a carcinoid tumor can also develop in the pancreas, a man's testicles, or a woman's ovaries. More than 1 carcinoid tumor can develop in the same organ. Carcinoid syndrome = serotonin (5-HT) secretion • Si/Sx = recurrent diarrhea, skin flushing, asthmatic wheezing, and carcinoid heart dz • Dx by ↑ 5-HIAA metabolite in urine • Tx = methysergide, a 5-HT antagonist Carcinoid Syndrome a. APUDoma (amine precursor uptake and decarboxylate) b. Occurs most frequently in the appendix c. Carcinoid syndrome results from carcinoid liver metastases that secrete serotonin (5-HT) d. Si/Sx =flushing, watery diarrhea and abd cramps, bronchospasm, right-sided heart valve lesions e. Dx = ↑levels of urine 5-hydroxyindoleacetic acid (5-HIM) f. Tx =somatostatin and methysergide

• Dx by ↑levels of urine 5-hydroxyindoleacetic acid (5-HIM) 5hydroxindoleacetic acid is used as a tumor marker in carcinoid syndrome. 1st Tests To Order Test Result serum chromogranin A/B elevated urinary 5-hydroxyindoleacetic acid elevated

Dermatology : Diagnostic Studies, Tinea corporis

● 1st Tests potassium hydroxide (KOH) microscopy hyphae (branching, rod-shaped filaments of uniform width with septa) Wood's lamp: green fluorescence if due to microsporum

Gastrointestinal/Nutritional : Clinical Intervention, Obesity

● ● Surgical options: include gastric banding, gastric sleeve, gastric bypass, Roux-en-Y gastric bypass, surgery. Laparoscopic sleeve gastrectomy

Pulmonology : Clinical Intervention, Hemothorax

● *Gold standard = thoracentesis* ○ Both diagnostic and therapeutic ● *Chest tube placement and drainage*

Gastrointestinal/Nutritional : Scientific Concepts, Gastric cancer

● *MC=Adenocarcinoma ● Risks: H.pylori, foods containing high nitrites (salted, cured, smoked, pickled), pernicious anemia, smoking, ETOH ● S/S: weight loss, early satiety, abdominal pain/fullness ● Patient often have iron deficiency anemia ● Dx: Upper endoscopy with biopsy ● Tx: gastrectomy, XRT, and chemotherapy ○ Poor prognosis

Pre-Operative/Post-Operative Care : Scientific Concepts, Nicotine effects on wound healing

● *Nicotine is a vasoconstrictor that reduces nutritional blood flow to the skin, resulting in tissue ischemia and impaired healing of injured tissue.* Smoking increases the risk of infection and postoperative ventilation ● Smoking should be stopped before surgery

Urology/Renal : Diagnostic Studies, Renal cell carcinoma ● Renal cell carcinoma: classic triad ○ Hematuria ○ Flank/abdominal pain ○ Palpable mass

● *Renal cell carcinoma: classic triad Hematuria Flank/abd. pain Palpable mass* ● Dx: ○ *CT scan usually is the first test show renal mass, regional lymphadenopathy, and/or visceral/bone metastases* ○ *Renal US, MRI* Tx =resection, systemic interleukin-2 immunotherapy, poor Px

Cardiovascular : History and Physical, Mesenteric artery ischemia

● Acute: due to occlusion 1. Embolus-A.fib 2. Thrombus-atherosclerosis ○ Severe abdominal pain out of proportion to physical findings ○ N/V, diarrhea ● Chronic: 1. Ischemic bowel disease → mesenteric atherosclerosis of GI tract leads to inadequate perfusion ○ Chronic, full abdominal pain worse with meals "intestinal angina" ○ Weight loss (anorexia)

Gastrointestinal/Nutritional : Clinical Therapeutics, Crohn's disease

● Aminosalicylates → corticosteroids → immune modifying agents ● 1st line for colonic disease → aminosalicylate (mesalamine, sulfasalazine) ● Steroids for acute exacerbation/breakthrough flairs ● Immunotherapy ○ Azathioprine ○ Anti-tumor necrosis factor (TNF) therapy with infliximab, adalimumab, ● Surgery is noncurative in Crohn's

Gastrointestinal/Nutritional : Diagnostic Studies, Mesenteric ischemia Acute Intestinal Ischemia a. Acute thrombosis of a mesenteric vessel secondary to atherosclerotic changes or emboli from the heart b. Si/Sx = rapid onset of pain that is out of proportion to exam, vomiting, diarrhea, and history of heart condition predisposing to emboli formation (e.g., atrial fibrillation) c. Dx = angiogram should be performed immediately to confirm or rule out diagnosis d. Tx =surgical treatment of acute obstructive intestinal ischemia includes revascularization, resection of necrotic bowel, and when appropriate a "second look" operation 24-48 hrs after revascularization

● Angiogram → definitive diagnosis

Cardiovascular : Diagnostic Studies, Arterial embolization

● Ankle-brachial index (ABI): simple, quick, noninvasive ○ Positive PAD if ABI <0.9; rest pain if < 0.4; NML is 1-1.2 ● Arteriography → gold standard because it shows length, location and degree of occlusion ○ Clinically usually only performed is revascularization is planned ● Duplex B mode US: noninvasive, used for visualizing stenosis ● Hand held doppler → may be used to help assess distal blood flow and pulses ○ Often used in the ER

Cardiovascular: Diagnostic Studies, Arterial embolization

● Ankle-brachial index (ABI): simple, quick, noninvasive ○ Positive PAD if ABI <0.9; rest pain if < 0.4; NML is 1-1.2 ● Arteriography → gold standard because it shows length, location, and degree of occlusion ○ Clinically usually only performed is revascularization is planned ● Doppler ultrasound watches blood flow, used for visualizing stenosis o Angiogram examines the blood vessels for abnormalities o Doppler ultrasound watches blood flowo MRI takes images of the body to locate blood clots

Dermatology : Clinical Therapeutics, Cellulitis

● Antibiotics for 7-10 days: ○ Cephalexin; Dicloxacillin ○ Erythromycin or Clindamycin (if PCN allergy) ○ Vancomycin or Linezolid (if MRSA) ● If due to cat bite (Pasteurella multocida): ○ Augmentin

Gastrointestinal/Nutritional : Clinical Intervention, Appendicitis PPP 158

● Appendectomy CT scan or diagnostic laparoscopy for "nonclassic" presentation o Appendectomy can be performed open or laparoscopically;

Endocrinology : Diagnostic Studies, Addison's Disease

● Baseline ACTH level is elevated in patients with primary failure due to absence of negative feedback ● ACTH stimulation test: Give ACTH and measure cortisol level after 30 minutes. If adrenal failure is present, there will be no increase in cortisol.● Test will show adrenal insufficiency→ little or no increase in cortisol levels ● Hyponatremia, hyperkalemia (due to ↓ aldosterone, which is normally produced in adrenals).

Pre-Operative/Post-Operative Care : Diagnostic Studies, Malnutrition

● CBC count with RBC indices and a peripheral smear. This could also help exclude anemias from nutritional deficiencies such as iron, folate, and vitamin B-12 deficiencies. Measures protein nutritional status include serum albumin, retinol-binding protein, prealbumin, transferrin, creatinine, and BUN levels. ● BMI, arm circumference Labs occasionally used: a. Serum albumin nml 3.5-5.5 g/dl, severe malnutrition <2.1 g/dl b. Serum pre-albumin levels: nml levels 15.7-29.6 mg/dl, severe malnutrition <8 mg/dl 344 c. Serum transferrin levels: nmllevels 200-400 mg/dl, severe malnutrition <1 00 mg/dl d. Serum retinal-binding protein e. Twenty-four hour collection for urine urea nitrogen

Cardiovascular : Diagnostic Studies, Thoracic aneurysm

● CT scan is the test of choice for thoracic aneurysms (Pance Prep Pearls) Abdominal ultrasound. This test is most commonly used to diagnose abdominal aortic aneurysms. ● CXR ○ Widened mediastinum. ○ Abnormal aortic contour. ○ "Calcium sign"

Endocrinology : Diagnostic Studies, Cold nodule

● Cold nodules (no/low iodine uptake) are highly suspicious for malignancy ● Thyroid ultrasound- often used to help obtain a specimen during Fine-needle aspiration (FNA) with biopsy, used to see if nodule is cystic or solid, to monitor suspicious nodule or to see if growing or shrinking ● Surgery- if thyroid cancer is suspected or if an indeterminate FNA with a cold thyroid scan ○ Total thyroidectomy vs. subtotal thyroidectomy

Gastrointestinal/Nutritional : Diagnostic Studies, Colorectal cancer

● Colonoscopy with biopsy: diagnostic test of choice ● Barium enema: apple core lesion (classic) ● Elevated CEA ● CBC: anemia (iron deficiency classic)

Gastrointestinal/Nutritional : History and Physical, Dumping syndrome

● Common after gastric surgery ● Early: 30-60 min after you eat ○ Feeling of fullness ○ Abdominal cramping or pain ○ Nausea and Vomiting ○ Severe Diarrhea ○ Sweating, flushing, lightheadedness ○ Tachycardia ● Late: 1-3 hours after eating ○ Fatigue or weakness ○ Flushing or sweating ○ Shakiness, dizziness, fainting ○ Tachycardia ○ Mental confusion

Endocrinology : Clinical Intervention, Pheochromocytoma

● Complete adrenalectomy ● Preop nonselective alpha-blockade: PHEnoxybenzamine or PHEntolamine x 7-14 days followed by beta blockers or CCBs to control HTN

24. Gastrointestinal/Nutritional : Clinical Intervention, Hemorrhoids

● Conservative treatment: high-fiber diet, increased fluids. Warm sitz bath and topical rectal corticosteroids (+/- analgesics like lidocaine) may be used for pruritus and discomfort/thrombosis ● Procedures: if failed conservative management, debilitating pain, strangulation or stage IV. Options include rubber band ligation (MC used), sclerotherapy or infrared coagulation ● Hemorrhoidectomy: for all stage IV or those not responsive to the aforementioned therapies.

. Pre-Operative/Post-Operative Care : Clinical Intervention, Deep vein thrombosis

● DVT prevented by early ambulation and mechanical compression stockings ● SQ heparin may be substituted for compression stockings and warfarin Dx (1) Doppler Utz is first line, sensitivity and specificity >90% (2) Gold standard is venography, but this is invasive

Gastrointestinal/Nutritional : Diagnosis, Hepatic encephalopathy

● Decreased liver clearance of accumulated ammonia from protein breakdown ● S/S: A cirrhotic patient presents with progressive confusion, drowsiness, and delirium. Physical examination reveals asterixis and tremor. and lethargy ● Physical: asterixis (flapping tremor) and increased ammonia levels

Pre-Operative/Post-Operative Care : Clinical Therapeutics, Diabetes

● Diabetic patients have increased infection risk, worse wound healing increased cardiac complications risk and increased postoperative mortality ●** Blood sugar levels should be well controlled via subcutaneous insulin sliding scale and frequent glucose checks**

Hematology : Diagnosis, von Willebrand disease ● Clinical manifestations: ○ Mucocutaneous bleeding: easy bruising, epistaxis, gums, GI, menorrhagia von Willebrand factor (vWF) deficiency (1) Most common inherited bleeding dz (2) Si/Sx = episodic ↑ bleeding time and ecchymoses, nml PT/PTT (3) Dx = vWF levels and ristocetin-cofactor test (4) Tx = DDAVP Desmopressin ↑vWF secretion + factor VIII) or cryoprecipitate for acute bleeding

● Diagnosis: 3) Dx = vWF levels and ristocetin-cofactor test ○ Decreased vWF levels and decreased factor VIII ○ Bleeding time and PTT prolongation worse with aspirin

Pre-Operative/Post-Operative Care : Clinical Intervention, Cancer surveillance

● Follow-up after resection of colorectal carcinoma (CRC) ○ Perioperative colonoscopy to remove any synchronous cancer National Comprehensive Cancer Network guidelines recommend 3- to 6-monthly patient clinic review for the first 2 years and then yearly until 5 years post-treatment. For stage I cancers, a colonoscopy should be performed at 1-year post surgery, repeated in 3 years, and then again at 5 years. For stage II and III cancers, a combination of history and physical examination, carcinoembryonic antigen, CT imaging, and colonoscopy at varied intervals is recommended

14. Dermatology : Clinical Therapeutics, Pseudomonas burn infection

● For pseudomonal or pediatric infections, infuse subeschar piperacillin, and plan for emergent operative debridement within 12 hours.

Dermatology : Clinical Therapeutics, Pseudomonas burn infection

● For pseudomonal or pediatric infections, infuse subeschar piperacillin/ tazobactam, ciprofloxacin or levofloxacin, cefepime, and plan for emergent operative debridement within 12 hours.

Pre-Operative/Post-Operative Care : Health Maintenance, Kidney transplant

● General health assessment: ○ Identify comorbidities such as heart disease, COPD, and diabetes. ○ Assess ability to handle immunosuppression and compliance. ● Laboratory evaluation consisting of chemistries, complete blood count ○ (CBC), urinalysis, serologies for hepatitis B and C, cytomegalovirus (CMV), and human immunodeficiency virus (HIV). ● Conduct ECG and chest x-ray (CXR). ● Further cardiac workup based on need. ● Psychosocial evaluation regarding compliance. ● May need bilateral nephrectomy for recurrent urinary tract infections (UTIs) or polycystic kidney disease (PCKD).

Pulmonology : Clinical Intervention, Hemothorax

● Gold standard = thoracentesis ○ Both diagnostic and therapeutic ● Chest tube placement and drainage (4) Massive hemothorax (a) Injury to the great vessels with subsequent hemorrhage into the thoracic cavity (b) Tx =chest tube, surgical control of the bleeding site (c) Surgical indications-Approximately 90% of penetrating thoracic injuries can be treated with tube thoracostomy alone

Endocrinology : Diagnostic Studies, Hyperthyroidism

● Hyperthyroidism ● Labs: Thyroid function tests (TFTs): increased T3 and/or T4 and decreased TSH (negative feedback of ↑ hormone levels). ● Radioactive iodide uptake test (RAIU): Scan shows diffusely increased uptake

Endocrinology : History and Physical, Hypocalcemia

● Hypocalcemia with decreased PTH ○ Hypoparathyroidism is MC cause: Etiology ■ Parathyroid gland destruction (autoimmune, postsurgical) ● Hypocalcemia with increased PTH: Etiologies ○ Chronic renal disease MC cause ○ Vitamin D deficiency (osteomalacia and Rickets) → increased PTH in response to hypocalcemia ○ Hypomagnesemia ● Clinical manifestations: ○ Neuromuscular: muscle cramping bronchospasm, syncope, seizures, finger/circumoral paresthesias ○ Tetany: Chvostek's sign → facial spasm with tapping of the facial nerve; Trousseau's sign → inflation of BP cuff above systolic BP causes carpal spasms; Increased DTR (deep tendon reflexes)

Cardiovascular : Clinical Therapeutics, Atrial fibrillation

● Irregularly irregular rhythm ● Rate control: ○ Vagal, CCB, or Beta-blocker ● Rhythm control: ○ Amiodarone, flecainide ○ A DC Cardioversion (Direct Current Cardioversion) is a procedure to convert an Atrial Fibrillation (AF) ● Prevention of Stoke: ○ Warfarin or aspirin as prophylaxis against thromboembolism

18. Endocrinology : Diagnostic Studies, Hyperthyroidism

● Labs: Thyroid function tests (TFTs)-increased T3 and/or T4 and decreased TSH (negative feedback of ↑ hormone levels). ● Radioactive iodine uptake test (RAIU): Scan shows diffusely increased uptake

Gastrointestinal/Nutritional : Diagnostic Studies, Acute pancreatitis

● Labs: leukocytosis, elevated glucose, elevated bilirubin, elevated triglycerides ○ Lipase: more specific than amylase (elevated 7-14 days) ○ Amylase, ALT elevated ○ Hypocalcemia ● Abdominal CT: diagnostic test of choice ● Abdominal US: assess to rule out gallstones, bile duct dilation, ascites, pseudocysts

Gastrointestinal/Nutritional : Clinical Intervention, Hepatic encephalopathy Hepatic Encephalopathy 1 . Seen in cirrhosis, may result from brain toxicity 2° to excess ammonia and other toxins not degraded by malfunctioning liver 2. Sx =hyperreflexia, asterixis (flapping of extended wrists), dementia, Sz, obtundation/coma 3. Tx = lactulose, neomycin, and protein restriction

● Lactulose: converted into lactic acid by bacteria, neutralizing the ammonia ● Rifaximin, Neomycin: antibiotics that decrease the bacteria producing ammonia in the GI tract ● Protein restriction: reduces the breakdown of protein into ammonia ● Liver transplantation- only definitive treatment

Dermatology : Clinical Therapeutics, Methicillin-resistant Staphylococcus aureus

● MRSA → cellulitis → Antibiotics ○ IV vancomycin or Linezolid, Daptomycin, and Oral: Bactrim

Gastrointestinal/Nutritional : Clinical Therapeutics, Chronic pancreatitis

● Management: oral pancreatic enzyme replacement, ETOH abstinence, pain control chronic symptoms 1st alcohol and cigarette smoking cessation adjunct analgesia adjunct pancreatic enzymes plus proton-pump inhibitor adjunct dietary modifications + enteral feeding adjunct octreotide adjunct antioxidants

Gastrointestinal/Nutritional : Clinical Intervention, Small Bowel Obstruction PPP 155

● Non-strangulated: NPO (bowel rest), IV fluids, bowel decompression (via NG tube suction) ● Strangulated: surgical intervention

Gastrointestinal/Nutritional : Clinical Intervention, Small Bowel Obstruction

● Non-strangulated: NPO (bowel rest), IV fluids, bowel decompression (via NG tube suction) ● Strangulated: surgical intervention

Pre-Operative/Post-Operative Care : Diagnostic Studies, Hypocalcemia

● Numbness and tingling of fingers, toes, and around mouth. ● Increased reflexes. ● Muscle and abdominal cramps. ● Convulsions. ● ECG-prolonged QT interval. ● Chvostek's sign: Tapping over the facial nerve in front of the tragus of the ear causes ipsilateral twitching. ● Trousseau's sign: carpopedal spasm. : a spasmodic contraction of the muscles of the hands, feet, and especially the wrists and ankles spasm following inflation of sphygmomanometer cuff to above systolic blood pressure for several minutes.

Gastrointestinal/Nutritional : Scientific Concepts, Anal fissure

● Painful linear tear in distal anal canal (range: epithelial to mucosa) ● *MC=posterior midline ● Etiologies: low fiber diet, passage of large, hard stools, anal trauma ● S/S: painful BM, bright red blood, rectal pain ● 80% resolve spontaneously ● Tx: ↑fiber diet, sitz bath, nitroglycerin, botox, lateral internal sphincterotomy

Urology/Renal : History and Physical, Testicular cancer

● Painless testicular nodule, solid mass or enlargement unable to be separated from the testicle. May have dull pain or testicular heaviness. ● Gynecomastia in <10% (seen especially with Leydig or Sertoli tumors that secrete estrogen).

Cardiovascular : Clinical Intervention, Peripheral arterial disease

● Platelet inhibitors ○ Cilostazol → mainstay of treatment - helpful for intermittent claudication (it is a vasodilator and ADP inhibitor) ○ Aspirin ○ Clopidogrel (Plavix) ADP inhibitor ● Revascularization ○ PTA (percutaneous transluminal angioplasty) ○ Bypass grafts - fem-pop bypass (autogenous in situ saphenous vein bypass grafts) ○ Endarterectomy ● Supportive care → foot care, exercise: fixed distance walking to the point of claudication, resting and continuing until symptoms occur (do this for 1 hour/day) ● Acute Arterial Occlusion treatment is heparin for acute embolism and thrombolytics of a thrombus; Embolectomy ● Amputation if severe or gangrene occurs

Endocrinology : Scientific Concepts, Hyperparathyroidism

● Primary: increased PTH production ○ Parathyroid adenoma: most common cause (80%) ○ Men I & IIa ○ Malignant tumor (rare) ○ In secondary hyperparathyroidism, the overactivity of the parathyroid glands occurs in a condition such as kidney failure where calcium levels tend to be low, and the parathyroid overactivity is an attempt on the body's part to keep the calcium levels normal. kidney failure (MC cause of secondary) ■ Kidneys convert vitamin D to its usable form ● Tertiary: prolonged PTH stimulation after secondary hypoparathyroidism → autonomous PTH production ● Clinical manifestations: ○ Signs of hypercalcemia → "stones, bones, abdominal groans, psychic moans" ○ Decreased DTR (deep tendon reflexes) MEN 1 = Parathyroid tumors, pancreatic tumors, and pituitary tumors. MEN 2a = Medullary thyroid cancers (MTC), pheochromocytoma, and parathyroid tumors. MEN 2b = Medullary thyroid cancers, pheochromocytoma and neuromas.

Endocrinology : Scientific Concepts, Hypoparathyroidism

● Rare ● Either low PTH or insensitivity to its action ● Etiologies: ○ 2 most common are postsurgical and autoimmune ○ Accidental damage/removal of parathyroid during neck/thyroid surgery ○ Autoimmune destruction of parathyroid gland ○ Radiation therapy, hypomagnesemia (magnesium required for production of PTH), congenital ● Clinical manifestations: ○ Signs of hypocalcemia: carpopedal spasms, Trousseau's & Chvostek's signs, increased DTR (deep tendon reflexes)

. Cardiovascular : History and Physical, Subclavian steal syndrome

● Retrograde blood flow in a vertebral artery associated with proximal ipsilateral subclavian artery stenosis or occlusion ● Difference in upper-extremity pulses and brachial systolic blood pressures between the patient's arms. ● Syncope, neurologic deficits, memory loss ● Arm claudication: poor circulation in the hands(blotchy)

Gastrointestinal/Nutritional : Diagnostic Studies, Irritable bowel syndrome Irritable bowel syndrome (IBS) Symptoms include chronic abdominal pain described as crampy and varying in intensity, accompanied by altered bowel habits. Patients complain of diarrhea, constipation, both diarrhea, and constipation, or normal bowel movements that alternate with diarrhea and constipation. Symptoms usually improve with defecation.

● Rome IV Criteria: ○ Recurrent abdominal pain on average at least 1 day/week in the last 3 months associated with at least 2 of the following: ■ improvement with defecation ■ Onset associated with change in stool frequency ■ Onset associated with change in stool form (appearance)

Cardiovascular : History and Physical, Hypertrophic cardiomyopathy

● S/S: ○ Often asymptomatic ○ MC initial complaint = Dyspnea, Angina pectoris, Syncope ○ Ventricular Fibrillation: Sudden cardiac death ● Harsh systolic crescendo-decrescendo murmur at LUSB ● Increase Murmur: (Valsalva and standing) & Exertion reduce venous return ● Decrease Murmur: (squatting or lying down) increase venous return

Cardiovascular : Clinical Intervention, Abdominal aortic aneurysm

● Surgical repair → definitive management ○ Endovascular stent graft or open repair ○ > 5.5 cm or > 0.5 cm expansion in 6 months → immediate surgical repair (even if asymptomatic), or symptomatic patients, or patients with acute rupture ○ > 4.5 cm → vascular surgeon referral ○ 3 - 4.5 cm → Monitor by US every 6 months ● Beta blockers reduce shearing forces

Cardiovascular : Clinical Intervention, Abdominal aortic aneurysm Ultrasound remains the definitive test for initial diagnosis and screening. Imaging with computed tomography angiography or magnetic resonance angiography is used for anatomical mapping to assist with operative planning. Patients presenting with a ruptured aneurysm require emergent repair. For patients with symptomatic aortic aneurysms, repair is indicated regardless of diameter. o Open aneurysm repair if aneurysm is causing symptoms or is larger than about 2 inches (roughly 5.0 to 5.5 centimeters) or is enlarging under observation. o Endovascular stent graft if pt is high risk for open aneurysm repair,

● Surgical repair → definitive management ○ Endovascular stent graft or open repair ○ > 5.5 cm or > 0.5 cm expansion in 6 months → immediate surgical repair (even if asymptomatic), or symptomatic patients, or patients with acute rupture ○ > 4.5 cm → vascular surgeon referral ○ 3 - 4.5 cm → Monitor by US every 6 months ● Beta blockers reduce shearing forces

Gastrointestinal/Nutritional : Clinical Intervention, Hepatocellular carcinoma Hepatocellular Carcinoma o Most common primary liver cancer o Most common cause: chronic HBV/HCV cirrhosis o Rapidly ↑ ascites o Bloody ascitic fluid ↑ AFP Alpha-fetoprotein is elevated

● Surgical resection if confined to a lobe and not associated with cirrhosis g. Tx = (1) surgical resection, (2) liver transplant, (3) chemoSorafenib

Endocrinology : Scientific Concepts, Medullary thyroid carcinoma Medullary thyroid cancer (MTC) is a form of thyroid carcinoma which originates from the parafollicular cells (C cells), which produce the hormone calcitonin.

● This is a type of thyroid cancer MC associated with MEN 2 ● Characteristics- More aggressive, arises from parafollicular C cells secrete calcitonin ● METS- Local cervical lymph node occurs early in disease. Distant METS occurs late (brain, bone, liver adrenal medulla) ● Prognosis- Poorer prognosis. ● Management- Total Thyroidectomy (including a complete neck dissection of all lymph nodes ) ○ Calcitonin levels used to monitor- if residual disease is present after tx or for recurrence ***Calcitonin decreases calcium levels in the blood***

Endocrinology : Scientific Concepts, Medullary tyroid carcinoma MC associated with MEN 2, aggressive cancer arises from parafollicular C cells secrete calcitonin

● This is a type of thyroid cancer ● MC associated with MEN 2, aggressive cancer arises from parafollicular C cells secrete calcitonin ● METS- Local cervical lymph node occurs early in the disease. Distant METS occurs late (brain, bone, liver adrenal medulla) ● Prognosis- Poorer prognosis. ● Management- Total Thyroidectomy (including a complete neck dissection of all lymph nodes and surrounding fatty tissues on the ipsilateral side) ○ Calcitonin levels used to monitor- if residual disease is present after tx or for recurrence ***Calcitonin decreases calcium levels in the blood***

Gastrointestinal/Nutritional : Diagnosis, Peptic ulcer disease

● Upper Endoscopy with biopsy

Gastrointestinal/Nutritional : Diagnostic Studies, Mallory-Weiss tear Mallory-Weiss syndrome or gastro-esophageal laceration syndrome refers to bleeding from a laceration in the mucosa at the junction of the stomach and esophagus. o patient will have a history of drinking alcohol and forceful vomiting o Complaining of hematemesis o Diagnosis is made by upper endoscopy Caused by an incomplete tear in the esophagus mucosa

● Upper endoscopy is test of choice → superficial longitudinal mucosal erosions in the gastro-esophagial junction

Gastrointestinal/Nutritional : Diagnosis, Esophageal Cancer

● Upper endoscopy with biopsy

Gastrointestinal/Nutritional : Scientific Concepts, Hepatic carcinoma

● Usually occurs with chronic liver disease or cirrhosis. Hepatitis B and C strongly predisposes to the development of chronic liver disease and subsequent development of hepatic carcinoma ●

15. Dermatology : Diagnostic Studies, Tinea corporis

● potassium hydroxide (KOH) smear microscopy hyphae Wood's lamp: green fluorescence if due to Microsporum


Related study sets

GEO Chapters 1 & Chapter 2 - Minerals & The Rock Cycle = Metamorphic, Igneous and Sedimentary Rocks.

View Set

AP Government: Landmark Supreme Court Cases long version

View Set

Ch. 9 section 4.3, 4.4, 4.5 Virtual Network Devices

View Set